EMT Study Questions Ch. 21-39

Réussis tes devoirs et examens dès maintenant avec Quizwiz!

Which of the following is a reason that the vehicle doors are removed to extricate the patient rather than pulling the patient through an open​ window? A. To avoid being cut by glass B. To avoid criticism from bystanders C. To allow for proper spinal immobilization D. To make the interior of the vehicle less accessible

. To allow for proper spinal immobilization

You are on the scene of a​ mass-casualty incident in which the grandstand at a race track has collapsed. As the triage​ officer, you have encountered a​ 14-year-old male whose leg has been amputated above the knee. He is responsive to verbal​ stimuli, and he has an open​ airway, shallow rapid​ breathing, and a​ weak, rapid carotid pulse. This patient would be classified as a Priority​ ________ for treatment and transport. A. 4 B. 1 C. 3 D. 2

1

Traditionally an APGAR score is taken at what time intervals after​ birth? A. 1 and 5 minutes B. 5 and 10 minutes C. 1 and 6 minutes D. 2 and 7 minutes

1 and 5 minutes

When using the rule of palm to estimate the approximate body surface area burned, the patient's palm equals about what percentage of the body's surface area? a. 1 percent b. 2 percent c. 5 percent d. 3 percent

1 percent

When performing chest compressions on a pregnant patient that has gone into cardiac​ arrest, what change do you need to make in the placement of your​ hands? A. 0.5 to 1 inch lower on the sternum B. 0.5 to 1 inch higher on the sternum C. 1 to 2 inches higher on the sternum D. 1 to 2 inches lower on the sternum

1 to 2 inches higher on the sternum

When clamping the umbilical​ cord, the clamp closest to the baby should be approximately​ ________ inch(es) from the​ infant's body. A. 12 B. 1 C. 10 D. 3

10

Safety studies have shown that drivers of other vehicles do not usually see or hear ambulances or other emergency vehicles until they are within​ ________ feet of it. A. 600 feet B. 500 feet C. 200 feet D. 100 feet

100 feet

What is an​ appropriate-sized landing zone for a​ helicopter? A. 75 feet by 75 feet B. 25 feet by 25 feet C. 50 feet by 50 feet D. 100 feet by 100 feet

100 feet by 100 feet

When actively rewarming a frostbitten extremity in warm​ water, you should change the water when the temperature falls below​ ________ degrees°F. A. 115 to 120 B. 80 to 90 C. 85 to 95 D. 100 to 105

100 to 105

A normal heart rate for a newborn​ is: A. 160 to 200 beats per minute. B. 120 to 160 beats per minute. C. 60 to 100 beats per minute. D. 80 to 120 beats per minute.

120 to 160 beats per minute.

What is the Glasgow Coma Score​ (GCS) measurement of altered mental​ status, which according to CDC guidelines necessitates transport to a trauma​ center? A. 8 B. 14 C. 13 D. 12

13

What is the Glasgow Coma Scale​ (GCS) of your adult male patient who has fallen off a​ horse, has his eyes​ open, can follow your commands to squeeze his​ hands, but is confused about what happened and his​ whereabouts? A. 14 B. 13 C. 15 D. 12

14

According to the Rule of Nines for infants and young children, the patient's head and neck account for what percentage of the total body surface area? a. 13.5 percent b. 14 percent c. 9 percent d. 18 percent

18 percent

Your patient is a 40-year-old man who was burned when he spilled gasoline on his pants as he was standing near the pilot light of his hot water heater. He has partial thickness burns from his feet to just above his knees, and circumferentially around both legs. Using the rule of nines, which of the following most accurately represents the extent of body surface area burned? A. 18 percent B. 9 percent C. 36 percent D. 4.5 percent

18 percent

The elderly population is at least​ ________ times as likely to use the EMS system as patients in other age groups. A. 5 B. 10 C. 2 D. 0.5

2

Where should a constricting band be placed to reduce lymphatic flow after a venomous snake​ bite? A. 2 inches above the bite B. 2 inches below the bite C. 2 inches above and 2 inches below the bite D. None of the above

2 inches above and 2 inches below the bite

A respiratory rate of less than​ ________ in infants is a significant finding and indicates a critical​ patient, who should be immediately transported to a trauma center if secondary to trauma. A. 30 B. 18 C. 25 D. 20

20

What is the height from which an adult fall would meet trauma triage criteria set forth by the​ CDC? A. 25 feet B. 10 feet C. 20 feet D. 15 feet

20 feet

Which of the following respiratory rate ranges is normal for a child between 1 and 3 years of​ age? A. 12 to 20 breaths per minute B. 40 to 60 breaths per minute C. 25 to 50 breaths per minute D. 20 to 30 breaths per minute

20 to 30 breaths per minute

Of the older patients seen in the emergency department due to a​ fall, ________% will die within 1 year. A. 10 B. 50 C. 35 D. 25

25

When the body is in​ water, how many times faster does it lose heat than when it is in still​ air? A. 25 B. 10 C. 2 D. 100

25

The chest cavity can hold up to ________ liter(s) of blood in an adult, leading to the possibility of massive internal hemorrhage without any external blood loss. A. 5 B. 0.5 C. 3 D. 1

3

How long does a typical hemodialysis treatment​ last? A. 3 to 4 hours B. 1 to 2 hours C. 10 to 12 hours D. 7 to 8 hours

3 to 4 hours

A body mass index​ (BMI) above​ ________ is considered obese. A. 35 B. 40 C. 30 D. 25

30

Obesity is defined as a body mass index greater​ than: A. 40. B. 25. C. 35. D. 30.

30

In a patient with extreme hypothermia who appears to be in cardiac​ arrest, you should assess the pulse for how​ long? A. 30 to 45 seconds B. 5 to 10 seconds C. 20 to 30 seconds D. 15 to 20 seconds

30 to 45 seconds

By​ definition, a premature infant is one who is born before how many weeks of the​ pregnancy? A. 28 B. 37 C. 40 D. 34

37

Jarl is a male who is 5 foot 7 inches tall and weighs 250 pounds. What is​ Jarl's body mass index​ (rounded to the nearest​ tenth)? A. 36.5 B. 35.2 C. 38.4 D. 39.1

39.1

Which patient is experiencing visceral pain? A. 24yearold male complaining of severe left flank pain B. 19yearold female complaining of severe cramps in the lower abdominal quadrants C. 45yearold female complaining of abdominal pain "all over" D. 28yearold male with rebound tenderness

45yearold female complaining of abdominal pain "all over"

An​ infant's birth weight is considered low if it is less than​ ________ pounds. A. 3.5 B. 5.5 C. 4.5 D. 2.5

5.5

Normal maternal blood loss during delivery of an infant usually does not exceed how​ much? A. ​1,000 cc B. 500 cc C. 250 cc D. 100 cc

500 cc

ICS systems recognize that the manageable span of control​ is: A. 12 people. B. 18 people. C. 6 people. D. 24 people.

6 people.

A normal heart rate for a​ 13- to​ 18-year-old patient at rest​ is: A. 60 to 105 beats per minute. B. 50 to 95 beats per minute. C. 90 to 135 beats per minute. D. 120 to 165 beats per minute

60 to 105 beats per minute.

How many cervical vertebrae are​ there? A. 5 B. 12 C. 4 D. 7

7

When submerged in​ water, biological death may be delayed if the water temperature is below ​________degrees°F. A. 50 B. 32 C. 98.6 D. 70

70

You are on the scene of a church bus​ roll-over collision. There are 30 passengers involved. As triage​ officer, which one of the following patients should you assign the highest​ priority? A. ​30-year-old man with multiple lower extremity fractures who is​ awake, has a strong radial​ pulse, and is complaining of severe pain B. ​70-year-old woman who is awake but having difficulty breathing and has absent lung sounds on the right side C. Unresponsive​ 9-year-old male with an open skull fracture who is in cardiac arrest D. ​35-year-old pregnant woman with a cut on her​ head, neck​ pain, and numbness and tingling in her arms

70-year-old woman who is awake but having difficulty breathing and has absent lung sounds on the right side

You are assessing a newborn patient 1 minute after delivery. You notice the patient has blue extremities with a pink​ trunk, a pulse of​ 120, and strong crying with good movement of all extremities. What is the​ newborn's APGAR​ score? A. 8 B. 7 C. 10 D. 9

9

A diastolic blood pressure over​ ________ mmHg is considered hypertensive. A. 65 B. 80 C. 50 D. 90

90

According to CDC​ guidelines, a systolic blood pressure​ (BP) of less than​ ________ indicates a patient should be transported to a trauma center. A. 80 B. 110 C. 90 D. 100

90

For which of the following patients should the EMT carefully continue to monitor the patient's ventilatory status throughout treatment and transport due to the greatest risk of respiratory failure? A A 17-year-old male with a blistering sunburn on his face B A 34-year-old male who opened the radiator of his car and had hot fluid spray on his chest, resulting in redness and pain in an area about the size of the patient's hand C A 28-year-old male who spilled a strong industrial acid on his legs D A 16-year-old male whose shirt caught on fire, resulting in circumferential burns of his chest

A 16-year-old male whose shirt caught on fire, resulting in circumferential burns of his chest

Which of the following patients has the greatest likelihood of being cared for in a burn center? a. A 45-year-old man who has a full thickness burn about 3 inches long by 1/2 inch wide on his posterior arm from backing into a barbecue grill b. A 30-year-old woman who has deep partial thickness burns on her hand and arm as a result of spilling hot cooking oil on herself c. A 12-year-old male with a superficial partial thickness burn involving his forearm as a result of making a torch by lighting aerosol from a can of hairspray d. A 16-year-old female who came into contact with a motorcycle exhaust pipe and has a full thickness burn on her leg about 2 inches in diameter

A 30-year-old woman who has deep partial thickness burns on her hand and arm as a result of spilling hot cooking oil on herself

What is the significance of a​ child's body surface area being larger in proportion to body​ mass? A. Labored or distressed breathing is more obvious from a distance. B. Abdominal organs are less protected. C. A child is more prone to heat loss. D. The extent of a burn is estimated differently

A child is more prone to heat loss.

Which of the following statements is NOT true? A. A comminuted break is when a bone is broken in only one place. B. An angulated break is when the broken bone is bent at an angle. C. A fracture is any break in a bone. D. A greenstick break is an incomplete break in a bone.

A comminuted break is when a bone is broken in only one place.

Which of the following is NOT a type of avulsion? A. The skin is partially torn away from the foot. B. A finger is cut off with a butcher's saw. C. An ear is partially torn away from the head. D. The skin is stripped off the hand, like removing a glove.

A finger is cut off with a butcher's saw.

Which of the following BEST describes an avulsion? a An injury caused by a sharp, pointed object b A flap of skin that is partially or completely torn away from the underlying tissue c The epidermis that is scraped away by a rough surface d An accumulation of blood beneath the skin, resulting in swelling

A flap of skin that is partially or completely torn away from the underlying tissue

What is continuous ambulatory peritoneal​ dialysis? A. A gravity exchange process for peritoneal dialysis in which a bag of dialysis fluid is raised above the level of an abdominal catheter to fill the abdominal cavity and lowered below the level of the abdominal catheter to drain the fluid out B. A gravity exchange process for peritoneal dialysis in which a bag of dialysis fluid is lowered below the level of an abdominal catheter to fill the abdominal cavity and raised above the level of the abdominal catheter to drain the fluid out C. A type of dialysis that is performed while walking D. A mechanical process for peritoneal dialysis in which a machine fills and empties the abdominal cavity of dialysis solution

A gravity exchange process for peritoneal dialysis in which a bag of dialysis fluid is raised above the level of an abdominal catheter to fill the abdominal cavity and lowered below the level of the abdominal catheter to drain the fluid out

How would you define​ anemia? A. A lack of a normal number of red blood cells in the circulation B. The process by which toxins and excess fluid are removed from the body by a medical system independent of the kidneys C. The loss of the​ kidneys' ability to filter the blood and remove toxins and excess fluid from the body D. An inherited disease in which a genetic defect in the hemoglobin results in abnormal structure of the red blood cells

A lack of a normal number of red blood cells in the circulation

Concerning behavioral​ emergencies, which of the following statements is true​? A. It is relatively easy to determine if the underlying cause of a behavioral emergency is related to drug abuse. B. A patient who does not respond to crisis management techniques may be restrained for transport to a psychiatric facility. C. Only a licensed psychiatrist can apply crisis management techniques with a patient having a behavioral emergency. D. A diabetic problem never mimics signs of a behavioral emergency.

A patient who does not respond to crisis management techniques may be restrained for transport to a psychiatric facility.

Which one of the following patients would justify the need to directly transport to a trauma center based on special patient​ considerations? A. An unlicensed teenage driver who has a pulse rate of 120 after a MVC and a 4 minute EMS response B. A pregnant female in the third trimester who is spotting​ (showing small amounts of vaginal​ bleeding) following a low mechanism of injury MVC C. An​ end-stage renal disease​ (ERSD) patient who tripped and fell and is complaining of shoulder pain on the same side as his shunt D. An elderly patient on anticoagulants who slipped out of her wheelchair and is complaining of pelvic pain

A pregnant female in the third trimester who is spotting​ (showing small amounts of vaginal​ bleeding) following a low mechanism of injury MVC

Which of the following most likely indicates respiratory failure in a pediatric​ patient? A. A bounding pulse B. A pulse of 40 beats per minute C. An irregular pulse D. A pulse of 190 beats per minute

A pulse of 40 beats per minute

When heading back to​ quarters, on what should you place your​ emphasis? A. Documentation B. Refueling the ambulance for the next call C. Beginning to reorganize the ambulance D. A safe return

A safe return

Which of the following BEST describes the compartment syndrome? A. A non-life-threatening condition caused by orthopedic injuries in which blood flow is stopped when the bone ends compress the blood vessels B. A life-threatening condition caused by trapping the blood flow by a fracture when the bone ends cut the blood vessels C. A serious condition caused by the bleeding and swelling from a fracture or crush injury that becomes so strong that the body can no longer perfuse the tissues against that pressure D. A serious condition caused by the amount of equipment that must be carried in the ambulance compartments

A serious condition caused by the bleeding and swelling from a fracture or crush injury that becomes so strong that the body can no longer perfuse the tissues against that pressure

Which of the following is a possible indication of abuse or​ neglect? A. Concern for the​ patient's condition B. Any injuries to the extremities C. A story that is inconsistent with the injuries D. Two or more calls to EMS within a year

A story that is inconsistent with the injuries

What is the definition of multiple​ trauma? A. Multiple injuries that affect more than one body system. B. A trauma in which there are multiple casualties. C. A trauma in which the patient has more than one serious injury. D. A trauma that requires the response of multiple agencies.

A trauma in which the patient has more than one serious injury.

An outbreak of which of the following illnesses would be LEAST likely to occur​ naturally? A. Anthrax B. Smallpox C. Venezuelan encephalitis D. Botulism

A. Anthrax

If the steering wheel must be displaced during extrication and the air bag has not​ deployed, which of the following actions is recommended by air bag​ manufacturers? A. Disconnect the battery cables. B. Secure a long backboard across the steering wheel to deflect the air bag from the patient should it deploy. C. Firmly strike the hub of the steering wheel with a mallet to deactivate the air bag before displacing the steering wheel. D. No action is​ required; only frontal impact will deploy air bags.

A. Disconnect the battery cables.

The formula dose multiplied by time represents which of the​ following? A. Exposure B. Lethal dose C. Critical level D. Concentration

A. Exposure

Which of the following is the BEST protection for an​ EMT's upper body during an extrication​ operation? A. Firefighting turnout coat B. Thick fireproof blanket draped around the shoulders C. Nylon jacket with asbestos lining D. Thick wool coat

A. Firefighting turnout coat

Which of the following types of radiation emits the most powerful​ rays? A. Gamma B. Alpha C. Delta D. Beta

A. Gamma

You are on the scene of a probable terrorist attack with several other​ agencies, both local and federal. In order to operate in an efficient​ manner, all agencies should follow​ the: A. Incident Command System. B. Interagency Communication System. C. Interagency Command System. D. Incident Control System

A. Incident Command System.

Which of the following is an unsafe behavior that contributes to most injuries at the scene of vehicle​ collisions? A. Not wearing proper protective gear B. Using improvised tools C. Unknowingly walking into hazardous materials situations D. Antagonizing bystanders

A. Not wearing proper protective gear

You respond to the scene of a motor vehicle collision involving a small pickup truck that struck the rear of a moving​ tractor-trailer. The driver of the​ tractor-trailer is walking around and complains of neck pain. The driver of the pickup truck is​ conscious, but cannot exit the vehicle because the door is stuck and the steering wheel is displaced onto his lap. The passenger of the pickup truck is sitting upright in the passenger​ seat, is​ unconscious, but has radial pulses. Access to the passenger is not obstructed. A fourth​ patient, who was sitting in the pickup truck​ bed, is found about 10 feet away and responds to commands appropriately. Assuming your ambulance is the only emergency vehicle on the scene​ currently, which of the following patients should be packaged for transport first by the next arriving​ ambulance? A. Passenger in the pickup​ truck's front seat B. Pickup truck driver C. Tractor trailer driver D. Passenger ejected from the pickup truck bed

A. Passenger in the pickup​ truck's front seat

Which of the following phases of extrication comes before the other three​ phrases? A. Preparing for the rescue B. Sizing up the situation C. Recognizing and managing hazards D. Gaining access to the patient

A. Preparing for the rescue

Biological warfare is MOST effective when the agent can gain access through which of the following​ routes? A. Respiratory tract B. Blood C. Skin D. Gastrointestinal tract

A. Respiratory tract

Stabilizing a vehicle is unnecessary when a vehicle is on its roof​ with: A. all roof supports collapsed. B. the trunk lid in contact with the ground. C. the vehicle horizontal and resting entirely on the roof. D. the front edge of the hood in contact with the ground.

A. all roof supports collapsed.

Terrorist incidents typically involve all of the following agents except​: A. allergens. B. biological agents. C. nuclear agents. D. explosives.

A. allergens.

When placing​ cribbing: A. always​ squat, so you can move away quickly. B. kneel on both knees for stabilization. C. kneel or squat as necessary. D. kneel on one knee only so you can move away quickly.

A. always​ squat, so you can move away quickly.

Tertiary injuries from a blast​ include: A. extremity fractures. B. lung injuries. C. bowel perforations. D. impaled objects.

A. extremity fractures.

The type of radiation that requires lead shielding for your protection​ is: A. gamma. B. beta. C. alpha. D. delta.

A. gamma.

When disconnecting a battery cable to disrupt a​ vehicle's electrical​ system, the EMT should disconnect​ the: A. negative cable from the battery. B. positive cable from the alternator. C. negative cable from the alternator. D. positive cable from the battery.

A. negative cable from the battery.

When controlling the scene of an emergency involving damaged power poles and downed electrical​ wires, the EMT must remember​ to: A. request appropriate personnel to cut the power source. B. use nonconductive implements to make a safe path to victims. C. render the scene safe before entering the danger zone. D. use​ double-thickness gloves to insulate the body from electrocution.

A. request appropriate personnel to cut the power source.

When attempting to extinguish a fire inside the passenger​ compartment, it is important​ to: A. use the extinguishing agent sparingly to avoid creating a cloud of powder. B. apply the extinguishing agent liberally to speed up the extinguishing process. C. aim the nozzle of the extinguisher away from the patient to avoid hitting the patient. D. resist the urge to extinguish the fire and focus on extricating any patients.

A. use the extinguishing agent sparingly to avoid creating a cloud of powder.

To assess skin temperature for a possibly hypothermic​ patient, the EMT should place the back of his hand against which part of the​ patient's body? A. Cheek B. Inner wrist C. Forehead D. Abdomen

Abdomen

What type of injury is LEAST likely in a pediatric patient who falls 5​ feet? A. Abdominal B. Extremities C. Head D. Upper neck

Abdominal

You arrive on the scene to find an approximately 60 year old male patient writhing on the floor. He is complaining of a tearing pain radiating to his lower back. He has absent femoral pulses and has a pulsatile mass just superior to his umbilicus. You suspect which of the following conditions? A. Ruptured appendix B. Myocardial infarction C. Acute pancreatitis D. Abdominal aortic aneurysm

Abdominal aortic aneurysm

Your patient is an​ 80-year-old male complaining of lower back pain that radiates through to the abdomen. He describes the pain as​ "ripping" in nature. He is​ pale, diaphoretic, and​ anxious, and has a blood pressure of​ 210/100 mmHg, a pulse of 112 beats per​ minute, and a respiratory rate of 20 breaths per minute. Which of the following conditions is MOST​ likely? A. Abdominal aortic aneurysm B. Bowel obstruction C. Diverticulitis D. Gallbladder disease

Abdominal aortic aneurysm

Which of the following statements concerning respiratory problems in pediatric patients is​ TRUE? A. About​ one-third of all pediatric trauma deaths are related to airway or respiratory compromise. .B. Respiratory disease is unrelated to cardiac arrest. C. Treating the respiratory system will likely not have an effect on the heart. D. Most respiratory disorders in children are of minimal concern

About​ one-third of all pediatric trauma deaths are related to airway or respiratory compromise.

The condition in which the placenta separates from the uterine wall is known as which of the​ following? A. Placenta previa B. Preeclampsia C. Abruptio placentae D. Ectopic pregnancy

Abruptio placentae

By what route does nitroglycerin spray enter the​ body? A. Injection B. Ingestion C. Absorption D. Inhalation

Absorption

When a substance enters the body through unbroken​ skin, it has entered the body by which of the following​ routes? A. Absorption B. Injection C. Ingestion D. Consumption

Absorption

Which of the following medications does NOT interfere with the​ blood-clotting process? A. Acetaminophen B. Clopidogrel C. Aspirin D. Warfarin

Acetaminophen

Which of the following communication strategies should be used in dealing with a patient with a behavioral​ emergency? A. Acknowledge the​ patient's feelings B. Interrupt the patient if he appears to be going off on a tangent about something not concerning the immediate situation. C. Show deep sympathy for the​ patient's predicament and let him know he​ can't control the things that are happening to him. D. Stay 2 to 3 inches from the patient at all times to make him feel secure.

Acknowledge the​ patient's feelings

Which of the following BEST describes the origins of chronic obstructive pulmonary disease​ (COPD)? A. Either a congenital or acquired disease B. Neither a congenital nor acquired disease C. Congenital disease D. Acquired disease

Acquired disease

Which of the following is a trade name for activated​ charcoal? A. Actifed B. Mesquite C. Charcoal D. Actidose

Actidose

Why would an EMT give activated charcoal to a patient who overdosed on the medication​ Soma? A. Activated charcoal coats the intestinal​ tract, preventing absorption by the body. B. Activated charcoal is the antidote to Soma poisoning. C. Activated charcoal prevents the patient from vomiting. D. Activated charcoal binds to the​ Soma, reducing absorption by the body.

Activated charcoal binds to the​ Soma, reducing absorption by the body.

What is the difference between acute and chronic renal​ failure? A. Chronic renal failure is​ life-threatening, but acute renal failure is not. B. Acute renal failure is​ life-threatening, but chronic renal failure is not. C. Chronic renal failure is​ sudden, while acute renal failure develops gradually. D. Acute renal failure is​ sudden, while chronic renal failure develops gradually.

Acute renal failure is​ sudden, while chronic renal failure develops gradually.

Which of the following is NOT part of the treatment for a​ 15-year-old female who has swallowed drain​ cleaner? A. Transport the patient. B. Call medical control. C. Maintain the airway. D. Administer activated charcoal.

Administer activated charcoal.

Your patient is a​ 3-year-old female who has taken an unknown quantity of aspirin. She is conscious and alert. Which of the following orders is likely to be given by medical​ control? A. Administer syrup of ipecac. B. Administer one to two glasses of milk. C. Administer activated charcoal. D. Induce vomiting by placing your finger in the​ child's throat.

Administer activated charcoal.

Your patient is an infant who has just been delivered 3 weeks before her due date. She is breathing​ adequately, has a heart rate of 140 beats per​ minute, and has cyanosis of her face and chest. Which of the following interventions should be done​ first? A. Administer​ blow-by oxygen. B. Begin CPR with a compression rate of​ 120/minute. C. Perform ventilations with a​ bag-valve-mask device and supplemental oxygen. D. Place a neonatal nonrebreather mask on the​ infant's face.

Administer​ blow-by oxygen.

What is the most important treatment for a patient who has inhaled a​ poison? A. Administer activated charcoal. B. Administer the correct antidote. C. Administer​ high-concentration oxygen. D. Administer epinephrine via​ auto-injector.

Administer​ high-concentration oxygen.

Which age group is most heavily influenced by their peers and what their peers​ think? A. School age B. Adolescents C. Toddlers D. Infants

Adolescents

When a poison attaches to the chemical structure of activated​ charcoal, which of the following BEST describes this mechanism of​ action? A. Inactivation B. Absorption C. Incorporation D. Adsorption

Adsorption

You are assessing a​ 27-year-old female who is 9 months pregnant with her first child. She has been having contractions for the past 6 hours. As you are about to assist her to your​ cot, she asks you to wait because she feels the need to use the bathroom first. Which of the following is the BEST course of​ action? A. Advise the patient that this could be an indication that the baby is ready to be born and you need to check to see if the​ baby's head is visible. B. Advise the patient that this could be a sign of a serious​ complication, have her lie on her left​ side, apply​ high-concentration oxygen, and transport immediately. C. Tell the patient she needs to be transported immediately and using the bathroom will have to wait until she arrives at the hospital. D. Allow the patient to use the bathroom as it will make​ transport, and also​ delivery, more comfortable.

Advise the patient that this could be an indication that the baby is ready to be born and you need to check to see if the​ baby's head is visible.

Which of the following is NOT a consequence of​ hypoperfusion? A. Cells are not supplied with nutrients. B. Aerobic metabolism will continue in the cell. C. Cellular waste products are not removed. D. Cells are not supplied with oxygen.

Aerobic metabolism will continue in the cell.

​________ patients are at high risk for acquiring the inherited disorder called sickle cell anemia. A. African American B. Native American C. Eastern European D. Hispanic

African American

You have transported a patient to the emergency department and are in the process of cleaning the ambulance while your partner is completing all required paperwork. At what point is the final phase of an ambulance call​ complete? A. As soon as you put the stretcher back in the ambulance with a clean sheet and you notify your dispatch center you are available B. After your partner completes all required paperwork and you have the ambulance completely cleaned and restocked C. As soon as you get back to the station and notify your dispatch center you are in quarters D. As soon as your partner completes the patient care report and you are restocking the supplies used during the call

After your partner completes all required paperwork and you have the ambulance completely cleaned and restocked

You are called to the residence of a​ 78-year-old widow who lives alone with no children. She was found by neighbors on the floor of the living room. She has fallen and fractured her left humerus. She is disoriented and responds to verbal stimuli only. She has a blood pressure of​ 78/40 and a pulse of 48. She has a history of high blood pressure and takes medication for it. She also takes medication for high cholesterol and a medication to slow down a fast heart rate. Emergency Medical Responders find the medication bottles in the medicine cabinet and they are empty. Your partner suspects that she accidentally overdosed on her medications. Do you agree or​ disagree? A. Agree. The vital signs agree with this hypothesis. B. Agree. It is obvious that the broken arm is causing the decreased heart rate. C. Disagree. The vital signs indicate that she has not taken her medication in several days. D. Disagree. The​ patient's vital signs are related to her broken arm.

Agree. The vital signs agree with this hypothesis.

You arrive on the scene to find a scuba diver on board a boat slumped over in the​ captain's chair with frothy blood in his mouth. The captain states that the diver was down no more than 15 feet when he ascended rapidly and called for help. Which of the following is most​ likely? A. The bends B. Air embolism C. ​Caisson's disease D. Decompression sickness

Air embolism

You respond to a boat dock for a diving injury. You find a​ 24-year-old male patient unresponsive with frothy blood in the mouth and lung sounds absent on the right side. The​ patient's friends state they were diving when he came out of the water complaining of chest pains and then collapsed. What condition do you​ suspect? A. Air embolism B. Decompression sickness C. Myocardial infarction D. Near drowning

Air embolism

While assessing a patient with a laceration to the​ neck, the EMT must be aware that which of the following conditions may​ develop? A. Deep vein embolus B. Air thrombosis C. Air embolus D. Deep vein thrombosis

Air embolus

While assessing a patient with partial thickness burns to his chest and neck, what should be your highest priority (even if there are no symptoms presently)? A. None of the choices B. Airway C. Hypothermia D. Bleeding

Airway

Which of the following is part of the appearance component of the Pediatric Assessment​ Triangle? A. Jaundice B. Skin color C. Alertness and verbal response D. Pupils

Alertness and verbal response

Which patient would MOST likely benefit from the administration of activated​ charcoal? A. ​48-year-old man who attempted suicide by drinking Drano® B. Alert​ 16-year-old female who overdosed on Tylenol® C. Unresponsive​ 28-year-old female who overdosed on sleeping pills D. ​28-year-old male who is suffering from food poisoning after eating eggs contaminated with salmonella

Alert​ 16-year-old female who overdosed on Tylenol®

At which of the following locations is an EMT likely to encounter a patient using an advanced medical​ device? A. Private residence B. Specialty care center C. Nursing home D. All of the above

All of the above

A​ 44-year-old male involved in a collision at 50 mph struck the windshield of his vehicle with his face. Which of the following injuries should you prepare to​ treat? A. Cervical spine trauma B. Airway obstruction C. Brain injury D. All of the above

All of the above

Carbon monoxide poisoning should be suspected when a patient has been in an enclosed area and has which of the following signs​ and/or symptoms? A. ​Flu-like symptoms B. Vomiting and altered mental status C. ​Headache, dizziness, and nausea D. All of the above

All of the above

During a power​ failure, how should the EMT verify adequate ventilations are being administered to an unresponsive patient with a tracheostomy tube using a​ bag-valve mask? A. Ensure a good connection between the​ bag-valve mask and the tracheostomy tube. B. Observe for chest rise and fall. C. Observe for improving skin color. D. All of the above

All of the above

In which of the following settings should the EMT be alert to the possibility of hazardous materials​ emergencies? A. Farm service agencies B. Manufacturing industries C. Shipping ports D. All of the above

All of the above

In which of the following situations should the mother be transported​ immediately? A. Delivery is not imminent and there is about 500 cc of​ painless, bright red bleeding. B. The mother experiences a​ severe, tearing sensation in her abdomen during labor and the outline of the fetus can be felt through the abdominal wall. C. The​ infant's arm is the presenting part. D. All of the above

All of the above

Obesity increases the risk for which of the following health​ problems? A. Liver disease B. Respiratory problems C. Type 2 diabetes D. All of the above

All of the above

The​ kidney's major​ function(s) include(s): A. maintaining blood volume. B. excreting urea. C. balancing electrolytes. D. All of the above

All of the above

Which of the following circumstances may result in​ hypoperfusion? A. Blood vessels are dilated. B. There is external bleeding. C. The heart is damaged. D. All of the above

All of the above

Which of the following conditions can mimic the signs of alcohol​ intoxication? A. Head injuries B. Diabetes C. Epilepsy D. All of the above

All of the above

Which of the following conditions should be present before the umbilical cord is​ cut? A. The cord is no longer pulsating. B. The cord is clamped in two places. C. The infant is breathing on his or her own. D. All of the above

All of the above

Which of the following effects may occur due to chronic abuse of​ alcohol? A. Abnormal blood sugar levels B. Poor nutrition C. Potential for gastrointestinal bleeding D. All of the above

All of the above

Which of the following factors suggest that a patient is at risk for​ suicide? A. Recent emotional trauma B. Sudden improvement in depression C. Alcohol and drug abuse D. All of the above

All of the above

Which of the following injuries may produce distended neck veins (JVD)? A. Tension pneumothorax B. Cardiac tamponade C. Traumatic asphyxia D. All of the above

All of the above

Which of the following injuries requires the use of an occlusive dressing? A. Open wound to the abdomen from which a loop of intestine is protruding B. Open wound to the neck C. Open wound to the chest D. All of the above

All of the above

Which of the following injuries requires the use of an occlusive dressing? a An open wound to the neck b An open wound to the abdomen from which a loop of intestine is protruding c An open wound to the chest d All of the above

All of the above

Which of the following is a cause of gynecological emergencies? A. Sexual assault B. Softtissue trauma to the external genitalia C. Disorders of the female reproductive organs D. All of the above

All of the above

Which of the following is a cause of gynecological​ emergencies? A. ​Soft-tissue trauma to the external genitalia B. Sexual assault C. Disorders of the female reproductive organs D. All of the above

All of the above

Which of the following is a complication of bone fractures? A. Hemorrhage B. Swelling C. Nerve damage D. All of the above

All of the above

Which of the following is a concern when caring for the patient with abdominal pain? A. Patient comfort B. Shock C. Airway management D. All of the above

All of the above

Which of the following is a consideration in determining a burn's severity? A. Other illnesses or injuries the patient may have B. Body surface area involved in the burn C. The type of agent that caused the burn D. All of the above

All of the above

Which of the following is a feature of a good disaster​ plan? A. It is specific to the geographical location. B. It has been practiced in mock disasters. C. It is based on the availability of resources in a particular area. D. All of the above

All of the above

Which of the following is a function of the rehabilitation sector in a hazardous materials​ incident? A. Performing medical assessment of rescue crews B. Providing for hydration of rescue crews C. Taking baseline vital signs before rescuers enter the operations area D. All of the above

All of the above

Which of the following is a major function of the skin? A. Excretion of wastes B. Protection from the environment C. Temperature regulation D. All of the above

All of the above

Which of the following is a sign of possible brain​ injury? A. Irregular breathing pattern B. Projectile vomiting C. Deep laceration of the scalp D. All of the above

All of the above

Which of the following is a sign or symptom of withdrawal from​ alcohol? A. Seizures B. Sweating C. Hallucinations D. All of the above

All of the above

Which of the following is an advantage of removing the roof of a vehicle that has sustained major damage in a​ collision? A. The EMT can better access the patients. B. It makes the entire interior of the vehicle accessible. C. It creates a large exit through which to remove the patient. D. All of the above

All of the above

Which of the following is of concern with a puncture wound? A. An object that remains impaled in the body B. Hidden internal bleeding with minimal external bleeding C. A strong possibility of contamination D. All of the above

All of the above

Which of the following is often associated with​ water-related emergencies? A. Alcohol use B. Cardiac arrest C. Hypothermia D. All of the above

All of the above

Which of the following is true concerning lacerations? A. They may indicate deeper underlying tissue damage. B. They may be caused by penetrating trauma. C. They may be caused by blunt trauma. D. All of the above

All of the above

Which of the following is true concerning lacerations? A. They may indicate deeper underlying tissue damage. B. They may be caused by penetrating trauma. C. They may be caused by blunt trauma. D. All of the above

All of the above

Which of the following is true of premature​ infants? A. They are more susceptible to infection. B. They are at risk for respiratory difficulty. C. They can easily develop hypothermia. D. All of the above

All of the above

Which of the following may be a hazard of an improperly fitting cervical​ collar? A. Allows flexion of the neck B. Prevents the patient from opening his mouth C. Allows hyperextension of the neck D. All of the above

All of the above

Which of the following may occur when there is bleeding from a large​ vein? A. Air embolism B. Hypoperfusion C. Transmission of bloodborne illnesses D. All of the above

All of the above

Which of the following may result from a woman in her third trimester of pregnancy lying in a supine​ position? A. Compression of the inferior vena cava B. Supine hypotensive syndrome C. Fetal compromise D. All of the above

All of the above

Which of the following medical conditions may be the underlying cause of an apparent behavioral​ emergency? A. Mental illness B. Stroke C. Hypoglycemia D. All of the above

All of the above

Which of the following questions may help the EMT assess a patient with abdominal pain? A. Do you have any allergies to foods or medicines? B. What medications are you taking? C. Do you have any medical problems, such as diabetes or heart problems? D. All of the above

All of the above

Which of the following should be considered during the scene​ size-up of a hazardous materials​ emergency? A. Potential for spread of the substance B. The properties and potential dangers of the substance C. Identification of the substance D. All of the above

All of the above

Which of the following should be done when the​ infant's head has been​ delivered? A. Check to see if the umbilical cord is around the neck. B. Suction the mouth. C. Suction the nose. D. All of the above

All of the above

Which of the following should increase the​ EMT's suspicion of internal​ bleeding? A. Penetrating trauma to the chest or abdomen B. Fall from a height three or more times the​ patient's height C. ​High-speed motor vehicle collision D. All of the above

All of the above

Which of the following should the EMT do to control maternal bleeding after delivery of the​ baby? A. Massage the uterus until it is firm and​ grapefruit-sized. B. Allow the infant to​ nurse, if possible. C. Apply direct pressure with a sanitary napkin over any perineal lacerations. D. All of the above

All of the above

While on the scene of an emergency call involving a patient who is dependent upon a medical device for​ survival, the EMT can ask which of the following individuals about the specifics of the​ device? A. ​Patient's home health aide or nurse B. ​Patient's family C. Patient D. All of the above

All of the above

While ventilating a conscious​ patient's tracheostomy tube with a connected​ bag-valve mask, the EMT should verify the​ patient's comfort level for which of the​ following? A. Ventilation rate B. Ventilation pressure C. Ventilation volume D. All of the above

All of the above

You are called for a women who is about to deliver. During the labor​ process, your​ patient's water breaks. She experiences a rush of warm water and an increase in uterine contractions. The purpose of the amniotic fluid is​ to: A. provide lubrication during the delivery of the baby. B. allow the fetus to float during development. C. help maintain a constant fetal body temperature. D. All of the above

All of the above

Your patient has come in contact with a strong acid substance. Using an alkaline solution to treat the patient may result in which of the​ following? A. Production of heat when the acid and alkali interact B. Further damage to the​ patient's tissues C. Neutralization of the acid substance D. All of the above

All of the above

Your patient is a 70yearold male whose tractor rolled over onto him. Your assessment makes you suspicious that the patient has a fractured pelvis. Which of the following complications should you anticipate? A. Damage to internal organs B. Damage to the nerves of the lower extremities C. Shock D. All of the above

All of the above

Your patient is a​ 50-year-old female with a history of alcoholism. She is found unresponsive in bed by her son. Which of the following possibilities should you keep in mind during your​ assessment? A. Heart attack B. Head injury C. Low blood sugar D. All of the above

All of the above

Which of the following is NOT appropriate in caring for a patient with closed soft-tissue injuries and a significant mechanism of injury? A. Treat for shock if you think there are internal injuries even if the patient's vital signs are normal. B. Anticipate vomiting. C. Splint any swollen, deformed extremities. D. All of the above are appropriate.

All of the above are appropriate.

Concerning attempted​ suicide, which of the following statements is true​? A. There is always an indication that a suicide attempt is imminent. B. Truly suicidal patients nearly always leave a note. C. All suicide attempts must be taken​ seriously, even if the method seems insincere. D. Attempted suicide is a matter for law​ enforcement, not EMS.

All suicide attempts must be taken​ seriously, even if the method seems insincere.

You are on a​ stand-by at a local high school football game. One of the football players comes over to the ambulance and complains that his face feels numb and that he has a burning sensation in his fingers. You notice that the​ patient's cheeks and fingers are bright red. How should you treat the​ patient? A. Tell the patient that his condition is superficial and allow him to return to the game. B. Warm the affected areas with gentle rubbing. C. Place the patient inside the ambulance and begin active rewarming measures. D. Allow the patient to warm up inside the ambulance by passive rewarming measures.

Allow the patient to warm up inside the ambulance by passive rewarming measures.

You are just leaving the scene of a​ multiple-casualty incident with two Priority 1 patients. Which of the following is appropriate regarding​ communication? A. Have dispatch relay your patient reports to the receiving facility. B. Notify the receiving hospital by cellular phone of the nature of the​ patients' injuries. C. Notify the receiving facility by radio of the nature of the​ patients' injuries. D. Allow the transportation sector officer to notify the receiving facility

Allow the transportation sector officer to notify the receiving facility

Which of the following is NOT likely to be a sign of respiratory distress in a pediatric​ patient? A. Altered mental status B. Use of abdominal muscles C. Grunting D. Nasal flaring

Altered mental status

Which of the following is the MOST sensitive indicator of​ hypoperfusion? A. Dilation of the pupils B. Increased heart rate C. Altered mental status D. Delayed capillary refill

Altered mental status

Which of the following terms describes the temperature of the surrounding​ air? A. Core temperature B. Wind chill index C. Shell temperature D. Ambient temperature

Ambient temperature

Regarding the use of emergency​ sirens, which of the following is false​? A. Dense shrubbery may block the sound of the siren. B. EMTs should assume that other drivers cannot hear the siren. C. Ambulance operators are not affected by siren noise. D. The sound of the siren may increase the​ patient's anxiety.

Ambulance operators are not affected by siren noise.

When seeking expert advice on how to proceed at a hazardous materials​ incident, which of the following information is important to​ provide? A. Your level of EMS certification B. Amount of the substance involved C. Your background in chemistry D. Both A and B

Amount of the substance involved

Although it can be difficult to definitively determine hip dislocation in the field, certain signs and symptoms are usually there. Which of the following statements is false ? A. Often there is lack of sensation in the limb. B. The posterior hip dislocation is the most common. C. A posterior hip dislocation presents with a bent knee and the foot may hang loose. D. An anterior hip dislocation would probably present with the entire lower limb rotated inward and the hip would usually be flexed.

An anterior hip dislocation would probably present with the entire lower limb rotated inward and the hip would usually be flexed.

What constitutes a​ multiple-casualty incident​ (MCI)? A. An incident with 3 to 15 patients that the service handles without calling for additional resources B. An event that places a great demand on EMS equipment and personnel resources C. An event that unfolds over a long period of time and requires​ federal, state, and local response D. Any event that requires additional ambulances to adequately transport the patients from the scene

An event that places a great demand on EMS equipment and personnel resources

According to​ coworkers, your​ 25-year-old female patient suddenly began acting aggressively and being verbally abusive. She tells you she is​ "starving" and you notice that she is pale and diaphoretic. Which of the following would be an appropriate general​ impression? A. An underlying physical illness B. Alcoholic intoxication C. Alcohol withdrawal D. Sudden onset of schizophrenia

An underlying physical illness

You are beginning triage at the scene of a​ mass-casualty incident in which a commuter train has derailed. Which of the following should you do​ first? A. Announce that everyone who can get up and walk needs to go to the parking lot of a nearby building. B. Check the airway status of all patients who do not appear to be moving. C. Identify all Priority 1 patients. D. Do a quick pulse check on all patients at the scene.

Announce that everyone who can get up and walk needs to go to the parking lot of a nearby building.

Multiple trauma and multisystem trauma patients are situations in which the EMT is expected to perform which of the​ following? A. Respond above and beyond the call of duty. B. Provide the best quality care​ possible, including definitive care​ on-scene. C. Practice outside the scope of practice in order to provide the greatest good. D. Anticipate and treat problems with a greater level of complexity than usual.

Anticipate and treat problems with a greater level of complexity than usual.

Which of the following is a substance that will neutralize a poison or its​ effects? A. Antidote B. Deactivator C. Curative D. Antevert

Antidote

What type of trauma triage criteria regarding transport would a finger amputation​ receive? A. Any hospital with surgical facilities B. Any hospital C. ​Patient's choice of destination D. Trauma center

Any hospital with surgical facilities

A fracture of which of the following bones would be considered a skull​ fracture? A. Temporal bone B. Maxilla C. Nasal bones D. Any of the above

Any of the above

Which of the following traumatic chest injuries may result in massive, often fatal internal hemorrhage? A. Aortic dissection B. Hemopneumothorax C. Cardiac tamponade D. Hemothorax

Aortic dissection

Your protocols call for you to use the Pediatric Assessment Triangle when assessing young children. You know that the second leg of the triangle assesses the​ "work of​ breathing" and the bottom leg of the triangle assesses​ "circulation to the​ skin." What does the first leg of the triangle stand​ for? A. Alteration of mental status B. Assessment C. Airway D. Appearance

Appearance

Your patient is a 25-year-old man who picked up an iron skillet with a very hot handle. He has a reddened area with blisters across the palm of his hand. Which of the following must be avoided in the prehospital management of this wound? A. Application of a dry, sterile dressing B. Application of antibiotic ointment C. Elevation of the wound above the level of the heart D. Keeping the site clean

Application of antibiotic ointment

Which of the following could worsen the effects of a snake​ bite? A. Immobilizing the bitten extremity B. Having the patient lie still C. Application of ice D. Constricting bands above and below the bite

Application of ice

Your patient is a​ 38-year-old male driver of a vehicle that left the roadway and struck a bus stop shelter and a tree. He is conscious and​ alert, he has some abrasions on his​ forehead, his skin is warm and​ dry, and he has a strong radial pulse and no difficulty breathing. Which of the following descriptions represents the MOST appropriate way for removing the patient from the​ vehicle? A. Apply a cervical collar and perform rapid extrication onto a long backboard. B. Apply a cervical collar and short spine immobilization device before removing to a long backboard. C. Place the backboard on the stretcher and have the patient​ stand, turn, and lie down on the backboard while you maintain manual​ in-line stabilization of the cervical spine. D. Have the patient stand up and then do a​ "standing take-down" onto a long backboard.

Apply a cervical collar and short spine immobilization device before removing to a long backboard.

Your patient is a 35-year-old female who spilled a cup of hot coffee on herself. She has an area about twice the size of the palm of her hand on her right thigh that is red and painful, but without blisters. When caring for this injury in the prehospital setting, which of the following is appropriate? A. Apply an antibiotic ointment. B. Apply a lotion containing a topical anesthetic and aloe vera. C. Apply a plastic bag full of ice to the skin. D. Apply a dry sterile dressing.

Apply a dry sterile dressing.

50. A 37-year-old male was hit by a trolley and his foot was almost severed. It is only connected by some skin and crushed bone. What should you do? a. Immediately apply a tourniquet to control bleeding. b. Complete the amputation. It will make the injury much easier to handle. c. Because of the seriousness of the injury, wait for ALS before providing any care. d. Apply a pressure dressing to control bleeding, stabilize the foot by splinting, apply oxygen, and transport as a priority patient.

Apply a pressure dressing to control bleeding, stabilize the foot by splinting, apply oxygen, and transport as a priority patient.

In caring for a 27-year-old male who has a large laceration on his anterior forearm, you have noticed that your pressure dressing has become saturated with blood. Which of the following should you do next? A. Remove the saturated dressings and apply a large trauma dressing. B. Apply an ice pack over the original dressing. C. Apply a tourniquet. D. Apply additional dressing material over the top of the original dressing and bandage it in place.

Apply additional dressing material over the top of the original dressing and bandage it in place.

When assisting with a delivery in the​ field, which of the following should be done as the head begins to emerge from the​ vagina? A. Check for crowning. B. Pull on the baby. C. Advise the mother not to push or strain. D. Apply gentle pressure to the head with your gloved hand.

Apply gentle pressure to the head with your gloved hand.

Bleeding from open abdominal injuries should initially be controlled with which one of the following techniques? A. Applying direct pressure to the wound B. Applying an ice pack or chemical cold pack C. Applying an occlusive dressing D. Packing the wound with rolled gauze

Applying direct pressure to the wound

Which of the following is acceptable when actively rewarming a hypothermic​ patient? A. Encouraging the patient to exercise B. Applying humidified oxygen C. Warming the patient as quickly as possible D. Giving coffee or brandy to drink

Applying humidified oxygen

Your patient is a 60yearold male who is complaining of severe epigastric abdominal pain and difficulty breathing. He is pale, sweaty, and pleads with you, "Don't let me die, I think I'm going to die." Which of the following measures is NOT part of your initial treatment of this patient? A. Giving 15 lpm of oxygen by nonrebreather mask B. Applying the defibrillator pads to his chest C. Determining whether you should assist the patient in taking nitroglycerin D. Placing the patient in a position of comfort

Applying the defibrillator pads to his chest

You respond to a construction site on a very hot summer day for a person​ "not acting​ right." You find a​ 38-year-old male patient seated in a work vehicle with the air conditioner on maximum. Coworkers state the patient was working for the last 2 hours pouring concrete when he started​ "speaking gibberish" and nearly collapsed. The patient is responsive to verbal stimuli. Your initial vital signs are blood pressure​ 90/60, pulse​ 136, and respiratory rate 24. The​ patient's skin is cool to the touch and dry but his shirt is wet with visible salt rings. Which is the best treatment for the​ patient? A. Have the patient sip water slowly to replace what he has lost. B. Cool the patient with tepid water. C. Have the patient drink at least 1 liter of water to replace what he has lost. D. Apply​ high-concentration oxygen and rapidly transport to the nearest medical facility.

Apply​ high-concentration oxygen and rapidly transport to the nearest medical facility.

You respond to a​ 32-year-old male who fell 20 feet off a ladder. He is responsive to painful stimuli. He has snoring respirations at 20 a minute with decent chest rise and fall. He has a broken​ femur, a broken​ wrist, and a lacerated radial artery that is bleeding profusely. Which of these injuries is the highest​ priority? A. Arterial bleed B. Snoring respirations C. Femur fracture D. Broken wrist

Arterial bleed

Which of the following occurs when gases leave a damaged lung in a diving accident and enter the​ bloodstream? A. Arterial gas embolism B. Hyperbaric arterial injury C. Arterial thrombosis D. Arterial blood gases

Arterial gas embolism

Which of the following vessels contain blood under the highest amount of​ pressure? A. Arteries B. Veins C. Venules D. Capillaries

Arteries

Which of the following vessels has the thickest muscular walls that allow constriction and​ dilation? A. Capillaries B. Arteries C. Lymphatic vessels D. Veins

Arteries

At which point should you complete your patient care report​ (PCR)? A. As soon as you are free from patient duties B. En route to the hospital C. At the end of your shift D. As soon as you arrive at the hospital

As soon as you are free from patient duties

You are on the scene of a pediatric patient who is on a specialized home monitoring system. This situation was not specifically addressed in your EMT class and your EMT partner has not encountered this​ situation, either. What is an advisable course of​ action? A. Request an advanced life support ambulance crew to respond for assistance. B. Ask the parent or caregiver what has been done in the past to correct the situation. C. Contact medical control and ask to speak to a specialist for advice. D. Load the patient and rapidly transport the patient to the closest hospital for evaluation.

Ask the parent or caregiver what has been done in the past to correct the situation.

Your patient is a​ 21-year-old male who slid​ head-first down a water slide at his fraternity house and impacted the bales of straw that his fraternity brothers had erected as a barrier to keep participants from sliding onto the adjacent highway. The patient is conscious and complaining of neck pain. Which of the following should be included in your​ assessment? A. Ask the patient to cautiously touch his chin to his chest to check for range of motion. B. If the patient has no numbness or​ tingling, ask him to stand and try to walk. C. Ask the patient to grasp and squeeze your hands. D. Apply painful stimuli to his​ extremities, starting distally and moving closer and closer to the body.

Ask the patient to grasp and squeeze your hands.

Your patient is a 14-year-old male who crashed his bicycle, landing prone and sliding along a gravel trail. He has deep abrasions to his hands, arms, chest, and knees. The patient has small pieces of gravel, twigs, and dirt embedded in the abrasions. Which of the following is the best way to manage this situation after taking cervical spine immobilization? A. Use your fingers to pick embedded debris from the wound, bandage with moist saline dressings in place, and transport. B. Do not attempt to remove any debris, apply pressure dressings over the embedded material if necessary, apply high-concentration oxygen, and transport. C. Use a tongue depressor to scrape large pieces of debris out of the wounds, place the patient on high-concentration oxygen, and transport. D. Assess for additional injuries, flush away large pieces of debris with a sterile dressing, place dressings on the abrasions, bandage them in place, and transport.

Assess for additional injuries, flush away large pieces of debris with a sterile dressing, place dressings on the abrasions, bandage them in place, and transport.

In cases of extreme​ hypothermia, you will find the patient​ unconscious, with no discernible vital​ signs, and skin cold to your touch with stiff joints as if they appear dead. What is the emergency care for these​ patients? A. Contact medical control for input into the best treatment for this patient. B. Check distal​ CSM, apply warming packs to the​ extremities, and transport the patient. C. Call the coroner as indications are the patient is cold and​ deceased, which is a definitive sign of death. D. Assess the carotid pulse for 30 to 45​ seconds; if there is no​ pulse, start CPR immediately and prepare to apply the AED.

Assess the carotid pulse for 30 to 45​ seconds; if there is no​ pulse, start CPR immediately and prepare to apply the AED.

You respond to a patient who was hit in the face with a chair. Upon​ arrival, you notice a patient leaning in the corner and bleeding profusely from the mouth and nose. Your first action should be which of the​ following? A. Take cervical spine precautions. B. Assure scene safety. C. Place the patient on a nonrebreather mask​ (NRB) at 15 lpm. D. Suction the airway and have the patient lean back.

Assure scene safety.

Which of the following is NOT a common cause of chest pain in the​ elderly? A. Pneumonia B. Asthma C. Aneurysm D. Angina

Asthma

Once you have established with reasonable certainty that your patient is suffering from a behavioral​ emergency, which of the following is the best course of​ action? A. Begin intense individual counseling with the patient. B. There is no need to assess for medical conditions if the patient does not have a history of past medical problems. C. Establish the best way to restrain the patient before he has the opportunity to become violent. D. Attempt to determine if there is a possible physical cause for the​ patient's behavior.

Attempt to determine if there is a possible physical cause for the​ patient's behavior.

Which of the following trauma triage criteria would justify transportation to a trauma center based on mechanism of injury​ alone? A. Auto versus pedestrian B. Vehicle rollover C. Intrusion into the occupant area greater than 8 inches D. Vehicle​ crash-generated telemetry data

Auto versus pedestrian

Which of the following is NOT essential equipment on an​ ambulance? A. Portable suction unit B. Automatic transport ventilator C. Pediatric oxygen administration devices D. Portable oxygen tanks

Automatic transport ventilator

Which of the following choices is NOT a justifiable reason for delaying transport of a critical trauma​ patient? A. Immobilizing the patient to a long spine board B. Suctioning the airway C. Ventilating a patient in respiratory distress D. Awaiting arrival of the​ patient's parents​ on-scene

Awaiting arrival of the​ patient's parents​ on-scene

To respond to a hazardous materials​ incident, the EMT should be trained at which of the four​ levels? A. Technician B. Operations C. Awareness D. Specialist

Awareness

Which of the following is true concerning​ disinfection? A. A solution of 1 part household bleach to 100 parts water is an effective disinfectant for health care purposes. B. A​ low-level disinfectant will kill germs on ambulance floors and walls. C. High level disinfection destroys all sources of infection. D. All of the above

A​ low-level disinfectant will kill germs on ambulance floors and walls.

Someone who has more hair than average on his body is at a higher risk than someone with less hair to be exposed to a toxin by which of the following​ routes? A. Ingestion B. Absorption C. Injection D. Inhalation

B. Absorption

Which of the following chemical agents prohibits the use of oxygen in the​ cells? A. Blister agents B. Cyanide C. Nerve agents D. All of the above

B. Cyanide

Which of the following is a common reason that EMTs are struck by motor vehicles at the scene of motor vehicle collisions on​ roadways? A. EMTs sometimes lose track of what they are doing and wander into traffic. B. Drivers are distracted by the scene of the collision and do not pay attention to what they are doing. C. Drivers who are angry at being delayed deliberately take out their frustrations on rescue workers. D. All of the above

B. Drivers are distracted by the scene of the collision and do not pay attention to what they are doing.

If you are first on scene of a​ multiple-casualty incident, what should you do first in determining where and how should you park your​ apparatus? A. Provide downstream blocking if you are driving one of the larger units. B. Establish Command and confirm the exact location of the incident with the dispatch center before parking. C. Immediately park in the same direction and in single file. D. Use apparatus to institute​ "downstream blocking" of the scene to protect the work area.

B. Establish Command and confirm the exact location of the incident with the dispatch center before parking.

Which of the following is a common hazard associated with the inner circle of a typical auto​ extrication? A. Penetrating injury from extrication tools B. Flying glass and metal fragments C. Carbon monoxide poisoning D. Both A and C

B. Flying glass and metal fragments

Which of the following phases of extrication follows the other three​ phases? A. Gaining access to the patient B. Immobilizing and extricating the patient C. Recognizing and managing hazards D. Stabilizing the vehicle

B. Immobilizing and extricating the patient

Which safety device prevents car doors from opening during a​ collision? A. Nader jam B. Nader pin C. Safety lock D. Safety pin

B. Nader pin

When working at a nighttime emergency on a​ roadway, which of the following safety considerations should the prudent EMS provider do to increase​ safety? A. Keep only the fog lights on to keep the light out of​ drivers' vision. B. Operate amber warning lights​ only, if the option is available. C. Keep all available response lights on to warn oncoming traffic. D. Switch the headlights to the​ "high beam" position for maximum lighting

B. Operate amber warning lights​ only, if the option is available.

Which of the following is the​ EMT's key role at the scene of an auto​ extrication? A. Incident commander B. Patient advocate C. Safety officer D. Rescue technician

B. Patient advocate

The current trend in international terrorism is that it tends to be motivated by which of the​ following? A. Environmental issues B. Religion C. Ethics D. Politics

B. Religion

You have arrived at the scene of a vehicle collision in which there are utility wires down around the vehicle. The three occupants appear dazed and other responding units have not yet arrived. Which of the following is the BEST course of​ action? A. Push the wires out of the way with a long​ pry-bar in order to access the patients. B. Stay at a distance and caution the occupants to stay inside the vehicle. C. Cool the wires with large amounts of water to prevent a fire from starting. D. Encourage the occupants to exit the vehicle by jumping free of the vehicle before touching the ground.

B. Stay at a distance and caution the occupants to stay inside the vehicle.

Which of the following BEST describes why it is important for the EMT to understand the extrication​ process? A. To provide direction to the rescue technicians if they appear to be making errors B. To anticipate any dangerous steps in the extrication process C. To take over the extrication process if requested by the rescue technicians D. All of the above

B. To anticipate any dangerous steps in the extrication process

You are on the scene of a vehicle collision involving a downed power line. You are walking in an area you feel is a safe distance from the line and begin to feel a tingling sensation in your feet. Which of the following would be a proper method of ensuring your safety while exiting the​ area? A. Turn 90 degrees to the direction you are walking and hop to a safe place on one foot. B. Turn 180 degrees to the direction you are walking and shuffle away from the danger area. C. Turn 180 degrees to the direction you are walking and quickly run to a safe location. D. Turn 90 degrees to the direction you are walking and shuffle away from the danger area.

B. Turn 180 degrees to the direction you are walking and shuffle away from the danger area.

Which of the following is the MOST common type of rescue across the United​ States? A. Hazardous materials response B. Vehicle rescue C. Water rescue D. Farm rescue

B. Vehicle rescue

Which of the following is a type of chemical​ weapon? A. Bomb containing radioactive material B. Vesicants C. Plastic explosives D. Bacterial toxin

B. Vesicants

Which of the following does NOT have an impact on the behavior of a chemical in a​ mass-casualty incident? A. Route of entry into the body B. Whether it was an intentional or accidental exposure C. Physical state of the agent D. Volatility of the agen

B. Whether it was an intentional or accidental exposure

Which of the following is the first concern when you are caring for a patient who has been in a vehicle​ collision? A. Preserving​ evidence, such as alcohol beverage bottles and vehicle positioning B. Your own safety C. Treating​ life-threatening injuries D. Helping law enforcement get the​ driver's name and insurance information

B. Your own safety

Dangerous organisms that can grow in a variety of environments and are capable of producing toxic substances are known​ as: A. biologicals. B. bacteria. C. toxins. D. viruses.

B. bacteria.

Two elements that differentiate a hazardous materials incident from a terrorist incident​ are: A. weapons of mass destruction and a political agenda. B. crime scene considerations and targeting responders. C. targeting responders and weapons of mass destruction​ (WMD). D. targeting responders and a political agenda.

B. crime scene considerations and targeting responders.

Primary injuries from a blast​ include: A. impaled objects. B. lung injuries. C. liver lacerations. D. extremity fractures.

B. lung injuries.

The best way for a terrorist to weaponize and disseminate a chemical nerve agent exposing a large number of people would be​ to: A. throw a paper bag loaded with nerve agent onto a busy subway train. B. steal a truck that sprays for mosquitos and aerosolized the nerve agent into a crowded subdivision. C. throw a Molotov cocktail loaded with nerve agent into a windshield of a passenger bus. D. put it in a paper bag in a crowded mall and puncture the bag with a stick.

B. steal a truck that sprays for mosquitos and aerosolized the nerve agent into a crowded subdivision.

The largest single hazard associated with EMS highway operations​ is: A. power lines. B. traffic flow. C. sharp objects. D. explosion.

B. traffic flow.

While performing night​ operations, which of the following combinations of lights provide maximum​ visibility? A. ​Red/amber warning lights​ ON, headlights​ OFF, fog lights​ OFF, and traffic directional boards OFF B. ​Red/amber warning lights​ ON, headlights​ OFF, fog lights​ OFF, and traffic directional boards ON C. ​Red/amber warning lights​ ON, headlights​ ON, fog lights​ OFF, and traffic directional boards ON D. ​Red/amber warning lights​ ON, headlights​ ON, fog lights​ ON, and traffic directional boards ON

B. ​Red/amber warning lights​ ON, headlights​ OFF, fog lights​ OFF, and traffic directional boards ON

Which of the following should be checked with the vehicle engine turned​ off? A. Windshield wiper operation B. ​Dash-mounted gauges C. Battery D. Warning lights

Battery

When dealing with a psychiatric​ emergency, which one of the following would generally be inappropriate behavior on your​ part? A. Act in a calm​ manner, giving the patient time to gain control of his emotions. B. Let the patient know that you are listening to what he is​ saying, and explain things to the patient honestly. C. Be as hurried as you can. It is extremely important to resolve the call and get the patient to the hospital as soon as possible. D. Stay alert for sudden changes in behavior.

Be as hurried as you can. It is extremely important to resolve the call and get the patient to the hospital as soon as possible.

Your EMS system requires EMTs to report suspected child abuse. Which of the following statements about reporting suspected abuse is​ TRUE? A. If you think the parent is the type of person who would be an​ abuser, report that assumption. B. Your subjective impression of the patient is important to report. C. Be careful to qualify statements of your suspicions with words such as​ "suspected" and​ "possible." D. Identify the person you suspect of being an abuser as the​ "child abuser."

Be careful to qualify statements of your suspicions with words such as​ "suspected" and​ "possible."

Why is it important to remove constricting items such as rings before thawing a frozen​ extremity? A. To prevent damage to the property such as rings and watches B. Because thawing leaves clots behind in the veins C. Because thawed areas often swell D. All of the above

Because thawed areas often swell

Full immobilization of a trauma​ patient, including placing a cervical collar and securing the patient to a​ backboard, should take place at which of the following points in​ time? A. En route to the hospital B. After checking for cervical range of motion C. Before moving the patient to the ambulance D. Before the primary assessment

Before moving the patient to the ambulance

Your patient is a​ 3-year-old child who is in severe respiratory distress. She is cyanotic and responds only to painful stimuli. According to the​ patient's mother, the child had complained of a sore throat earlier and has had a fever since early in the morning. You determine that the child may have epiglottitis. Which of the following is the BEST course of​ action? A. Begin gentle ventilations with a​ bag-valve mask​ (BVM) and supplemental oxygen. B. Insert an oropharyngeal airway​ (OPA) and begin​ bag-valve-mask ventilations with supplemental oxygen. C. Perform abdominal thrusts and finger​ sweeps; attempt to ventilate. D. Take the​ patient's temperature with an oral thermometer.

Begin gentle ventilations with a​ bag-valve mask​ (BVM) and supplemental oxygen.

When a person acts in a manner that is unacceptable to himself or those around​ him, this would be considered which of the​ following? A. Psychotic episode B. Mental illness C. Behavioral emergency D. Psychosomatic reaction

Behavioral emergency

Which of the following is the highest priority to the EMT in delivery of an infant with​ meconium-stained amniotic​ fluid? A. Checking for fever B. Vigorously rubbing the​ infant's back immediately upon delivery to stimulate breathing C. Being prepared to suction the infant immediately before he takes a breath D. None of the above

Being prepared to suction the infant immediately before he takes a breath

Your patient has attempted suicide by slitting his wrists. You notice that he has run the knife across his​ wrist, perpendicular to the​ arm, and that the wound is not deep. Which of the following statements is true regarding the likelihood for serious blood​ loss? A. You should expect severe blood loss. B. Blood loss is probably not​ life-threatening. C. There is most likely tremendous internal blood loss. D. A tourniquet will probably be necessary.

Blood loss is probably not​ life-threatening.

You are on the scene of a​ 50-year-old male who lacerated his arm on a sheet of plate glass. He is​ pale, diaphoretic, and mumbling incoherently. You have controlled an arterial bleed with direct pressure. His blood pressure is​ 70/40, pulse​ 120, and respiratory rate of 28. Which of the following signs is the most​ concerning? A. Blood pressure B. Altered mental status C. Tachycardia D. Respiratory rate

Blood pressure

If you do not have a commercial tourniquet​ available, what common device found on the ambulance can be used as a​ substitute? A. Ice packs tied in place with triangle bandage B. Use a rope​ tie-down to make a tourniquet C. Air or vacuum splint D. Blood pressure cuff

Blood pressure cuff

Which of the following is part of the​ body's compensatory response to blood​ loss? A. Blood vessels dilate and the heart rate increases. B. Blood vessels constrict and the heart rate decreases. C. Blood vessels constrict and the heart rate increases. D. Blood vessels dilate and the heart rate decreases.

Blood vessels constrict and the heart rate increases.

Which of the following is the leading cause of internal​ bleeding? A. Penetrating trauma B. Blunt trauma C. Aortic aneurysm D. Crush injury

Blunt trauma

Which of the following describes a breech​ presentation? A. The infant presents buttocks first. B. The infant presents with both feet first. C. The infant presents face first. D. Both A and B

Both A and B

Which of the following describes an open extremity injury? A. A gunshot wound has penetrated the skin and then fractured the bone. B. Bone ends have lacerated the soft tissues and skin from the inside. C. The joint capsule has been torn open during a dislocation. D. Both A and B

Both A and B

Which of the following is appropriate when assessing an emotionally disturbed​ patient? A. Establish eye and verbal contact. B. Avoid arguing with the patient. C. Make it clear that you are in control of the​ situation, not the patient. D. Both A and B

Both A and B

When preparing to move a patient that is using a specialty medical device from his house to the​ ambulance, which of the following questions would be appropriate to ask the​ family? A. What worked best for moving the patient the last time he was transported by​ ambulance? B. How do you normally move the​ patient? C. Can you carry the patient to the ambulance since you know the best way to move the​ patient? D. Both A and B are correct.

Both A and B are correct.

Which of the following is NOT part of the circulatory​ system? A. Brain B. Blood C. Blood vessels D. Heart

Brain

Which of the following spiders can cause a characteristic wound with a bite that is often​ painless? A. Black widow spider B. Brown recluse spider C. Sheet web spider D. Argiope spider

Brown recluse spider

Your patient is a 40-year-old male who has been exposed to a dry chemical powder and is complaining of severe pain on both of his hands, the site of the contact. He is working in an illegal chemical manufacturing plant and there is no decontamination shower on site. Which of the following would be the BEST way to manage this situation? a. Have the fire department connect to a hydrant and spray down the patient from head to toe. b. Brush away as much powder as possible and then pour a bottle of sterile saline solution over his hands. c. Brush away as much of the powder as possible and then have the patient hold his hands under running water from a faucet or regular garden hose. d. Brush away the powder and bandage the hands in a position of function.

Brush away as much of the powder as possible and then have the patient hold his hands under running water from a faucet or regular garden hose.

Your patient is a​ 48-year-old male who has been exposed to a toxic powder that can be absorbed through the skin. Which of the following measures should be taken by the​ EMT? A. Brush off the powder and flush the​ patient's skin with a solution of baking soda and water. B. Wipe the powder away with a damp cloth. C. Flush the​ patient's skin with copious amounts of water. D. Brush off the powder and flush the​ patient's skin with copious amounts of water.

Brush off the powder and flush the​ patient's skin with copious amounts of water.

You assess a 35-year-old female patient with a chemical burn to her right forearm and hand. As you assess the burn, you notice a white powder on the burn. What should be your next step? a. Transport the patient immediately to the closest burn center. b. Brush the powder off the patient's arm and hand, and then flush with copious amounts of water. c. Flush the arm and hand with copious amounts of water. d. Brush off the powder, bandage the arm, and transport the patient to the closest trauma center.

Brush the powder off the patient's arm and hand, and then flush with copious amounts of water.

You have arrived at the scene of a motor vehicle collision to find that there are several critically injured patients. Which of the following should you do​ first? A. Begin extrication of the most critically injured patient. B. Begin to care for the patient closest to your vehicle. C. Call for additional rescue units and transporting ambulances. D. Begin triage of all patients before deciding on additional resources.

C. Call for additional rescue units and transporting ambulances.

Which of the following is NOT traditionally a responsibility of an EMT on the scene of a hazardous materials incident caused by​ terrorism? A. Scene​ size-up B. Assessment of the toxicological risk C. Containment and control D. Activation of the incident management system​ (IMS)

C. Containment and control

Which two types of terrorism does the Federal Bureau of Investigation define as occurring in the United​ States? A. Chemical and biological B. Violent and intimidation C. Domestic and international D. Psychological and economic

C. Domestic and international

Which of the following is adequate protection for the​ EMT's head while at the scene of an auto​ extrication? A. Any hat with a rear brim B. Watch cap C. Firefighting helmet D. Nomex hood

C. Firefighting helmet

Which of the following animals is the direct source of exposure of the bacterium that causes the plague to​ humans? A. Squirrels B. Rats C. Fleas D. Bees

C. Fleas

Which of the following should be used to protect a patient during​ extrication? A. Netting B. Plastic blanket C. Goggles D. None of the above

C. Goggles

Which of the following is the rule of thumb for determining the stability of a vehicle that has been involved in a​ collision? A. It is stable if the ignition is turned off. B. It is stable if it is upright on all four wheels. C. It should only be considered stable after assessment by trained rescue personnel. D. It is stable if the transmission is in​ "Park" or the parking brake is on.

C. It should only be considered stable after assessment by trained rescue personnel.

Once access to the patient is​ gained, which of the following is the next goal during an auto​ extrication? A. Stabilization of the vehicle B. Extrication of the patient from the vehicle C. Primary assessment and rapid trauma exam D. Disentanglement of the patient

C. Primary assessment and rapid trauma exam

Which of the following activities should NOT occur during the scene​ size-up? A. Determining the need for additional resources B. Determining the mechanism of injury C. Providing airway management D. Determining the extent of the​ patient's entrapmen

C. Providing airway management

An unresponsive patient is found sitting in the front seat of a vehicle with his legs pinned under the dash. Once his legs have been​ freed, which of the following methods should be used for removal of the​ patient? A. Quick extraction B. Scoop stretcher C. Rapid extrication technique D. Rapid​ take-down

C. Rapid extrication technique

Concerning the threat of nuclear devices used in terrorist​ attacks, which of the following is MOST likely to be used in such an​ attack? A. Radiological dispersal device B. Military nuclear device C. Sabotage of facilities for nuclear research D. Improvised nuclear weapon

C. Sabotage of facilities for nuclear research

Which of the following describes vehicle glass that is designed to break into small pieces rather than shattering into sharp​ fragments? A. Alloy glass B. Plexiglas C. Tempered glass D. Leaded glass

C. Tempered glas

You are at the scene of a vehicle collision on an interstate highway. There are two cars involved and both are out of the lanes against the median wall. As an EMS​ provider, what is your primary safety​ concern? A. Searching for sharp edges of broken glass and torn metal B. Evaluating the risk of fire caused by ruptured fuel lines C. Watching for distracted drivers who are driving past the scene D. Ensuring the patient is properly packaged for transport

C. Watching for distracted drivers who are driving past the scene

When responding to an emergency located on an interstate highway in a​ medium-duty ambulance, the vehicle should be​ positioned: A. across as many lanes as possible to block oncoming traffic. B. in a safe location in the emergency lane of the interstate. C. in the same direction as other response vehicles in a single line. D. to one side of the damaged vehicles for easy egress.

C. in the same direction as other response vehicles in a single line.

Secondary injuries from a blast most likely​ include: A. liver lacerations. B. extremity fractures. C. lung injuries. D. impaled objects.

C. lung injuries.

A type of harm that you may be exposed to as an EMT responding to a terrorist incident that is characterized by a resulting​ post-traumatic stress disorder​ (PSTD) is typically called​ a(n): A. etiological harm. B. radiological harm. C. psychological harm. D. thermal harm.

C. psychological harm.

You are on an ambulance responding to a​ mass-casualty incident at a shopping mall on a warm Saturday afternoon. En​ route, you listen to the incident commander broadcast information to dispatch and all inbound units about 20 patients complaining of respiratory difficulty and a burning sensation in their eyes. Your suspicions of a potential terrorist incident are the result​ of: A. the location of the call. B. the timing of the event. C. the type of event. D. ​on-scene warning sign

C. the type of event.

Dangerous organisms that grow inside of living cells and reproduce are known​ as: A. bacteria. B. toxins. C. viruses. D. biologicals.

C. viruses.

Three methods for minimizing exposure to harmful incidents​ are: A. ​technique, distance, and serum. B. ​time, decontamination, and shielding. C. ​time, distance, and shielding. D. ​time, decontamination, and serum.

C. ​time, distance, and shielding.

Which of the following abbreviations is NOT​ correct? A. CAPD for continuous acute peripheral disease B. SCA for sickle cell anemia C. WBC for white blood cells D. RBC for red blood cells

CAPD for continuous acute peripheral disease

At​ 0330, you responded to the scene of an overturned tanker truck. What organization could you contact to obtain information about the liquid the truck is​ carrying? A. Regional poison control center B. ​CHEM-CALL C. Environmental Protection Agency​ (EPA) D. CHEMTREC

CHEMTREC

​________ is the response​ agency, established in​ Washington, DC, as a service of the Chemical Manufacturers Association that can provide either you or your dispatcher with information about the hazardous material and contact the shipper to provide you with information about the material. A. EPA B. OSHA C. HAZMART D. CHEMTREC

CHEMTREC

Which of the following is the BEST course of action for the EMT to take when caring for a patient with an AICD who goes into cardiac​ arrest? A. The AED should be applied and​ used, but CPR should not be started while​ on-scene or transporting the patient. B. CPR and an AED should be used as indicated while​ on-scene and transporting the patient. .C. No action beyond transport is required since the patient already has an implanted defibrillator. D. CPR should be​ started, but the AED should not be used while​ on-scene or transporting the patient.

CPR and an AED should be used as indicated while​ on-scene and transporting the patient.

You are enjoying some time at the beach on your day off when you hear a swimmer crying for help. As you spot the swimmer about 30 feet from​ shore, she cries out again but appears to be getting weaker. Although there is no lifeguard on​ duty, there is a rowboat and a ring buoy available. Assuming you do NOT know how to swim or consider yourself a poor​ swimmer, which of the following should you do​ first? A. Row the boat out to the swimmer. B. Use the buoy to float out to the swimmer. C. Call for help and try to throw the buoy to the swimmer. D. Find someone who can swim to try to swim out and save the swimmer.

Call for help and try to throw the buoy to the swimmer.

You have just arrived on the scene of an agricultural business and see three men coming out of a​ building, choking and holding their heads. One of the men tells you there are two workers still inside. What should you do​ next? A. Cover your nose and mouth with a wet towel and check on the status of the two workers inside the building. B. Call for properly trained assistance and stay a safe distance away from the scene. C. Evaluate the three men and administer​ high-concentration oxygen, then help them get a safe distance away from the scene. D. Get the assistance of the men who made it out of the building to prevent delay in finding the workers inside.

Call for properly trained assistance and stay a safe distance away from the scene.

In which of the following vessels does the vital exchange take place and also has the thinnest walls through which​ oxygen, nutrients, and wastes can​ pass? A. Capillaries B. Veins C. Microvenules D. Arterioles

Capillaries

Which of the following blood vessels cannot stop bleeding by​ constricting? A. Capillaries B. Vena cava C. Arterioles D. Arteries

Capillaries

Your patient is a​ 6-year-old child who has fallen down while running on a sidewalk. She has abrasions on both knees and the palms of both​ hands, which are oozing blood. This is an example of bleeding from which of the following types of​ vessels? A. Lymphatic vessels B. Arteries C. Capillaries D. Veins

Capillaries

External bleeding may be classified according to types. Which one of the following is one of those​ types? A. Capillary bleeding B. Liver bleeding C. Brain bleeding D. Cardiac bleeding

Capillary bleeding

Which of the following signs of shock is useful in pediatric patients but not in​ adults? A. Restlessness B. Tachycardia C. Tachypnea D. Capillary refill

Capillary refill

Which of the following is the cause of the black residue found in a​ patient's mouth and nose following smoke​ inhalation? A. Carbon B. Cyanide C. Carbon monoxide D. Tar

Carbon

Which of the following is one of the MOST common inhaled poisons associated with fire​ suppression? A. Carbon dioxide B. Methane gas C. Carbon monoxide D. Polyvinyl chloride

Carbon monoxide

44. You are dispatched to an industrial plant for a burn. You arrive and find a 60-year-old male who came in contact with a broken steam line and appears to have partial thickness burns on both hands and arms. He is lying on the ground and coworkers are gently spraying him with water from a nearby hose. What should you do? a. Because of his age, immediately put him in your ambulance and transport him to a burn center. b. Call for air transport and continue to pour water on the burns until transport arrives. c. Care for the burn and do a complete patient assessment, including cervical spine precautions. d. Before doing treatment, call the local burn center for instruction.

Care for the burn and do a complete patient assessment, including cervical spine precautions.

51. Which of the following is not considered soft tissue? a. The skin b. Muscles c. Membranes d. Cartilage

Cartilage

Which of the following allows for smooth movement of bone surfaces against one another at joints? A. Ligaments B. Smooth muscle C. Cartilage D. Peritoneum

Cartilage

Which of the following may be a source of ignition when an ambulance is parked off the roadway in a field of dried​ grass? A. Catalytic converter B. Leaking coolant C. Mobile radio D. Battery

Catalytic converter

Which of the following is the toxic effect of​ alcohol? A. Damage to the liver B. Irreversible damage to the kidneys C. Central nervous system depression D. Central nervous system​ (CNS) stimulation

Central nervous system depression

Which portions of the spine are the most vulnerable to​ injury? A. Cervical and lumbar B. Cervical and sacra C. Thoracic and lumbar D. Thoracic and sacra

Cervical and lumbar

Your patient is a​ 10-year-old male whose jacket hood caught on a branch as he jumped out of a tree. He was momentarily suspended about 12 inches off the ground but was immediately lowered to the ground by his brothers. Which of the following injuries should you​ suspect? A. Cervical spine injury B. Lumbar spine injury C. ​Soft-tissue injury of the neck only D. Thoracic spine injury

Cervical spine injury

You arrive on scene to find an unresponsive homeless man lying in the alleyway clutching a wine bottle. He is wearing a​ long-sleeved shirt, but does not have pants on. His skin feels cold to the touch. The​ week's temperatures have not been above 50 degrees F. Which of the following should be done​ first? A. Apply the AED B. Ensure an open airway C. Begin passive rewarming D. Check a carotid pulse

Check a carotid pulse

A painter falls from his ladder and tells you he has dislocated his shoulder again. When you attempt to splint the shoulder, it "pops back into place." What should your next step be? A. Contact medical control for input into the best treatment for this patient. B. Check distal CSM, apply a sling and swathe, and transport the patient. C. Check distal CSM, apply a traction splint, and transport the patient. D. Continue splinting and report the popping sound to the triage nurse when you arrive at the hospital.

Check distal CSM, apply a sling and swathe, and transport the patient.

Your patient is an 11yearold male who has a swollen, painful, and angulated right lower arm after falling from his bicycle onto his hands. Which of the following should be considered in the immobilization of his injured extremity? A. Check pulse, movement, and sensation distal to the injury before and after splinting. B. Use an upper extremity traction splint. C. Do not attempt to realign the extremity before splinting. D. Immobilize from the shoulder to the wrist.

Check pulse, movement, and sensation distal to the injury before and after splinting.

Which of the following is LEAST often seen in carbon monoxide​ poisoning? A. Confusion B. Nausea C. Cherry red lips D. Headache

Cherry red lips

Which of the following would NOT indicate severe airway obstruction of a pediatric​ patient? A. Child cries loudly B. ​Child's cough becomes ineffective C. Cyanosis D. Respiratory difficulty accompanied by stridor

Child cries loudly

Which of the following statements about shock in pediatric patients is​ TRUE? A. Children rarely develop shock because of their large blood volume. B. Children are unable to compensate for shock. C. Children decompensate quickly but also recover quickly. D. Children compensate for shock for a long​ time, then crash quickly.

Children compensate for shock for a long​ time, then crash quickly.

Which of the following statements regarding the skin and body surface area of children as compared to adults is​ TRUE? A. Children have a larger body​ surface-to-mass ratio. B. Children have more subcutaneous fat. C. A child is not more prone than an adult to heat loss through the skin. D. The skin of a child is thicker.

Children have a larger body​ surface-to-mass ratio.

You are called to a residential neighborhood at 12:30 A.M. Your patient has just finished eating a supersized meal of deep fried fish. He is now complaining of a "crampy" pain in the right upper quadrant and has had two episodes of nausea and vomiting with a green emesis. What condition do you suspect that your patient is experiencing? A. Peptic ulcer B. Intestinal obstruction C. Cholecystitis D. Appendicitis

Cholecystitis

Which of the following describes the normal appearance of amniotic​ fluid? A. Thin​ fluid, greenish-yellow in color B. Clear and colorless fluid C. A fluid containing blood and mucus D. Thick​ fluid, greenish-black in color

Clear and colorless fluid

You respond to assist law enforcement with a​ 35-year-old male patient​ who, according to​ them, is​ "high as a​ kite." The patient is​ anxious, tachycardic, and angry. What condition do you​ suspect? A. Marijuana overdose B. Cocaine overdose C. GHB overdose D. Alcohol overdose

Cocaine overdose

In which of the following zones is the EMT expected to be staged at the scene of a hazardous materials​ incident? A. Ground zero B. Warm zone C. Hot zone D. Cold zone

Cold zone

Which of the following is NOT an acceptable way of attempting to identify a hazardous​ material? A. Using binoculars to obtain information from the placard on a container B. Checking the material safety data sheet C. Asking the driver of the​ tractor-trailer truck involved in the incident D. Collecting a sample of the material for laboratory analysis

Collecting a sample of the material for laboratory analysis

Which of the following is a principle of effective Incident​ Command? A. The command location must not be revealed to anyone on the scene. B. Command is most effective when one person is responsible for not more than 6 other people. C. ​Police, fire, and EMS must establish separate command locations. D. None of the above

Command is most effective when one person is responsible for not more than 6 other people.

You are on an EMS standby for a boxing tournament. During one of the matches, one of the female boxers delivers a forcible uppercut to the chest of her opponent, who falls to the ground. The match is declared over on the basis of a TKO. However, the opponent fails to arise following a 1 to 2 minute interval. EMS is summoned to the ring. You find the patient pulseless and breathing agonal gasps. You suspect which of the following traumatic conditions? A. Cardiac tamponade B. Aortic dissection C. Commotio cordis D. Tension pneumothorax

Commotio cordis

Which of the following is typically helpful in assessing a pediatric​ patient? A. Tell the patient to answer with​ "yes" or​ "no" only. B. Communicate with the patient at eye level. C. Ask the parent to leave the room. D. Let the patient know that you are in charge.

Communicate with the patient at eye level.

Which of the following is appropriate in the examination of a painful, swollen extremity of a conscious patient? A. Seeing if you can elicit crepitus on palpation B. Gently attempting to flex the midportion of the bone to check for angulation C. Asking the patient to see if he can bear weight on the extremity D. Comparing the injured side to the uninjured side

Comparing the injured side to the uninjured side

Which of the following is a classification of the severity of​ shock? A. Compensated B. Consumption C. ​Non-compensated D. Uncompensated

Compensated

Which of the following injuries is considered an indirect brain​ injury? A. Depressed skull fracture with cerebral penetration by bone fragments B. Gunshot wound to the head C. Cerebral laceration D. Concussion

Concussion

Which of the following should always be done next for a behavioral patient once you have established scene​ safety? A. Conduct a primary assessment B. Restrain the patient C. Obtain baseline vital signs D. Assess the​ patient's blood glucose

Conduct a primary assessment

Which of the following will worsen the condition of the patient in​ shock? A. Elevating the​ patient's legs 8 to 10 inches B. Transporting without first splinting all extremity fractures C. Applying​ high-concentration oxygen when it is not needed D. Conducting oneself in a manner that increases the​ patient's fear and anxiety

Conducting oneself in a manner that increases the​ patient's fear and anxiety

A patient who is lying on cold ground is losing the most amount of heat by what​ mechanism? A. Shivering B. Radiation C. Conduction D. Convection

Conduction

A patient gives you a story of having gone out to the movies last​ night, but who according to family​ members, has not left the house in years. This condition is​ called? A. Apoplexy B. Confabulation C. Lying D. Depression

Confabulation

Your pediatric patient is showing signs and symptoms of hypothermia. Which of the following should be done FIRST after getting the patient out of the cold​ environment? A. Complete the patient care report. B. Confirm the​ patient's axillary temperature. C. Conserve the​ patient's body heat. D. Contact medical direction.

Conserve the​ patient's body heat.

Which of the following is a function of the autonomic nervous​ system? A. Solving complex math problems B. Speaking C. Running or walking D. Constriction of blood vessels

Constriction of blood vessels

Your patient is a​ 22-year-old male who has ingested a large amount of alcohol and is vomiting. He is conscious but uncooperative. He allows you to examine him but refuses transport to the hospital. You have sought assistance from law enforcement on scene. Which of the following is the BEST​ action? A. Obtain a witnessed refusal. B. Contact medical control for further advice. C. Stay with the patient until he has stopped vomiting. D. Find a relative or neighbor to come over and stay with the patient.

Contact medical control for further advice.

Your patient is a​ 17-year-old male who is spitting and coughing after swallowing some gasoline while siphoning from a gas tank. Which of the following should you do​ first? A. Insert a Combitube or another blind insertion​ device, if you are trained to do so. B. Contact medical control. C. Have the patient drink a glass of milk. D. Administer syrup of ipecac.

Contact medical control.

Blunt trauma resulting in a closed chest injury creates the potential for which of the following internal injuries? A. Abrasion B. Contusion C. Evisceration D. Avulsion

Contusion

Which of the following is NOT an open tissue injury? A. Contusion B. Abrasion C. Evisceration D. Avulsion

Contusion

Which of the following is NOT an open tissue injury? a Abrasion b Contusion c Avulsion d Evisceration

Contusion

Which of the following is the process in which heat is lost from the body as wind passes over​ it? A. Hydrodynamic cooling B. Condensation C. Convection D. Exposure

Convection

Which of the following complications may arise from properly splinting an injured extremity? A. Compromising circulation to the extremity B. Converting a closed fracture to an open one C. Ignoring lifethreatening problems while focusing on an extremity injury D. All of the above

Converting a closed fracture to an open one

Of the following venomous​ snakes, which one usually has the highest incidence of a​ "dry bite?" A. Copperhead B. Water moccasin C. Mississauga rattlesnake D. Coral snake

Coral snake

Which of the following types of snake is NOT a pit​ viper? A. Water moccasin B. Rattlesnake C. Copperhead D. Coral snake

Coral snake

Which of the following is the correct technique to check for crowning in the assessment of a woman in​ labor? A. Place your hand on the​ woman's abdomen, just above the​ umbilicus, and check for the firmness of the uterus during contractions. B. Ask the woman to​ "push" or​ "bear down" as you inspect the vaginal opening. C. Cover her with a​ sheet, have her remove her​ underwear, wait for a​ contraction, and then visualize the vaginal opening. D. None of the above

Cover her with a​ sheet, have her remove her​ underwear, wait for a​ contraction, and then visualize the vaginal opening.

Which of the following does NOT indicate that a patient may be about to become​ violent? A. Tense body posture or clenched fists B. Loud voice and rapid pacing C. Profane language D. Crying

Crying

Which of the following is NOT an example of a specialty rescue​ team? A. Water rescue B. High angle rescue C. Confined space D. Advanced cardiac life support

D. Advanced cardiac life support

On arrival at a vehicle​ collision, you observe a small fire in the engine compartment. A bystander is attempting to smother the fire with a jacket as you approach with an​ A-B-C extinguisher from your ambulance. Which of the following is the BEST method of extinguishing the​ fire? A. Aim the nozzle to the side of the fire​ and, with a sweeping​ motion, push the fire at its base. B. Sweep the nozzle of the extinguisher left and right using short bursts until you are out of extinguishing agent. C. Close the hood to contain the fire within the engine compartment. D. Aim at the base of the fire and use short bursts until the fire is out.

D. Aim at the base of the fire and use short bursts until the fire is out.

A terrorist incident may have features in common with which of the following types of​ incidents? A. Mass casualty B. Infectious disease C. Hazardous materials D. All of the above

D. All of the above

Anthrax can infect an individual through which of the following routes of​ exposure? A. Inhalation B. Skin C. Ingestion D. All of the above

D. All of the above

The extent of radiation depends on which of the following​ factors? A. Shielding from the source B. Duration of exposure C. Distance from the source D. All of the above

D. All of the above

Under which of the following conditions are safety vests with reflective trim effective in increasing the visibility of the​ EMT? A. On sunny days B. At night C. On cloudy days D. All of the above

D. All of the above

Which of the following human factors can lead to an increased risk of injury for an EMT at the scene of a vehicle​ collision? A. Careless attitude toward personal safety B. Physical problems that impede strenuous effort C. Lack of proficiency in the use of rescue tools D. All of the above

D. All of the above

Which of the following is a consideration when responding to a terrorist​ incident? A. Presence of booby traps such as secondary explosive devices B. Crime scene considerations C. Presence of mass casualties D. All of the above

D. All of the above

Which of the following is an important reason for controlling spectators at the scene of an extrication​ incident? A. Spectators may get injured. B. Spectators may interfere with rescue efforts. C. It ensures respect for the​ patient's privacy. D. All of the above

D. All of the above

Which of the following may be a clue that an incident is the result of terrorist​ violence? A. Occurrence on the anniversary of a significant or symbolic historical event B. Number of people present C. Function of the target D. All of the above

D. All of the above

Which of the following should the EMT consider when protecting a patient during​ extrication? A. Debris B. Weather C. Noise D. All of the above

D. All of the above

Which of the following can serve as appropriate eye protection for an EMT on the scene of an​ extrication? A. Prescription eyeglasses B. Plastic face shield of the type used for Standard Precautions C. Shield of a​ Firefighter's helmet D. Face conforming goggles

D. Face conforming goggles

Which of the following is NOT generally recognized as a form of terrorist​ incident? A. Industrial sabotage B. Biological weapons C. Explosion D. Harassment

D. Harassment

Which of the following features increases the appeal of an infectious agent for use in terrorist​ attacks? A. Low virulence B. Low potency C. Instability D. High infectivity

D. High infectivity

Which of the following is NOT a consideration in the general protection against agents used in terrorist​ attacks? A. Shielding from the agent B. Distance from event C. Duration of exposure D. Immunization

D. Immunization

Which of the following provides the BEST protection for the​ EMT's hands during a vehicle rescue​ operation? A. Fabric work gloves with​ non-slip palms B. ​Elbow-length chemical-proof gloves C. ​Double-thickness latex gloves D. Leather gloves

D. Leather gloves

Upon arrival at the scene of a motor vehicle​ collision, you find a vehicle that is on fire. The fire is confined to the engine compartment and the hood is tightly closed. Of the​ following, which is the BEST course of​ action? A. Discharge a dry chemical fire extinguisher onto the​ patient's clothing to protect him from the fire. B. Use a​ long-handled pry-bar to open the hood and put out the fire with a fire extinguisher. C. Wet the interior of the vehicle to keep the fire from spreading. D. Let the fire burn and concentrate on extricating the patient.

D. Let the fire burn and concentrate on extricating the patient.

Which of the following considerations should be determined when conducting a scene​ size-up at night of a single vehicle collision involving a​ rollover? A. Make and model of the vehicle involved B. How many EMS supervisors are needed for command C. Which patient is the least injured D. Mechanism of injury

D. Mechanism of injury

Which of the following methods of decontamination is NOT usually a prehospital activity during a terrorism​ incident? A. Isolation B. Dilution C. Absorption D. Neutralization

D. Neutralization

Which of the following is a good choice for preventing further injury to a patient during auto​ extrication? A. Disposable blanket B. Placing your body over the top of the patient C. Sheet D. None of the above

D. None of the above

Which of the following BEST defines domestic terrorism​? A. Politically motivated act of violence that occurs within the United States B. Interpersonal violence between family members C. Acts of violence perpetrated by American citizens against the government of another country D. Small group of people directing violent acts at the population or government without foreign influence

D. Small group of people directing violent acts at the population or government without foreign influence

You are managing a patient who is being extricated from a vehicle by a rescue crew using hydraulic tools. Which of the following items should you use in order to protect your patient during the extrication​ process? A. Aluminized rescue blanket to protect the patient from sharp edges B. Short spine board to prevent debris from contacting the patient C. Oxygen mask to prevent inhalation of noxious fumes D. Wool blanket to protect the patient from broken glass

D. Wool blanket to protect the patient from broken glass

When extrication will involve cutting the roof off a​ vehicle, stabilization of the vehicle should​ include: A. placing chocks on both sides of at least two wheels. B. deflating all tires by slashing them. C. placing wheel chocks on both sides of all wheels. D. deflating all tires by pulling the valve stems.

D. deflating all tires by pulling the valve stems.

Standard management of a chemical burn caused by a terrorist incident​ includes: A. leaving any corrosive materials on the skin. B. rinsing the area with ice water. C. using a neutralizing agent. D. irrigating vigorously with cool water.

D. irrigating vigorously with cool water.

An oxygen level of​ ________ is required for normal breathing. A. ​16.5% B. ​10.5% C. ​25.5% D. ​19.5%

D. ​19.5%

You arrive on the scene of an unknown emergency. As you scan the surrounding​ area, you notice a large number of people wearing unique armbands and shouting​ anti-government comments though you are not aware of any public gathering scheduled for today. Your suspicions of a potential terrorist incident are the result​ of: A. the timing of the event. B. the type of event. C. the location of the call. D. ​on-scene warning signs.

D. ​on-scene warning signs.

Assessing blood pressure on a patient with an AV​ shunt, fistula, or graft could lead to which of the​ following? A. Damage to the AV​ shunt, fistula, or​ graft, requiring surgery .B. Inaccurate blood pressure readings C. ​Nothing, as there is no need to avoid assessing blood pressure in a​ patient's arm with an AV​ fistula, shunt, or graft. D. Uncontrollable hemorrhage in the extremity

Damage to the AV​ shunt, fistula, or​ graft, requiring surgery

Why might you consider contacting medical direction before applying a tourniquet in the case of uncontrollable bleeding from an AV​ fistula? A. The patient may be in danger of losing the limb. B. Medical direction should be contacted before any use of tourniquets by the EMT. C. Damage to the fistula may occur. D. Hemodialysis anticoagulants may make bleeding more severe.

Damage to the fistula may occur.

You are called for a young man who was diving head first off a dock into a lake. Bystanders say he struck his head on the bottom because the water was too shallow. They said he was not breathing when they pulled him from the water and they have been performing rescue breathing for him. He is​ awake, but he is unable to breathe on his own. What type of damage or injury does this​ indicate? A. Closed head injury B. Damage to his thoracic spine C. Damage to​ C-3, C-4, or C5 D. Open head injury

Damage to​ C-3, C-4, or C5

Which of the following is the most significant mechanism of injury for a driver in a vehicle​ accident? A. ​Rear-end collision B. Spidering of the windshield C. Death of a passenger in the same vehicle D. Encroachment greater than 12 inches of the​ driver's compartment

Death of a passenger in the same vehicle

Which of the following is caused by trapped nitrogen gas in the tissues due to a rapid ascent from a scuba​ dive? A. Nitrogen narcosis B. Pulmonary embolism C. ​"Squeeze" injuries of the ear and sinuses D. Decompression sickness

Decompression sickness

Which of the following is NOT a responsibility of an EMT at the scene of a hazardous materials​ emergency? A. Treating injured patients B. Rehabilitating the hazmat team C. Decontaminating injured victims D. Staying in the cold zone

Decontaminating injured victims

Your patient is a 35yearold female with abdominal pain. Which of the following findings CANNOT be attributed to the patient experiencing pain? A. Shallow respirations B. Decreased level of consciousness C. Increased heart rate D. Slight increase in blood pressure

Decreased level of consciousness

While listening to an otherwise healthy elderly​ patient's lungs, you note decreased breath sounds. Which of the following may explain this​ finding? A. Decreased blood flow to the lungs B. Decreased lung capacity C. Previous abdominal injuries D. Increased chest wall flexibility

Decreased lung capacity

Which of the following is the likely result of diminished thyroid function in the​ elderly? A. Increased toxicity of medications B. ​Thin, dry, wrinkled skin C. Decreased tolerance of heat and cold D. Difficulty chewing and swallowing

Decreased tolerance of heat and cold

Which of the following is a potential complication of hyperventilating a patient with a brain​ injury? A. Decreasing the​ patient's blood pressure B. Increasing blood flow to the brain C. Decreasing blood flow to the brain D. Increasing the amount of carbon dioxide in the blood

Decreasing blood flow to the brain

You respond to the county jail for a​ 48-year-old inmate arrested two days ago for public intoxication. Guards state the patient is a known alcoholic and​ "frequent flier." The guards state that for several hours the patient was​ "acting crazy" and seeing​ "bugs on the​ walls." The patient then began seizing and they called for an ambulance. You notice the patient is no longer​ seizing, diaphoretic, or confused. What condition do you​ suspect? A. LSD abuse B. Delirium tremens C. Alcohol poisoning D. Acute episode of paranoid schizophrenia

Delirium tremens

Which of the following is NOT a mechanism of decontamination from hazardous​ materials? A. Deodorizing B. Disposing C. Disinfecting D. Emulsifying

Deodorizing

Gamma-hydroxybutyrate (GHB) is an example of which of the following types of​ drugs? A. Volatile spirit B. Stimulant C. Cannabis D. Depressant

Depressant

Which of the following is defined as an area of the body surface that is innervated by a single spinal​ nerve? A. Malar B. Dermatome C. Transverse process D. Spinous process

Dermatome

What are some of the most important critical decisions an EMT can make on the scene of a serious​ trauma? A. Whether to await ALS care​ on-scene or begin BLS transport B. Load and go versus stay and play C. Determining patient​ priority, amount of time​ on-scene, and hospital transport decision D. Physiological​ determinants, anatomic​ criteria, and mechanism of injury

Determining patient​ priority, amount of time​ on-scene, and hospital transport decision

Which of the following is the main focus of the EMT's assessment and history taking of the patient with abdominal pain? A. Determining the possible need for immediate surgery B. Determining if the patient meets criteria to refuse treatment and transport C. Determining the presence of shock D. Determining the cause of the pain

Determining the presence of shock

You are called to assess a pregnant woman who is approximately 7 months pregnant. She states that her pregnancy has been uneventful but she is experiencing intermittent headaches. Her vital signs are pulse​ 118, respirations​ 22, blood pressure​ 138/88, and blood sugar 148. Which of the following is true regarding a pregnant​ woman? A. Diabetes may be made worse during pregnancy. B. Her pulse rate should be lower than normal. C. Her blood pressure is usually higher during pregnancy. D. Her respirations usually remain the same during pregnancy.

Diabetes may be made worse during pregnancy.

By which of the following means does the​ fetus's blood pick up nourishment from the​ mother? A. Diffusion B. Direct circulation C. Indirect circulation D. Osmosis

Diffusion

Which of the following is the underlying cause of neurogenic​ shock? A. Extreme emotional response to paralysis B. Failure of the heart to adequately pump blood C. Dilation of blood vessels D. Blood loss from damaged spinal blood vessels

Dilation of blood vessels

Bleeding from soft-tissue injuries should initially be controlled with which one of the following techniques? a Application of an ice pack or chemical cold pack b Direct pressure c Elevation of the injured part d Running cold water over the wound

Direct pressure

Your patient is a​ 33-year-old man who has a gunshot wound to his right leg and has​ active, steady, dark red bleeding. He is​ awake, pale, and diaphoretic. He has a strong radial pulse of 112 per​ minute, a respiratory rate of 24 breaths per​ minute, and a blood pressure of​ 122/82 mmHg. He has no other injuries or complaints. Which of the following is the BEST sequence of steps in the management of this​ patient? A. ​High-concentration oxygen, elevation of the​ extremity, and application of ice B. Direct​ pressure, high-concentration​ oxygen, and splinting the leg C. Cervical spine​ immobilization, high-concentration​ oxygen, direct​ pressure, and pressure point compression D. ​High-concentration oxygen,​ tourniquet, PASG, and elevation of the extremity

Direct​ pressure, high-concentration​ oxygen, and splinting the leg

Which of the following refers to a condition that interferes significantly with a​ person's ability to engage in activities of daily​ living? A. Terminal B. Disability C. Disabled D. Handicapped

Disability

You are assessing a patient in the front seat of a vehicle that was involved in a​ head-on collision. As you examine the interior of the​ vehicle, you notice the air bags have not deployed. What action should you take in order to render the scene safe to​ work? A. Disconnect the battery and wait 2 minutes before entering the vehicle. B. Turn the ignition to the​ "off" position and wait 2 minutes before entering the vehicle. C. Disconnect the battery and continue assessing the patient. D. Use a long spine board to shield yourself from the undeployed air bag.

Disconnect the battery and wait 2 minutes before entering the vehicle.

What is the process that removes the biological​ (etiological) contamination hazards as it destroys microorganisms and their​ toxins? A. Dilution B. Absorption C. Disinfection D. Emulsification

Disinfection

You are dispatched to a motor vehicle crash on a rural mountain highway. You have a patient who was unconscious on​ arrival, had a​ seizure, and is currently awake but combative. You suspect he may have a head injury. What is considered the BEST approach regarding transport of this​ patient? A. Begin transport to the trauma center on the​ ground, which is 1.5​ hours' driving time. B. Dispatch the medical helicopter to meet your unit at the community hospital. C. Transport to a local community hospital approximately 15 minutes away via ground. D. Dispatch and await the medical​ helicopter, which is 20 minutes away.

Dispatch and await the medical​ helicopter, which is 20 minutes away.

Which of the following actions may need to be taken by the EMT when treating and transporting a patient who was exposed​ to, and contaminated​ by, a hazardous​ material? A. Wearing heavy cloth gloves B. Wearing a​ gown, mask, and goggles to prevent secondary contamination C. Placing towels on the floor of the ambulance to soak up contaminated runoff water D. Disposing of patient care equipment such as​ blood-pressure cuffs and backboards

Disposing of patient care equipment such as​ blood-pressure cuffs and backboards

You are transporting a stable patient who was involved in a minor fall from a ladder at a height of about 10 feet to a local community hospital. You assumed full spinal precautions not only because the patient has midline back pain in the​ sacrum, but also because he was knocked unconscious. While​ transporting, the patient begins to become increasingly​ confused, develop an irregular respiratory​ rate, and experience a drop in heart rate with an increase in blood pressure. You just called in a radio report and are about 7 minutes from the hospital. A trauma center is about 10 minutes away. Which of the following is the BEST transport​ decision? A. Divert to the trauma center because the patient is becoming symptomatic. B. Call the trauma center for advice. C. Continue transporting to the local hospital since​ it's the closest facility. D. Continue transporting to the local hospital because​ you've already given report and they accepted the patient.

Divert to the trauma center because the patient is becoming symptomatic.

Barbiturates may be referred to as which of the​ following? A. Downers B. Acid C. Meth D. Uppers

Downers

Which of the following BEST describes when positive pressure is applied to the​ patient's airway using a CPAP​ machine? A. Between inhalation and exhalation B. During inhalation only C. During both inhalation and exhalation D. During exhalation only

During both inhalation and exhalation

You are treating a 16yearold skateboarder who has fallen at the skate park. She has an angulated left forearm that she has in a guarded position. When do you splint this injury? A. During the secondary exam B. Immediately C. During the primary exam D. En route to the hospital

During the secondary exam

What criteria based on the CDC guidelines allows a discretionary approach to trauma​ triage? For​ example, a patient not meeting ordinary trauma triage criteria may be transported to a trauma center based on what​ consideration? A. EMS provider judgment B. Review of morbidity and mortality C. ​"Golden Hour" criteria D. CDC​ "No-protocol Protocol"

EMS provider judgment

You are an EMT on the scene of a​ mass-casualty incident in which there are 50 patients from a bus​ roll-over. Incident Command has been established and​ EMS, law​ enforcement, and rescue sectors are established. You have just discovered that one of the bus passengers was carrying a suspicious package that is now leaking a white powdery substance. Which of the following entities should you contact about​ this? A. Rescue sector officer B. Incident Commander C. EMS sector officer D. Dispatch

EMS sector officer

When​ log-rolling a patient with a suspected spinal​ injury, which of the following EMTs directs the​ move? A. EMT at the heaviest portion of the patient B. EMT with the highest level of training C. EMT with the most seniority D. EMT at the head of the patient

EMT at the head of the patient

Which of the following has been shown to actually interfere with the normal grieving and healing process after a​ multiple-casualty incident? A. Critical incident stress debriefing​ (CISD) sessions B. Mental health personnel available on the scene C. EMT engaging in psychoanalysis D. Mental health personnel available during the two months following a major incident

EMT engaging in psychoanalysis

Which of the following BEST describes a localized cold injury with a clear line of demarcation of its​ limits? A. Late frostbite B. Immersion foot C. Early frostnip D. Deep frostnip

Early frostnip

Which of the following statements about shock in infants and children is the MOST​ accurate? A. Hypovolemia from head trauma does not occur in children. B. Wait for signs of hypotensive shock to clearly develop before treating a child for shock. C. Early signs of shock can be subtle in children. D. Definitive care for children with shock takes places in the prehospital environment.

Early signs of shock can be subtle in children.

You are examining a 24yearold female patient with lower quadrant abdominal pain. What is the MOST LETHAL possibility? A. Pelvic inflammatory disease B. Mittelschmerz C. Ectopic pregnancy D. Cholecystitis

Ectopic pregnancy

Which of the following BEST describes a disease such as​ deafness? A. Congenital disease B. Acquired disease C. Neither a congenital nor acquired disease D. Either a congenital or acquired disease

Either a congenital or acquired disease

Which of the following groups is the most susceptible to the effects of a​ poison? A. Teenager B. Elderly C. ​Pre-teens D. Adults

Elderly

Which of the following groups is MOST likely to be successful in a suicide​ attempt? A. Young women B. ​Middle-aged men C. Teenagers D. Elderly men

Elderly men

Your patient is a​ 12-year-old boy who ran his arm through a glass window and has an​ 8-inch laceration on his anterior forearm. You have applied a pressure dressing and​ bandage, but these have become saturated due to continued bleeding. Which of the following should you do​ now? A. Apply additional dressing​ material, bandage it in​ place, and apply pressure to the brachial artery. B. Elevate that arm and prepare to apply a tourniquet or consider administering a hemostatic agent. C. Remove the pressure dressing and​ bandage, apply direct pressure with your gloved​ hand, and elevate the arm. D. Remove the pressure dressing and​ bandage, apply an ice pack to the​ wound, and bandage it in place with an elastic bandage.

Elevate that arm and prepare to apply a tourniquet or consider administering a hemostatic agent.

Which of the following is the LEAST effective method of controlling​ bleeding? A. Hemostatic agent B. Elevation C. Direct pressure D. Tourniquet

Elevation

The medical acronym EMD stands for which of the​ following? A. Emergency Medical Dispatch B. Emergency Medical Director C. Emergency Medical Doctor D. Emergency Medical Driver

Emergency Medical Dispatch

Which of the following is a resource for specific actions to be taken by personnel dealing with a hazardous materials​ incident? A. Emergency Response Guidebook B. Food and Drug Administration​ (FDA) toll-free hotline C. Medical dictionary D. Medical protocols

Emergency Response Guidebook

When should the EMT calculate a GCS with a patient who suffered a fall of 20 feet from his apartment​ building? A. At the hospital before writing the care report B. Before departing from the scene C. As he approaches the patient D. En route to the hospital

En route to the hospital

You have responded to a call about a​ "psychiatric patient" to find a​ 24-year-old woman talking to herself in her home. She is rocking back and forth and you can see from a distance that she is perspiring profusely. You also see an empty medication bottle on the floor. She does not acknowledge your presence. When you attempt to speak to​ her, she continues to talk to herself as if you were not there. Which of the following should be included in your actions at the​ scene? A. Ensure that law enforcement is responding and wait for their arrival. B. Look through the house with your partner to try to find some clues to the​ patient's problem. C. Distract her so your partner can look around the house. D. Shake the​ patient's shoulder to check for responsiveness to physical stimuli.

Ensure that law enforcement is responding and wait for their arrival.

You respond to a farm for a possible snake bite. You find a​ 36-year-old male patient seated against a tree. Bystanders state the patient was bitten on the arm by a rattlesnake and is​ "really sick." As you​ approach, you notice that the patient appears in obvious​ distress, diaphoretic, and holding his right wrist. Which of the following is your highest​ priority? A. Ensure the snake is no longer a threat. B. Confirm the type of snake and contact medical control for specific instructions. C. Immediately apply a constricting band to minimize the spread of the venom. D. Perform a primary assessment and identify any potential life threats.

Ensure the snake is no longer a threat.

Which of the following is the highest priority during​ extrication? A. Controlling major bleeding B. Gaining access to the patient C. Maintaining the​ patient's airway D. Ensuring your personal safety

Ensuring your personal safety

By​ definition, a systemic poison causes harm to which of the​ following? A. Entire body B. Specific organ system C. Localized areas of tissue D. Stomach and intestines

Entire body

Which of the following is the outermost layer of the skin? A. Dermis B. Epidermis C. Adipose tissue D. Fascia

Epidermis

You are called to a local hardware store for a male patient complaining of difficulty breathing. As you pull into the parking lot you notice several people in front of the building coughing and in obvious distress. What would be an appropriate course of action at this​ point? A. Request a hazardous materials incident response and begin treating all patients displaying respiratory difficulties. B. Move the people with respiratory difficulties to a distant corner of the parking lot and set up a treatment sector. C. Locate the patient for whom you were requested and begin treatment while another ambulance is en route for the other patients. D. Evaluate the scene further for indications of a hazardous environment and request a hazardous materials response.

Evaluate the scene further for indications of a hazardous environment and request a hazardous materials response.

You are dispatched to a 42-year-old male who was shot in the abdomen and thrown from a vehicle. The patient is critical and a high-category trauma; however, due to the mechanism of injury, it is necessary to backboard the patient prior to transport. What is an important assessment before securing the patient? A. Examining the patient for entrance and exit wounds B. Performing a distal neurological assessment C. Searching for presence of diaphoresis, tachycardia, and hypotension D. Verifying trauma center ER bed availability

Examining the patient for entrance and exit wounds

Most state statutes allow an emergency vehicle operator to do which of the following on emergency​ calls? A. Pass a stopped school bus with its red flashers on B. Be exempt from liability in the event of a collision C. Drive around lowered​ cross-arms at a railroad crossing D. Exceed posted speed limits if life and property are not endangered

Exceed posted speed limits if life and property are not endangered

A patient who has abused​ "uppers" will display which of the following signs and​ symptoms? A. Slurred​ speech, constricted​ pupils, and frequent urination B. Constricted​ pupils, hypotension, and blurred vision C. Excessive​ talkativeness, dilated​ pupils, and dry mouth D. Dilated​ pupils, excessive​ salivation, and food cravings

Excessive​ talkativeness, dilated​ pupils, and dry mouth

What is a cycle of filling and draining of the abdominal cavity during peritoneal dialysis​ called? A. Set B. Trade C. Exchange D. Rotation

Exchange

Your patient is a 60yearold woman who stepped off a curb and injured her ankle. Your exam shows that her left ankle is swollen and painful. Which of the following should you do? A. Explain to the patient that her ankle is sprained and transport her with her ankle elevated on a pillow and a cold pack applied to the injury. B. Explain to the patient that you cannot tell if her ankle is sprained or fractured until she is Xrayed at the emergency department, then splint the ankle. C. Explain to the patient that her ankle is fractured and you must splint her ankle to prevent further injury and reduce pain. D. Transport the patient immediately to a trauma center, applying highconcentration oxygen en route.

Explain to the patient that you cannot tell if her ankle is sprained or fractured until she is Xrayed at the emergency department, then splint the ankle.

Which of the following is NOT a mechanism of musculoskeletal injury? A. Indirect force B. Twisting or rotational forces C. Direct force D. Extensive force

Extensive force

In a hypothermic patient the coldest blood is found in what part of the​ patient's body? A. Extremities B. Intestines C. Head D. Heart and lungs

Extremities

EMS operations generally include the Mobile command center and what other six​ areas? A. ​Extrication, Operations,​ Logistics, Triage,​ Finance, and Transportation B. ​Extrication, Staging,​ Triage, Treatment,​ Transportation, and Rehabilitation C. ​Extrication, Staging,​ Triage, Air​ operations, Transportation, and Rehabilitation D. ​Extrication, Staging,​ Logistics, Triage,​ Operations, and Transportation

Extrication, Staging,​ Triage, Treatment,​ Transportation, and Rehabilitation

Glasgow Coma Scale​ (GCS) is a neurological assessment that looks at which of the​ following? A. Grip​ strength, verbal​ response, and eye opening B. Verbal​ response, motor​ response, and AVPU C. Motor​ response, arm​ movement, and speech D. Eye​ opening, verbal​ response, and motor response

Eye​ opening, verbal​ response, and motor response

Heat stroke is caused by which of the following​ mechanisms? A. ​Heat-induced swelling of brain tissue B. Blockage of blood flow to the brain C. Failure of temperature regulation mechanisms D. Extreme dilation of all the blood vessels

Failure of temperature regulation mechanisms

Which of the following is a common cause of seizures in young children but NOT in​ adults? A. Meningitis B. Hypoglycemia C. Oxygen deficiency D. Fever

Fever

Which of the following is the MOST common cause of seizures in infants and​ children? A. Hypoglycemia B. Hypoxia C. Poisoning D. Fever

Fever

Your patient is a​ 30-year-old construction worker who fell from scaffolding and has been impaled by a piece of concrete reinforcement bar. The patient responds to verbal stimuli. You have determined the bar is too long to adequately secure during patient transport. Which of the following is your best course of​ action? A. Firmly stabilize the reinforcement bar in place so that the rescue crew can cut it short. B. Remove the reinforcement bar and pack the orbit with sterile moist dressings to keep the scene time under 10 minutes. C. Test the reinforcement bar for stability and remove it only if it is loose enough to be easily pulled from the wound. D. Transport with the reinforcement bar in place to prevent delay at the scene.

Firmly stabilize the reinforcement bar in place so that the rescue crew can cut it short.

Johnny is an EMT with a local ambulance service. As part of his​ job, he is trained to recognize the existence of a potentially dangerous scene involving hazardous materials and know when to request a hazmat response.​ Johnny's level of hazardous materials training is at the​ ________ level. A. First Responder Operations B. Hazardous Materials Specialist C. First Responder Awareness D. Hazardous Materials Technician

First Responder Awareness

Soraya is an EMT who volunteers for a local fire department. On the scene of a hazardous materials​ incident, she helps prevent the incident from becoming larger and ensures bystanders remain safe.​ Soraya's level of hazardous materials training is at the​ ________ level. A. First Responder Awareness B. Hazardous Materials Specialist C. Hazardous Materials Technician D. First Responder Operations

First Responder Operations

You are assessing a patient in labor. Her contractions are 2 minutes apart lasting 30 seconds with increasing pain. The patient states that she feels the urge to push. These signs indicate which stage of​ delivery? A. Third stage B. First and second stage C. Fourth stage D. Second stage

First and second stage

Based on the following presentations of patient​ injuries, which one would be considered MOST​ severe, justifying immediate transportation to a trauma​ center? A. Scalp laceration B. Flail chest C. Midline cervical spine pain D. Open​ (compound) midshaft femur fracture

Flail chest

Which of the following increases a scuba​ diver's risk of decompression​ sickness? A. Breathing​ 100% oxygen before a dive B. Flying within several hours after a dive C. Taking cold or sinus medications before a dive D. Diving on a full stomach

Flying within several hours after a dive

Which of the following is the opening at the base of the​ skull? A. Orbits B. Spinous process C. Temporomandibular joint D. Foramen magnum

Foramen magnum

The PASG may be used as a splinting device for patients with which of the following suspected injuries? A. Fractured pelvis B. Compression fracture of the lumbar or sacral spine C. Hip dislocation D. None of the above

Fractured pelvis

Which one of the following statements is NOT true concerning a knee injury? A. Fractures can occur to the proximal femur. B. There could be pain and tenderness. C. Fractures can occur to the proximal tibia and fibula. D. Fractures can occur to the patella.

Fractures can occur to the proximal femur.

You are caring for a 27-year-old male who has a puncture wound to the right upper chest. The patient was stabbed with a serrated steak knife by his ex-girlfriend. You have placed an occlusive dressing to the site and began emergent transport to the closest trauma center. However, while en route the patient begins to complain of increasing shortness of breath. You notice a decrease in ventilatory volume and an increase in thoracic diameter. Which of the following options would be the best step to perform next? A. Call dispatch for an ALS intercept en route to the hospital. B. Begin providing CPR to the patient. C. Begin providing BVM-assisted ventilations to the patient. D. Free a corner or edge of the dressing and have the patient exhale to release pressure buildup. And re-seal the wound.

Free a corner or edge of the dressing and have the patient exhale to release pressure buildup. And re-seal the wound.

Which of the following is the correct way to time the frequency of contractions in the pregnant​ woman? A. From the beginning of one contraction to the beginning of the next B. After counting the number of contractions in a​ 15-minute period, multiply by 4 C. From the end of one contraction to the beginning of the next D. From the beginning of a contraction to the end of the same contraction

From the beginning of one contraction to the beginning of the next

A burn extending into the subcutaneous fat would be classified as which type of burn? a Deep partial thickness b Full thickness c Superficial partial thickness d Superficial

Full thickness

You are called to the scene of a local night club for a​ 21-year-old female patient who is bradycardic and in respiratory arrest. The​ patient's airway is patent and the chest easily rises with​ bag-valve-mask ventilation. The​ patient's friends state the patient had only one or two drinks when she suddenly complained of dizziness and​ "seeing things." Her friends state that she then passed​ out, started twitching like she was​ seizing, and then stopped. Her friends adamantly state that she does not do drugs. What situation do you​ suspect? A. Cocaine overdose B. Anaphylactic reaction to the alcoholic drink C. Alcohol overdose D. GHB overdose

GHB overdose

You respond to a 48yearold female having a syncopal episode in the bathroom. You find the patient sitting on the commode vomiting into the trash can. The vomitus appears to look like coffee grounds and has a foul smell. The patient is pale and has been weak for the past few days. She has: A. peritonitis. B. abdominal aortic aneurysm. C. hernia. D. GI bleeding.

GI bleeding.

Your patient is a 37yearold man who tripped while walking down a hill and now has a painful, deformed right leg. Your assessment reveals that the foot is cold and mottled in appearance. You cannot detect a pulse in the foot or ankle. Which of the following is the BEST course of action? A. Explain to the patient that, because you cannot detect circulation in his foot, his leg will most likely have to be amputated above the site of the injury. B. Transport rapidly to the nearest trauma center. C. Splint the leg in the position in which it was found and transport without delay. D. Gently attempt to straighten the leg to regain a pulse before splinting.

Gently attempt to straighten the leg to regain a pulse before splinting.

Which of the following is NOT a principle of splinting that must be considered by the EMT? A. Gently replace protruding bone ends back beneath the skin to prevent further contamination. B. Splint an isolated extremity injury before moving the patient to the stretcher. C. Check the distal neurovascular function before and after splinting. D. Immobilize the site of an extremity injury from the joint above it to the joint below it.

Gently replace protruding bone ends back beneath the skin to prevent further contamination.

Your patient is a​ 44-year-old female with a history of alcoholism. She has been walking around at an outdoor fair on a​ hot, sunny day. She is disoriented to​ time; has​ hot, dry​ skin; and appears to be generally weak. Which of the following is the appropriate sequence of treatment for this​ patient? A. Get as much ice as possible from the food vendors at the​ fair, place the patient in a large container of​ ice, and apply oxygen by nonrebreather mask. B. Give oxygen by nonrebreather​ mask, have the patient sip a sports drink or electrolyte​ solution, remove heavy​ clothing, and place cold packs on her​ neck, armpits, and groin. C. Have the patient drink an electrolyte solution or sports drink and apply cold packs to her​ neck, armpits, and groin. D. Give oxygen by nonrebreather​ mask, remove heavy​ clothing, and place cold packs on her​ neck, armpits, and groin.

Give oxygen by nonrebreather​ mask, remove heavy​ clothing, and place cold packs on her​ neck, armpits, and groin.

Your patient is a​ 24-year-old man who smoked a cigarette dipped in formaldehyde and then went outside with no shoes on and walked in the snow for about an hour. He has deep local cold injuries to both feet. Which of the following should be included in your management of this​ patient? A. Massage the feet briskly. B. Have the patient sit with his feet lower than the rest of his body. C. Gradually rewarm both feet. D. Break blisters before wrapping both feet in sterile dressings.

Gradually rewarm both feet.

Which of the following should the EMT do during the treatment of localized cold​ injury? A. Gradually warm the affected area. B. Massage the affected area. C. Rub the affected area with snow. D. Encourage the patient to use the affected part.

Gradually warm the affected area.

Which of the following is NOT one of the three corners of the Pediatric Assessment Triangle​ (PAT)? A. Grimace B. Appearance C. Work of breathing D. Circulation to skin

Grimace

You are assessing a​ 2-year-old child whose mother states she has had a fever for several hours. Which of the following signs is cause for​ concern? A. Grunting at the end of expiration .B. Respiratory rate of 28 breaths per minute C. Crying D. Absence of nasal flaring with inhalation

Grunting at the end of expiration

For which of the following wounds should the EMT apply an absorbent dressing moistened with sterile saline and then cover it with an occlusive dressing? A. Gunshot wound (GSW) to the abdomen from which a loop of intestine is protruding B. Laceration to the neck C. The stump of an amputated extremity D. Stab wound to the chest

Gunshot wound (GSW) to the abdomen from which a loop of intestine is protruding

The drug ecstasy is classified as what type of​ drug? A. Hallucinogen B. Tranquilizer C. Depressant D. Barbiturate

Hallucinogen

Where could you find the phalange bones? A. Skull and neck B. Lower extremities C. Upper extremities D. Hands and feet

Hands and feet

The term poison is BEST described as any substance that can do which of the​ following? A. Harm the body B. Deactivate nerve transmission C. Depress the​ body's respirations D. Increase cellular activity

Harm the body

Your patient is a​ 15-year-old female who has been diagnosed with a personality disorder. She has no history of violent behavior. The staff at her residential care facility wants her to be transported for evaluation of a possible urinary tract infection. Which of the following is the BEST way to handle this​ situation? A. Refuse to transport the patient without a police escort. B. Transport the​ patient, but do not speak to her or perform an assessment. C. Ask that the patient be given a tranquilizer injection prior to transport. D. Have a female EMT attend to the patient.

Have a female EMT attend to the patient.

Your patient is a​ 3-year-old male with a stoma who has swallowed a household cleaner. Medical direction gives you an order for milk to dilute the stomach contents. Which of the following is the correct way of carrying out this​ order? A. Have the patient drink a glass of milk while you plug the stoma to prevent leakage. B. Have the patient drink one glass of milk. C. Administer one glass of milk through the stoma. D. Refuse the order and explain why.

Have the patient drink one glass of milk.

When treating a patient with acute abdominal pain, you should do which of the following? A. Have him drink milk to coat the stomach and reduce the pain B. Have the patient lie still and assume a position of comfort C. Administer sips of water if patient complains of thirst D. Have him take antacids in an attempt to decrease the pain

Have the patient lie still and assume a position of comfort

Which of the following is NOT recommended when controlling​ epistaxis? A. Pinching the nostrils together B. Having the patient tilt the head backward to elevate the nose C. Placing the unconscious patient in the recovery position D. Keeping the patient calm and quiet

Having the patient tilt the head backward to elevate the nose

Andrea is an EMS supervisor with a combined​ EMS/Fire department. On the scene of a hazardous materials​ incident, she uses her knowledge of hazardous materials to support the activities of other responders and manage the scene.​ Andrea's level of hazardous materials training is at the​ ________ level. A. First Responder Awareness B. First Responder Operations C. Hazardous Materials Technician D. Hazardous Materials Specialist

Hazardous Materials Specialist

Eduardo is an EMT with a major metropolitan fire department. He is assigned to a station within an industrial complex. On the scene of a hazardous materials​ incident, Eduardo functions on a team where he plugs a leaking drum.​ Eduardo's level of hazardous materials training is at the​ ________ level. A. First Responder Operations B. Hazardous Materials Technician C. Hazardous Materials Specialist D. First Responder Awareness

Hazardous Materials Technician

You are responding to a 52yearold male patient complaining of heartburn with epigastric pain. The patient's vital signs are stable and he does not have any pain upon palpation. He has a history of reflux disease and is on several medications for heartburn and acid reflux. What is your greatest concern with this patient? A. He will develop peritonitis. B. He is suffering from a myocardial infarction. C. He will aspirate on vomit. D. His appendix will rupture.

He is suffering from a myocardial infarction.

You walk up the stairs of a house to respond to a call of an elderly man. You notice that the stairs do not have any handrails. After responding to the reason why he called​ 911, what suggestion would you give to this patient to avoid falls in the​ future? A. ​Nothing, there is no problem. B. He should install a handrail on the front steps to avoid any falls. C. It is a good idea there are no handrails on the front steps as this negatively impacts his balance as he walks up the stairs. D. It is illegal to not have a handrail on the front steps and it must be fixed immediately.

He should install a handrail on the front steps to avoid any falls.

Most radiant heat is lost through which part of the​ body? A. Hands and feet B. Buttocks C. Head D. Torso

Head

When using a short spine immobilization​ device, which part of the body is secured​ last? A. Legs B. Torso C. Head D. Arms

Head

An elderly patient struck her head during a ground level fall. She presents with​ confusion, memory​ loss, elevated blood pressure and decreasing pulse. Which of the following conditions is most​ likely? A. Head injury B. Stroke C. Low blood sugar D. Psychiatric emergency

Head injury

Which of the following is the correct position to use when transporting a patient who has a tracheostomy​ tube? A. Shock position B. Head slightly elevated C. Prone D. Supine

Head slightly elevated

Your patient has been involved in a motor vehicle collision. He has a contusion on his​ forehead, is​ confused, and is bleeding from his nose. His heart rate is 90 beats per​ minute, blood pressure is​ 80/58 mmHg, respirations are 20 breaths per​ minute, and his skin is cool and clammy. Which of the following sets of injuries should you​ suspect? A. Head injury and spine injury B. Head injury and internal bleeding C. Head​ injury, spine​ injury, and internal bleeding D. Head injury

Head​ injury, spine​ injury, and internal bleeding

Which of the following is NOT a common cause of shock in infants and​ children? A. Dehydration B. Blood loss C. Heart failure D. Infection

Heart failure

Which of the following is the major cause of shock that the EMT will​ encounter? A. Excessive sweating B. High blood pressure C. Vomiting D. Hemorrhage

Hemorrhage

The greatest danger to the pregnant woman and her fetus involved in trauma is which of the​ following? A. Neurogenic shock B. Hemorrhagic shock C. Supine hypotensive shock D. Distributive shock

Hemorrhagic shock

Once you encounter uncontrolled bleeding from an AV​ fistula, which of the following methods would you consider using to control bleeding in addition to direct pressure and​ elevation? A. Internal wound management B. Tourniquet C. Pressure points D. Hemostatic dressings

Hemostatic dressings

The detection of a bulging mass through the belly button that is not pulsating should make the EMT suspicious that the patient may be suffering from which of the following? A. Hernia B. Ulcer C. Abdominal aortic aneurysm D. Gastroenteritis

Hernia

Your patient is a​ 23-year-old male who is unresponsive in the restroom of a bar. His respirations are slow and​ shallow, he has a heart rate of 50 beats per​ minute, he is sweating​ profusely, and he has constricted pupils. Which of the following substances is MOST likely responsible for the​ patient's condition? A. Ecstasy B. LSD C. Heroin D. PCP

Heroin

Which of the following is the MOST important means of managing a patient who has inhaled a​ poison, after the airway has been​ established? A. Hyperventilation B. High concentrations of oxygen C. Monitoring pulse oximetry D. Administering a specific antidote

High concentrations of oxygen

What is the underlying cause of bluish or reddish facial discoloration following a traumatic asphyxiation? A. High pressure on the chest leads to blood being forced from the right atrium into the face and neck. B. Bluish or reddish facial discoloration is not associated with traumatic asphyxiation; a pale discoloration is usually present. C. The patient has become hypoxic due to a chest injury and the finding suggests central cyanosis. D. The physiological strain of the body results in a flushed appearance and increased risk of a hypertensive event.

High pressure on the chest leads to blood being forced from the right atrium into the face and neck.

You are assessing a 30-year-old male patient that had his arm caught in a piece of machinery. By the time you arrive he has been freed. The patient tells you that he does not understand why you were called, but as you inspect the injured limb you notice a small puncture wound. You should have a high index of suspension of which of the following injuries? A. Puncture B. Crush injury C. Chemical burn D. High-pressure injection

High-pressure injection

Which of the following is NOT advisable following complete birth of the​ infant? A. Place the infant on his side with his head slightly lower than his body. B. Hold the infant by the ankles to allow the airway to drain. C. Keep the infant at the level of the​ mother's vagina until the cord is clamped and cut. D. Dry the infant and wrap him in a blanket.

Hold the infant by the ankles to allow the airway to drain.

In most​ cases, which of the following is the correct way to provide initial management of a suspected cervical spine​ injury? A. Hold the​ patient's head still in a​ neutral, "eyes​ forward" position. B. Gently apply pressure to the top of the​ patient's head. C. Provide approximately 15 pounds of upward cervical traction. D. Maintain the​ patient's head and neck in the position they are found.

Hold the​ patient's head still in a​ neutral, "eyes​ forward" position.

Which of the following questions is MOST helpful in distinguishing between normal​ age-related changes and the effects of an illness when assessing an elderly​ patient? A. When was the last time you had a bowel​ movement? B. Are you taking your medications the way you are supposed to be taking​ them? C. How is your​ appetite? D. How do you feel today compared to last​ week

How do you feel today compared to last​ week

Which of the following fluids is NOT checked by an​ EMT? A. Oil B. Hydraulic C. Transmission D. Coolant

Hydraulic

Which of the following may result from the application of a cervical collar that is too large for the​ patient? A. Excessive lateral movement of the mandible B. Rotation of the head and neck C. Hyperflexion of the neck D. Hyperextension of the neck

Hyperextension of the neck

Which of the following describes the condition of having an abnormally high body​ temperature? A. Hyperthermia B. Hyperdynamic state C. Septic shock D. Heat shock

Hyperthermia

Shock is the circulatory​ system's failure to provide sufficient blood and oxygen to all the​ body's tissues. Which of the answers is NOT a major type of​ shock? A. Hemorrhagic B. Cardiogenic C. Hypervolemic D. Hypovolemic

Hypervolemic

Which of the following must be treated by the EMT if present in a patient with an apparent behavioral​ emergency? A. Severe clinical depression B. Suicidal ideology C. Hypoglycemia D. Acute alcohol intoxication

Hypoglycemia

Which of the following is another name for describing the condition of​ shock? A. Hypoperfusion B. Hypotension C. Internal bleeding D. Hemorrhage

Hypoperfusion

Which of the following signifies a failure in the​ patient's compensatory response to blood​ loss? A. ​Pale, cool skin B. Tachypnea C. Hypotension D. Tachycardia

Hypotension

Your trauma patient is​ anxious, tachycardic,​ pale, and hypotensive. Which of these signs indicates failure of the​ body's compensatory​ mechanisms? A. Anxiousness B. Pallor C. Tachycardia D. Hypotension

Hypotension

Which of the following is NOT a classification of localized cold​ injury? A. Frostnip B. Frostbite C. Hypothermia D. All of the above

Hypothermia

Which of the following may cause a patient to exhibit abnormal​ behavior? A. Hypoxia B. Snakebites C. Gastroenteritis D. Allergic reaction

Hypoxia

Localized cold injury occurs due to vasoconstriction​ and:

Ice crystal formation in the tissues.

In which of the following circumstances should a helmet be​ removed? A. If you suspect a skull fracture and need to palpate the head B. If the helmet interferes with airway management In which of the following circumstances should a helmet be​ removed? C. If you want to place a nasal cannula on the patient but cannot because his ears are covered by the helmet D. If the helmet fits so snugly that you cannot inspect the ears for the presence of blood or fluid

If the helmet interferes with airway management In which of the following circumstances should a helmet be​ removed?

Which of the following is NOT true concerning a​ patient's reaction to toxic gas​ exposure? A. Signs and symptoms may be immediate and severe. B. The patient may present without respiratory difficulty. C. If the patient is treated​ immediately, there will be no​ long-term effects. D. Signs and symptoms may be delayed.

If the patient is treated​ immediately, there will be no​ long-term effects.

Which one of the following is incorrect in the application of a​ tourniquet? A. If​ possible, the tourniquet should be placed on a joint. B. The tourniquet should be 2 to 4 inches wide. C. A blood pressure cuff can be used as a tourniquet. D. The tourniquet should be placed approximately 2 inches above the bleeding.

If​ possible, the tourniquet should be placed on a joint.

When deciding where to transport a patient who is in hypovolemic shock or who has the potential for developing hypovolemic​ shock, which of the following is the MOST important service to be provided by the receiving​ hospital? A. Rehabilitation services B. Immediate surgical capabilities C. Availability of a chaplain D. ​Critical-care nursing

Immediate surgical capabilities

Your patient is a 28yearold male who was ejected from his motorcycle after striking a parked vehicle. He has multiple deformities to his upper and lower extremities on both sides. Which of the following would be the BEST way to immobilize this patient's extremities prior to transport? A. Immobilize the patient to a long backboard without splinting the extremities individually. B. Use traction splints for the lower extremities and allow the upper extremities to be immobilized by the long backboard. C. Use padded board splints for the upper extremities and PASG for the lower extremities. D. Use moldable splints for the upper and lower extremities, padding any voids to fully stabilize the fractures.

Immobilize the patient to a long backboard without splinting the extremities individually.

Which of the following causes worsening of the damage in a brain​ injury? A. Failure to keep the patient awake and talking B. Improper management of airway and ventilation C. Administration of​ 100% oxygen D. Allowing seepage of cerebrospinal fluid​ (CSF) from the ears or nose

Improper management of airway and ventilation

Distinguishing between a knee dislocation and a patella dislocation can sometimes be difficult. Which of the following statements is NOT true? A. In a knee dislocation, the tibia is forced anteriorly or posteriorly in relation to the distal femur. B. In a patellar dislocation, the knee will be stuck in flexion but the knee cap will not be displaced. C. You should always check for a distal pulse. D. The lack of a distal pulse could be a signal of a real emergency.

In a patellar dislocation, the knee will be stuck in flexion but the knee cap will not be displaced.

Which of the following is NOT a general rule of splinting? A. Align longbone injuries to anatomical position. B. Expose the injury. C. Assess distal CSM. D. In order to avoid loss of use of a limb, it is important to splint before moving, even if the patient is unstable.

In order to avoid loss of use of a limb, it is important to splint before moving, even if the patient is unstable.

Where are automatic implanted cardiac defibrillators typically placed in patients who require​ one? A. In the upper right chest B. In the upper left chest C. In the lower right chest D. In the lower left chest

In the upper left chest

The senior EMT on the first vehicle that arrived on a scene with multiple patients should assume which of the following roles until relieved by a senior​ official? A. Incident Commander of the scene B. Triage sector supervisor C. Transport sector supervisor D. Commander of the entire incident

Incident Commander of the scene

A​ 65-year-old man was doing some work on his roof when he lost his footing and fell to the​ ground, approximately 15 feet. He is unconscious but his respirations are normal. You note an obviously angulated left leg. You are more concerned about a possible head injury. Which of the following would indicate a possible head​ injury? A. Glasgow Coma Scale​ (GCS) of 15 B. Increased pulse rate C. Increased blood pressure D. Constricted pupils

Increased blood pressure

As pressure within the cranium​ increases, which of the following is the​ result? A. Increased blood​ pressure, increased pulse B. Increased blood​ pressure, decreased pulse C. Decreased blood​ pressure, increased pulse D. Decreased blood​ pressure, decreased pulse

Increased blood​ pressure, decreased pulse

A bulging fontanelle in a quietly resting child may be an indication of which of the​ following? A. Anxiety and distress B. Normal development C. Increased intracranial pressure D. Dehydration

Increased intracranial pressure

You are treating a​ 54-year-old female patient who was involved in a domestic dispute. You notice an abrasion to the side of her head. The patient is unresponsive with a blood pressure of​ 200/110, a pulse of 60 beats per​ minute, and slightly irregular breathing. The​ patient's presentation is most likely caused by which of the​ following? A. Increased arterial pressure B. Increased intracranial pressure C. ​Coup-contrecoup injury D. Minor closed head injury

Increased intracranial pressure

Which of the following is NOT a warning sign or physical symptom of​ stress? A. Increased salivation B. Heart palpitations C. Chest pain D. GI distress

Increased salivation

Which of the following is NOT an indication of shock in an infant or small​ child? A. Capillary refill greater than 2 seconds B. Not producing wet diapers C. Increased urine output D. Absence of tears when crying

Increased urine output

Which of the following should bring to mind the possibility of abuse or neglect of your special needs​ patient? A. Empathy for the​ patient's pain and suffering B. An insistence that the patient be transported to the hospital C. A lack of knowledge of the​ patient's condition D. Indifference to the​ patient's condition

Indifference to the​ patient's condition

Your patient is a 12yearold female who fell onto her outstretched hands while rollerblading. She has a deformity of her forearm, about 2 inches proximal to her wrist. This injury is a result of which of the following mechanisms? A. Indirect force B. Sudden acceleration C. Direct force D. Twisting motion

Indirect force

Which of the following does NOT make infants and children more prone to​ hypothermia? A. Little body fat B. Large body surface area C. Small muscle mass D. Inefficient metabolism

Inefficient metabolism

Which of the following is optional​ equipment? A. Flashlights B. Infant oxygen masks C. Disinfectant solution D. Lubricating jelly

Infant oxygen masks

Carbon monoxide poisoning occurs by which of the following​ routes? A. Radiation B. Absorption C. Inhalation D. Aspiration

Inhalation

Your patient is a​ 16-year-old male who was ejected from an​ all-terrain vehicle and struck his head on a large rock. He was not wearing a helmet. He is unresponsive with​ shallow, irregular​ respirations; a blood pressure of​ 170/110 mmHg; and a heart rate of 50 beats per minute. Which of the following interventions would be MOST​ appropriate? A. Insert a nasopharyngeal airway and ventilate via​ bag-valve-mask at​ 10-12 breaths per minute. B. Insert an oropharyngeal airway and give oxygen by nonrebreather mask at 10 lpm. C. Insert a nasopharyngeal airway and give oxygen by nonrebreather mask​ (NRB) at 15 lpm. D. Insert an oropharyngeal airway​ (OPA) and ventilate via​ bag-valve mask​ (BVM) at 30 breaths per minute.

Insert a nasopharyngeal airway and ventilate via​ bag-valve-mask at​ 10-12 breaths per minute.

You have responded to a local pool for a drowning patient. Lifeguards have pulled the patient out of the pool prior to your arrival. You find the​ 16-year-old patient unresponsive with agonal respirations and a weak carotid pulse. What is your first​ action? A. Apply​ high-concentration oxygen by​ bag-valve mask. B. Insert an oropharyngeal airway. C. Insert a Combitube. D. Apply​ high-concentration oxygen by nonrebreather mask.

Insert an oropharyngeal airway.

Which of the following traumatic conditions may account for signs and symptoms of a behavioral​ emergency? A. ​10% BSA partial thickness burn B. Internal hemorrhage C. Fractured lumbar vertebra D. Fractured tibia

Internal hemorrhage

What is the BEST option an EMT has when encountering poor BVM compliance when attempting to ventilate a​ patient? A. Address ventilation en route to the hospital or ALS intercept. B. Begin chest compressions. C. Place the patient on supplemental oxygen. D. Involve two people in the procedure.

Involve two people in the procedure.

Which of the following would you expect to see in a patient with severe​ hypothermia? A. Tachycardia B. Irrational behavior C. Loss of muscle tone D. Rapid respirations

Irrational behavior

Your patient is a​ 2-year-old conscious male who has swallowed a small quantity of bleach. Which of the following findings should you evaluate the patient​ for? A. Irregular pulse B. Respiratory depression C. Irritation in and around the mouth D. Dilated pupils

Irritation in and around the mouth

Which of the following is most characteristic of parietal abdominal pain? A. Is lying on the floor very still and quiet with his knees drawn up to his chest. B. Is rolling about on the floor complaining of pain. C. Walks out to the ambulance informing you he has the worst "belly ache." D. Is sitting upright in a chair, moaning in pain, and drinking antacid.

Is lying on the floor very still and quiet with his knees drawn up to his chest.

Which of the following is an appropriate question to ask while evaluating a woman in​ labor? A. Is this your first​ pregnancy? B. When was the last time you were sexually​ active? C. Do you know who the father is and what is his medical​ history? D. None of the above

Is this your first​ pregnancy?

Which of the following is NOT the purpose of making airway management the highest priority of patient care when managing the patient in​ shock? A. It allows for improved elimination of carbon dioxide. B. It minimizes the chances of aspiration of blood or vomit. C. It allows for oxygenation of the lungs. D. It allows the bronchoconstriction of the smaller airways to be reversed.

It allows the bronchoconstriction of the smaller airways to be reversed.

Which of the following is a characteristic of venous​ bleeding? A. It cannot lead to​ life-threatening amounts of blood loss. B. It commonly requires the use of pressure point compression. C. It often requires the use of a tourniquet. D. It can be​ profuse, but is generally easily controlled.

It can be​ profuse, but is generally easily controlled.

Which of the following statements regarding syrup of ipecac is NOT​ true? A. It can cause a patient to aspirate stomach contents into the lungs. B. It removes only about​ one-third of stomach contents. C. It has an immediate action. D. It typically causes emesis in patients with a single dose.

It has an immediate action.

What is the purpose of the Rule of Nines for burn​ injuries? A. It helps to determine the age of the burn. B. It helps to determine the extent of the surface area burned. C. It helps to determine the depth of the burn. D. It helps to determine the need for oxygen.

It helps to determine the extent of the surface area burned.

Regarding activated​ charcoal, which of the following is NOT ​true? A. Ingestion of strong acids or alkalis is a contraindication to its use. B. It can reduce the amount of poison absorbed by the gastrointestinal system. C. It is an antidote to many poisons. D. It does not work on all poisons.

It is an antidote to many poisons.

Which of the following is a characteristic of referred pain? A. It is felt in a location other than the organ causing it. B. It is usually described as "crampy" or "colicky." C. It is only felt in hollow organs. D. It is caused by psychological stress.

It is felt in a location other than the organ causing it.

Which of the following is true concerning parietal pain? A. It is generally localized to a particular area. B. It is often described as "crampy" or "colicky." C. It is usually intermittent in nature. D. It arises from solid organs.

It is generally localized to a particular area.

Which of the following statements is NOT true when you are​ on-scene and treating a patient that appears to be in​ shock? A. Airway management is of top priority. B. It is important to spend​ on-scene time to be sure you have corrected and dealt with any of the causes of the shock so that it does not get worse. This is more important than rapid transport. C. Prompt transportation is a very high priority. D. The patient should be promptly put on​ high-concentration oxygen.

It is important to spend​ on-scene time to be sure you have corrected and dealt with any of the causes of the shock so that it does not get worse. This is more important than rapid transport.

Which of the following is NOT a desirable characteristic of the rehabilitation sector of a hazardous materials​ incident? A. It is protected from weather elements. B. It is large enough to accommodate multiple rescue crews. C. It allows for rapid​ re-entry to the emergency operation. D. It is located in the warm zone.

It is located in the warm zone.

Which statement regarding the trachea in pediatric patients is​ TRUE? A. It is more rigid than in adults. B. It is not easily obstructed. C. It is harder than in adults. D. It is narrower than in adults.

It is narrower than in adults.

Which one of the following statements is NOT​ true? A. In any given​ year, 26.2% of adult Americans suffer from a diagnosable mental disorder. B. It is very unusual for an EMT to be called for a psychiatric emergency. C. Just over​ 20% of the population of adult Americans has anxiety or panic issues. D. Almost​ 10% of the population of adult Americans has a mood disorder such as depression.

It is very unusual for an EMT to be called for a psychiatric emergency.

Which of the following statements concerning behavioral emergencies is true​? A. The person with a behavioral emergency requires prolonged institutionalization. B. A person with a behavioral emergency is mentally ill. C. Emotional outbursts are considered behavioral emergencies. D. It may be difficult to determine what behavior is abnormal for a given person in a given situation.

It may be difficult to determine what behavior is abnormal for a given person in a given situation.

Which of the following BEST describes the benefit of a three-sided occlusive dressing over a four-sided occlusive dressing for an open chest wound? A. It eliminates the need to continue monitoring the patient's respiratory status. B. It reduces the chances of developing a tension pneumothorax. C. It allows easy access for re-examination of the wound en route to the hospital. D. It prevents the development of a hemothorax by allowing blood to escape.

It reduces the chances of developing a tension pneumothorax.

Which of the following is NOT a benefit of splinting an injury to bones and connective tissues? A. It prevents neurological damage due to movement of bone ends or fragments. B. It restricts blood flow to the site of the injury to prevent swelling. C. It reduces pain. D. It may prevent a closed injury from becoming an open injury.

It restricts blood flow to the site of the injury to prevent swelling.

53. Which of the following statements is not true concerning the proper transport of an avulsed ear? a. It should be labeled with the patient's name, as well as the date and time it was bagged. b. It should be kept as cool as possible. c. It should not be immersed in cooled water or saline. d. It should be in a dry sterile dressing.

It should be in a dry sterile dressing.

Which of the following statements is NOT true concerning a pelvic wrap? A. You can consider its use based on mechanism of injury (MOI). B. It can be used if the pelvis shows instability. C. It should not be used unless the patient shows signs of shock. D. It can be used if the pelvis shows deformity.

It should not be used unless the patient shows signs of shock.

Which of the following BEST describes the function of​ blood? A. It​ clots, flows,​ transports, protects, and excretes on a daily basis. B. It is a​ life-giving liquid that supports all the​ body's functions to maintain hypoperfusion. C. It flows from the heart with the vital gases and nutrients to maintain lack of perfusion. D. It transports gases along with​ nutrients, aids in​ excretion, and provides protection and regulation.

It transports gases along with​ nutrients, aids in​ excretion, and provides protection and regulation.

Why is it important to gather a detailed medical history when dealing with a psychiatric​ emergency? A. It will alert you to past issues as well as medication. B. It is not. You want to get the call resolved as quickly as possible and this will just delay it. C. It will help you to determine if the police are needed. D. It will help you to determine if you need to restrain the patient.

It will alert you to past issues as well as medication.

Which of the following statements regarding behavioral emergencies is NOT​ true? A. Patients experiencing a behavioral emergency are not always dangerous to themselves or others. B. It would be unusual to find a person with schizophrenia outside a mental health facility. C. Differences in culture may initially appear to be abnormal behaviors to the EMT. D. Apparent behavioral emergencies may be due to an underlying medical cause.

It would be unusual to find a person with schizophrenia outside a mental health facility.

Which of the following organs is located in the retroperitoneal space? A. Kidneys B. Uterus C. Stomach D. Liver

Kidneys

Which of the following structures is (are) NOT located in the abdominal cavity? A. Liver B. Kidneys C. Stomach D. Spleen

Kidneys

Which of the following is classified as an open head​ injury? A. Contusion without a skull fracture B. Laceration with a skull fracture C. Laceration without a skull fracture D. Both A and B

Laceration with a skull fracture

Seizures due to complications of pregnancy generally occur during which of the following time​ periods? A. In the second trimester B. In the first trimester C. Before the mother even knows she is pregnant D. Late in pregnancy

Late in pregnancy

The technique for central rewarming requires the application of heat to which of the following areas of the​ patient's body? A. ​Head, neck,​ chest, and groin B. ​Chest, back,​ neck, and armpits C. ​Head, neck,​ chest, and back D. Lateral​ chest, neck,​ armpits and groin

Lateral​ chest, neck,​ armpits and groin

Your patient is a 32-year-old man with a fish hook that has perforated his hand between the thumb and index finger. Which of the following is the best way to manage the situation in the prehospital setting? A. Push the hook through the wound to avoid further damage from the barbed end. B. Leave the hook in place and try not to disturb it. C. Apply a pressure dressing over the hook. D. Pull the hook out from the same direction in which it entered the hand.

Leave the hook in place and try not to disturb it.

Your patient is a 40yearold female who has been experiencing abdominal pain and vomiting for 2 days. She is now responsive to verbal stimulus; has cool, dry skin; a heart rate of 116; respirations of 24; and a blood pressure of 100/70. Which of the following is the BEST position for transporting this patient? A. Sitting up at a 90degree angle B. Supine with the knees bent C. Left lateral recumbent with the legs bent D. Sitting up at a 45 degree angle

Left lateral recumbent with the legs bent

Which of the following transport positions is indicated for the patient in​ shock? A. Legs raised 8 to 12 inches B. Recumbent C. Sitting D. Left lateral recumbent

Legs raised 8 to 12 inches

Which of the following is typically helpful in assessing a younger pediatric​ patient? A. Ask the parent to direct the child to cooperate. B. Remove the child from the parent or guardian before starting the assessment. C. Tell the child that you need him or her to cooperate during the assessment. D. Let the parent hold the child when possible.

Let the parent hold the child when possible.

You respond to a​ 22-year-old male patient who fell while exiting the local bar. Bystanders state he drank at least 10 beers and could not keep his balance. Physical exam reveals that the patient is alert to verbal stimuli only. He has a Glasgow Coma Scale of​ 3, 4,​ 6; slurred​ speech; and an obvious scalp laceration to the back of his head. He is refusing treatment and transport and wants his friends to drive him home. The nearest hospital is 5 minutes​ away, a Level II Trauma Center is 10 minutes​ away, and a Level I Full Service Trauma Center is 30 minutes away. Which of these is the most appropriate facility for the​ patient? A. Level I Trauma Center B. Level II Trauma Center C. The nearest community facility D. ​Nowhere, since the patient is an adult and​ refusing; as​ such, you cannot take him

Level II Trauma Center

You are at the scene of a motor vehicle collision in which a single vehicle has collided with a tree. You note that the​ driver's side air bag has deployed. Which of the following is recommended by air bag​ manufacturers? A. Sharply strike the front bumper to make sure the second air bag has deployed. B. Cut away the fabric portion of the air bag to get it out of the way. C. Use a portable fan to ventilate the car to remove the powder from the air bag. D. Lift the air bag and inspect the steering wheel for damage.

Lift the air bag and inspect the steering wheel for damage.

Which of the following structures connect bone ends, making joints more stable? A. Cartilage B. Periosteum C. Tendons D. Ligaments

Ligaments

Which of the following is a vascular organ in the abdomen that can produce blood loss quickly enough to result in life-threatening hemorrhage following high mechanism of injury blunt trauma? A. Intestines B. Pancreas C. Liver D. Kidneys

Liver

Which of the following distinguishes decompensated shock from compensated stage of​ shock? A. Tachycardia B. Delayed capillary refill time C. Low blood pressure D. Altered mental status

Low blood pressure

Which of the following signs is LEAST likely to indicate a traumatic brain​ injury? A. Irrational behavior B. Vomiting C. Irregular breathing pattern D. Low blood pressure

Low blood pressure

Which of the following problems is MOST likely in a child who is​ wheezing? A. Lower airway disorder B. Shallow breathing C. Partial upper airway obstruction D. Fluid in the oropharynx

Lower airway disorder

An elderly patient you are transporting to the hospital complains of feeling faint and abdominal pain. The patient states a recent history of​ "tarry stool". Which of the following conditions is most​ likely? A. Constipation B. Myocardial infarction C. Abdominal aortic aneurysm D. Lower gastrointestinal bleeding

Lower gastrointestinal bleeding

During​ transport, where should the catheter bag be after loading the patient and stretcher into the ambulance if the patient has an indwelling urinary​ catheter? A. Lower than the​ patient, but not on the floor B. In the​ patient's lap, but not higher than his heart C. On the ambulance floor D. Hanging from the ceiling of the ambulance or IV pole higher than the patient

Lower than the​ patient, but not on the floor

Supine hypotensive syndrome is easily prevented by transporting the pregnant female into which of the following​ positions? A. On her hands and knees with her hips elevated B. Tilted slightly onto the right side C. ​Supine, with the head lower than the hips D. Lying on her left side

Lying on her left side

While treating a patient of sexual​ assault, your treatment should follow which of the following​ sequences? A. Maintain scene​ safety, treat immediate life​ threats, allow the patient to shower if the patient is capable to help treat psychological​ needs, and transport. B. Treat immediate life​ threats, treat psychological​ needs, and protect criminal evidence. C. Maintain scene​ safety, treat immediate life​ threats, and treat only the secondary injuries that may become life threats to protect criminal evidence. D. Maintain scene​ safety, treat immediate life​ threats, treat medical and psychological​ needs, and protect criminal evidence.

Maintain scene​ safety, treat immediate life​ threats, treat medical and psychological​ needs, and protect criminal evidence.

You are dispatched to a suicide attempt. You arrive to find a​ 16-year-old, who is extremely agitated and pacing up and down in the living room of his house.​ Apparently, he had threatened to go​ out, get a​ gun, and shoot himself. The parents called it in as an attempted suicide. The scene is safe and there are apparently no weapons accessible to the patient. Which of the following would NOT be appropriate in caring for this​ patient? A. Do not take any action that may be considered threatening by the patient. To do so may bring about hostile behavior directed against you or others. B. Do not isolate yourself from your partner or other sources of help. C. Make certain the patient gets between you and the door. The patient should always feel he has an escape route. D. Always be on the watch for weapons.

Make certain the patient gets between you and the door. The patient should always feel he has an escape route.

Which of the following is acceptable when managing the patient with a behavioral or psychiatric​ emergency? A. Go along with the​ patient's hallucinations or false beliefs. B. Make supportive statements such​ as, "That must have been very hard for​ you." C. Allow family members to confront the patient about his behavior. D. Sit as close to the patient as you can to reassure him that you will not abandon him.

Make supportive statements such​ as, "That must have been very hard for​ you."

Which of the following is important to remember when communicating with an elderly patient with a hearing​ impairment? A. It is not worth the effort to try to get information from an elderly person with a hearing impairment. B. Make sure the patient can see you when you are speaking to him. C. You may need to shout in order for the patient to hear you. D. Speak directly into the​ patient's ear.

Make sure the patient can see you when you are speaking to him.

You are called to the scene of an attempted suicide. You arrive to find a​ 25-year-old man sitting on the sofa who apparently cut his wrists. Family members have bandaged​ them, and there does not appear to be any bleeding risk at this time. The scene is secure. Which action would you NOT take in treating this​ patient? A. Contact the receiving hospital and report on current mental status and other essential information. B. Make sure you take charge of the situation. Let the patient know that what he has done is wrong. Tell him he is coming with you whether he likes it or not. Do not worry about gaining the​ patient's confidence. Take charge. The patient must know that you are the boss. C. As soon as​ possible, perform a history and physical exam and provide emergency care on the​ wrist, if necessary. D. Perform a detailed physical exam only if it is safe and you suspect the patient may have an injury.

Make sure you take charge of the situation. Let the patient know that what he has done is wrong. Tell him he is coming with you whether he likes it or not. Do not worry about gaining the​ patient's confidence. Take charge. The patient must know that you are the boss.

What is another name for the zygomatic​ bone? A. Maxillae B. Malar C. Mandible D. Temporal

Malar

Which of the following is a strategy to maintain an occlusive dressing to bloody or diaphoretic skin? A. Do not use occlusive dressings in this case. B. Do not use adhesive tape. C. Manually maintain pressure. D. Wrap the dressing circumferentially with gauze.

Manually maintain pressure.

You are dispatched to a motorcycle crash with one patient involved. Which of the following interventions should be performed​ first? A. Stabilize the pelvis to a long spine board. B. Begin chest compressions. C. Manually stabilize the cervical spine. D. Suction the vomit and secretions from the airway.

Manually stabilize the cervical spine.

What is the first step in a rapid takedown of a standing patient A. Applying oxygen B. Positioning a long spine board behind the patient C. Manually stabilize the​ patient's head and neck D. Applying a properly sized cervical collar

Manually stabilize the​ patient's head and neck

Which of the following statements regarding musculoskeletal injuries is correct? A. Most musculoskeletal injuries are simply splinted and not a life threat to the patient. B. All musculoskeletal injuries are life threatening due to the bone bleeding, leading to hypoperfusion. C. Splints do not adequately fit the patient's extremities and must be modified with padding to ensure immobilization. D. Many musculoskeletal injuries have a grotesque appearance, and the EMT cannot be distracted from lifethreatening conditions by a deformed limb.

Many musculoskeletal injuries have a grotesque appearance, and the EMT cannot be distracted from lifethreatening conditions by a deformed limb.

Which of the following types of bleeding is most​ serious? A. Arterial bleeding B. External bleeding C. Internal bleeding D. Massive bleeding of any type

Massive bleeding of any type

While assessing a 78-year-old male patient who escaped an apartment fire with partial thickness burns to both arms, the EMT must be aware of which of the following? a. Being involved in a crime makes the patient part of the chain of evidence, requiring a police officer to ride with you to the hospital. b. The burn is the most serious injury to the patient. c. Medical conditions may be aggravated by the burn. d. The patient may need to be questioned by police and fire officials about the cause of the fire.

Medical conditions may be aggravated by the burn.

Upon your arrival for a medical call at a private​ residence, you find an elderly male patient complaining of difficulty breathing. Which of the following findings would provide you with immediate information about the​ patient's condition? A. Baseline vital signs B. Temperature in the house C. Medication list D. Mental status

Mental status

Which of the following substances may be ordered by medical control to dilute a​ poison? A. Milk or milk of magnesia B. Milk or water C. Milk of magnesia or magnesium citrate D. Water with syrup of ipecac

Milk or water

Which of the following statements about burn management in pediatric patients is​ TRUE? A. Moist dressings should be used with caution. B. It is best to keep the patient exposed to minimize the risk of infection. C. Always apply​ wet, sterile burn sheets. D. Pediatric patients are not prone to hypothermia due to burns.

Moist dressings should be used with caution.

On assessment of the midsection of a 32-year-old male who was struck by a car, you find an abdominal evisceration with several loops of his large intestine exposed. The abdomen appears to have a clean-cut laceration and the bleeding is controlled. Which of the following is the BEST approach toward managing the exposed intestines? A. Leave the abdominal contents in the place in which they were found and transport immediately. B. Cover the abdomen with an occlusive dressing of aluminum foil. C. Gently replace the intestines after moistening with sterile saline solution. D. Moisten a sterile dressing with saline solution and cover the abdominal contents.

Moisten a sterile dressing with saline solution and cover the abdominal contents.

You respond to the scene of a local campground. Your patient is a​ 15-year-old female patient who was stung in the arm by a bee. The patient is anxious and hyperventilating at 28 times a minute. The patient is alert and oriented to​ time, place,​ person, and event. Lung sounds are clear bilaterally. The left arm is swollen and the stinger is not present. Blood pressure is​ 118/72 and pulse is 110. The​ patient's mother states she is allergic to bee stings and has an epinephrine​ auto-injector. She called 911 because she was afraid the patient would stop breathing. Which of the following is the best treatment​ plan? A. Place the patient in the Trendelenburg position. B. Place a constricting band around the arm to minimize the spread of the venom. C. Assist the patient in administering her epinephrine​ auto-injector. D. Monitor the patient for shock and transport.

Monitor the patient for shock and transport.

Which of the following is of greatest concern for the EMT in the prehospital care of a woman with vaginal bleeding? A. Obtaining a thorough gynecological history B. Monitoring for hypovolemic shock C. Preventing infection D. Finding out if the patient is currently sexually active

Monitoring for hypovolemic shock

Which of the following is of greatest concern for the EMT in the prehospital care of a woman with vaginal​ bleeding? A. Preventing infection B. Finding out if the patient is currently sexually active C. Monitoring for hypovolemic shock D. Obtaining a thorough gynecological history

Monitoring for hypovolemic shock

Which of the following statements about injuries to pediatric patients is​ TRUE? A. Most injuries to children are caused by blunt trauma. B. Most injuries to children are caused by penetrating trauma. C. Most injuries to children are caused by abuse. D. Most injuries to children are unavoidable accidents.

Most injuries to children are caused by blunt trauma.

Which of the following statements is true regarding helicopter​ transport? A. Helicopter transport is relatively inexpensive. B. Patient care is seldom affected during the flight. C. Pressure changes do not affect equipment such as​ PASG, ET​ tubes, and so on. D. Most treatment should be done prior to loading the patient.

Most treatment should be done prior to loading the patient.

Which of the following is the MOST common type of​ multiple-casualty incident that EMS providers will respond​ to? A. Hazardous materials incidents B. Motor vehicle collisions C. Structure fires D. Outbreaks of influenza

Motor vehicle collisions

Because infants and small children rely more heavily on the diaphragm for​ breathing, respiratory distress can be detected by observing which one of the following signs that is not prominent in​ adults? A. Bulging of the intercostal spaces B. Expansion of the chest on inhalation C. Use of the sternocleidomastoid muscles of the neck D. Movement of the abdomen with respiratory effort

Movement of the abdomen with respiratory effort

Which of the following situations are frequently problematic for patients who have tracheostomy​ tubes? A. Air embolism B. Tracheal tears C. Mucus buildup D. Tube malplacement

Mucus buildup

What is the definition of multisystem​ trauma? A. A trauma in which there are multiple casualties B. Multiple injuries that affect more than one body system C. A trauma that requires the response of multiple agencies D. Trauma in which the patient has more than one serious injury

Multiple injuries that affect more than one body system

How often do most peritoneal dialysis patients require​ treatment? A. Once a day B. Multiple treatments a day C. Once a week D. Three to five times a week

Multiple treatments a day

Which of the following is NOT a cause of parietal pain? A. Muscle spasm B. Bleeding into the abdominal cavity C. Inflammation D. Infection

Muscle spasm

When you are assessing a​ child's appearance in the Pediatric Assessment​ Triangle, which of the following is observed when assessing the​ patient's tone? A. Skin color B. Muscle tone C. Pitch of the voice or cry D. Apparent mood or emotion

Muscle tone

Chemical cold packs are carried on the ambulance for treatment of which of the​ following? A. Abdominal pain B. Seizures C. Musculoskeletal injuries D. Snake bites

Musculoskeletal injuries

Pain felt in the epigastric region of the abdomen is of concern because of the possibility of which of the following? A. Kidney stones B. Extreme diarrhea with dehydration C. Influenza D. Myocardial infarction

Myocardial infarction

OxyContin is an example of which of the following types of​ drugs? A. Antidepressant B. Narcotic C. Barbiturate D. Appetite suppressant

Narcotic

Which of the following types of drugs may induce sleep or​ stupor? A. Tranquilizers B. Narcotics C. Hallucinogens D. Both A and B

Narcotics

Which of the following is a common sign and symptom in poisonings of all​ types? A. Headache and diaphoresis B. Dizziness and palpitations C. Nausea and vomiting D. Pain and diaphoresis

Nausea and vomiting

You are called by law enforcement to evaluate a toddler who was found alone in an apartment. Neighbors state that the child is often left home alone. What form of abuse is​ this? A. Emotional abuse B. Psychological abuse C. Physical abuse D. Neglect

Neglect

You are treating a​ 35-year-old male patient that has been involved in a motorcycle incident. The patient is unresponsive with a blood pressure of​ 60/40, a pulse of 66 beats per​ minute, and respirations of 18 breaths per minute. The​ patient's presentation is most likely caused by which of the​ following? A. Septic shock B. Increased intracranial pressure C. Cardiogenic shock D. Neurogenic shock

Neurogenic shock

Which of the following statements is true when talking about neurogenic​ shock? A. Neurogenic shock is very common in the field. B. Neurogenic shock is sometimes caused by spinal injuries. C. Neurogenic shock is caused by the blood vessels overfilling with​ blood, causing leaking into the nerves. D. Neurogenic shock is the result of the blood vessels decreasing in size.

Neurogenic shock is sometimes caused by spinal injuries.

Which of the following should be considered by the EMT as a result of the proportionally larger size of a small​ child's head? A. A child is susceptible to serious brain trauma. .B. It will be difficult to obtain a good face mask seal for ventilations. C. Neutral alignment of cervical spine and airway will be difficult. .D. Hyperextension of the neck can close off the trachea.

Neutral alignment of cervical spine and airway will be difficult.

There are general rules when dealing with psychiatric emergencies. Which of the following is NOT one of those rules and would be considered​ inappropriate? A. Identify yourself and your role. B. Listen to the patient. You can show you are listening by repeating part of what the patient says back to him. C. Never make eye contact with the​ patient, as it will just increase his nervousness. D. Speak slowly and​ clearly, using a calm and reassuring tone.

Never make eye contact with the​ patient, as it will just increase his nervousness.

A 36-year-old male was accidentally shot with a nail gun into the chest. You see the nail, which protrudes about 2 to 3 centimeters from the thorax, when you visualize the injury site. Under which of the following circumstances should you remove the nail from the injury site? A. The patient begins to complain of shortness of breath. B. The patient develops a tension pneumothorax. C. Bleeding from the patient's wound is minimal. D. None of the above

None of the above

A 36-year-old man has accidentally shot a nail into his thigh while using a nail gun. Under which of the following circumstances should the EMT remove the nail from the injury site? a The nail is less than 2 inches in length. b The patient's distal pulse, motor function, and sensation are intact. c Bleeding from the wound is minimal. d None of the above

None of the above

A​ 36-year-old male was accidentally shot with a nail gun into the head. You see the​ nail, which protrudes about 2 to 3 centimeters from the​ skull, when you visualize the injury site. Under which of the following circumstances should you remove the nail from the injury​ site? A. Bleeding from the​ patient's wound is minimal. B. The patient develops excessive intracranial pressure​ (ICP). C. The patient begins to complain of shortness of breath. D. None of the above

None of the above

Which of the following statements is believed to be true regarding domestic elder​ abuse? A. The elders are usually in good health. B. There is seldom a family history of elder abuse. C. The abusers are usually​ well-adjusted. D. None of the above.

None of the above

Which of the following agencies have developed regulations for dealing with hazardous materials​ emergencies? A. DOT and FCC B. NHTSA and NRC C. NAEMT and NHTSA D. OSHA and EPA

OSHA and EPA

Bariatrics is defined as the branch of medicine that deals with the​ cause, prevention, and treatment of which of the following​ diseases? A. Obesity B. Sleep apnea C. Diabetes D. Blindness

Obesity

Concerning​ aging, which of the following statements is NOT​ true? A. Some elderly people have a low threshold for pain. B. The systolic blood pressure tends to increase with age. C. Many elderly patients have a high threshold for pain. D. Older people are less likely to use EMS compared to younger people.

Older people are less likely to use EMS compared to younger people.

You are treating a 38yearold female patient with abdominal distress. The patient's vital signs are stable and you are getting ready to transport. What is the BEST position to transport the patient? A. Trendelenburg B. Right lateral C. On side with knees bent D. Left lateral

On side with knees bent

In what position should the patient complaining of severe abdominal pain be placed if there are no signs or symptoms of shock? A. Supine with feet elevated B. One of comfort C. Semi-Fowler with knees bent D. Left lateral recumbent

One of comfort

Occlusive dressings must be carried on an ambulance to treat which of the following​ injuries? A. Impaled object in the eye B. Tension pneumothorax C. Open pneumothorax D. Amputations

Open pneumothorax

Which of the following BEST describes an evisceration? A. Flap of skin that is partially or completely torn away from the underlying tissue B. Accumulation of blood beneath the skin, resulting in swelling C. Open wound of the abdomen from which organs protrude D. Epidermis that is scraped away by a rough surface

Open wound of the abdomen from which organs protrude

Which of the following is meant by the​ "1 percent​ rule" of​ aging? A. About 1 percent of the population makes it to their 85th birthday. B. We lose about 1 percent of our memory capacity every year beginning at age 50. C. Our organ systems lose about 1 percent of function per year beginning at age 30. D. About 1 percent of the population older than age 65 has significant organ function decline.

Our organ systems lose about 1 percent of function per year beginning at age 30.

When transporting a patient who is morbidly obese to the​ hospital, it is important to monitor which of the following vital​ signs? A. Oxygen saturation B. Blood glucose level C. Pulse rate D. Blood pressure

Oxygen saturation

Compare the pain a patient experiences when receiving a shock from a pacemaker against the pain felt when receiving a shock from an automatic implanted cardiac defibrillator​ (AICD). A. Both are painless. B. Pacemakers are painless and AICDs are painful. C. Pacemakers are painful and AICDs are painless. D. Both are painful.

Pacemakers are painless and AICDs are painful.

Which of the following is NOT part of a basic obstetrics​ kit? A. Packet of suture material B. Surgical scissors C. Umbilical cord clamps D. Baby blanket

Packet of suture material

Which of the following is NOT a common field finding in spinal​ injuries? A. Impaired breathing B. Tenderness C. Deformity D. Pain

Pain

When the EMT is assessing compromise to an extremity, perhaps due to an orthopedic injury, the EMT should initially check what "six Ps"? A. Pain, pallor, paresthesia, pulses, placement, and pressure B. Pain, pallor, position, pulses, placement, and pad C. Pain, pallor, position, pulses, placement, and pressure D. Pain, pallor, paresthesia, pulses, paralysis, and pressure

Pain, pallor, paresthesia, pulses, paralysis, and pressure

Your patient is a​ 27-year-old male who has been involved in a motorcycle collision in which he was not wearing a helmet. He does not respond when you speak to​ him, but he makes incomprehensible sounds when you press your knuckles on his sternum. Which of the following BEST describes his level of​ consciousness? A. Unresponsive B. Verbal C. Alert D. Painful

Painful

You are responding to a​ 6-year-old child with a fever and difficulty breathing. His mother reports that he was playing normally this morning but when he came in for lunch he had spiked a fever.​ Now, he is sitting up with his mouth​ open, drooling. Which of the following signs would point to​ epiglottitis? A. Painful swallowing B. Loud​ "seal bark" cough C. Some hoarseness D. ​Low-grade fever

Painful swallowing

If assisting in a prehospital delivery while​ off-duty, which of the following would be the BEST choice for tying or clamping the umbilical​ cord? A. Section of wire coat hanger B. Clothespin C. White cotton thread D. Pair of shoelaces

Pair of shoelaces

A method of assessing compromise to an extremity when a musculoskeletal injury is suspected is to learn and follow the six Ps. Which of the items below is NOT one of the six Ps? A. Paresthesia B. Pallor C. Parenthesis D. Pain

Parenthesis

Allowing a​ patient's body temperature to increase by preventing further heat loss is referred to as which of the​ following? A. Natural rewarming B. Active rewarming C. Passive rewarming D. Core rewarming

Passive rewarming

You respond to the scene of a​ 14-year-old patient. He is unresponsive and hypothermic. Emergency Medical Responders have moved the patient inside and secured the airway prior to your arrival. What is your next​ action? A. Vigorously rub the patient to increase the body temperature. B. Assess for signs of frostbite and treat immediately. C. Actively rewarm the patient. D. Passively rewarm the patient.

Passively rewarm the patient.

Which patient is the highest​ priority? A. Patient with a broken femur B. Patient with a Glasgow Coma Scale of 15 C. Patient with a penetrating chest injury D. Patient with a Revised Trauma Score of 12

Patient with a penetrating chest injury

Which trauma patient is the most​ critical? A. Patient with a Glasgow Coma Scale of 7 B. Patient with decerebrate posturing C. Patient who withdraws to painful stimuli D. Patient with decorticate posturing

Patient with decerebrate posturing

You are attending to an​ end-stage renal disease​ (ESRD) patient who has missed dialysis. Which of the following statements is most​ accurate? A. Calling for ALS would not be very helpful since they cannot really do anything that an EMT cannot do in these situations. B. Patients who have missed dialysis and who become unresponsive and pulseless do not respond very well to the use of an AED. C. Patients who have missed dialysis and who become unresponsive and pulseless respond very well to the use of an AED. D. You should delay transport if necessary to wait for ALS.

Patients who have missed dialysis and who become unresponsive and pulseless do not respond very well to the use of an AED.

Burns pose a greater risk to infants and children for which of the following reasons? a. Pediatric patients have a greater risk of heart problems associated with the burn. b. Pediatric patients have a greater risk of shock from the burn. c. Infants and children have a greater risk of infection from the burn. d. None of the above

Pediatric patients have a greater risk of shock from the burn.

You are called to the scene of a​ 17-year-old female patient who is unresponsive. Her mother suspects that she tried to commit suicide by taking her pain pills. The patient is unresponsive to painful​ stimuli, has agonal​ respirations, and has vomited. She has a weak carotid pulse. After securing the airway and providing oxygen by​ bag-valve mask, what is your next​ action? A. Perform a focused assessment. B. Perform a rapid physical examination of the patient. C. Ask the mother the name of her pain medication. D. Question the mother about the​ patient's suicidal tendencies.

Perform a rapid physical examination of the patient.

When providing emergency care to an aggressive or hostile​ patient, what is the highest​ priority? A. Finding out if the patient is oriented to​ person, time, and place B. Performing a scene​ size-up C. Calming the patient down D. Checking the patient for possible physical causes of the behavior

Performing a scene​ size-up

Which of the following BEST describes the delivery of oxygen and nutrients at the​ body's cellular​ level? A. Osmosis B. Hydrostatic pressure C. Circulation D. Perfusion

Perfusion

Which of the following systems includes the pairs of nerves that enter and exit the spinal cord between each pair of​ vertebrae? A. Autonomic nervous system B. Central nervous system C. Peripheral nervous system .D. All of the above

Peripheral nervous system

Which of the following is the main advantage of peritoneal dialysis over​ hemodialysis? A. Peritoneal dialysis is much faster. B. Peritoneal dialysis is more effective. C. Peritoneal dialysis is likely to be done at the​ patient's home. D. Peritoneal dialysis is less prone to infection.

Peritoneal dialysis is likely to be done at the​ patient's home.

Patients who dialyze at home are at high risk for what type of​ infection? A. Cellulitis B. Fistula infection C. Peritonitis D. Decubitus ulcers

Peritonitis

When responding to an attempted​ suicide, which of the following is the​ EMT's primary​ concern? A. Personal safety B. Contacting law enforcement since suicide is illegal C. Managing the​ patient's airway D. Determining whether the patient suffers from clinical depression

Personal safety

Which of the following is the most significant way in which the body cools​ itself? A. Radiation B. Respiration C. Perspiration D. Vasoconstriction

Perspiration

Which of the following is NOT relevant in determining whether or not delivery is imminent for a woman in​ labor? A. Asking how long ago the contractions began B. Determining if the patient feels as if she needs to move her bowels C. Phoning the​ patient's obstetrician for advice D. Finding out how many pregnancies the patient has had

Phoning the​ patient's obstetrician for advice

What trauma triage guidelines did the Centers for Disease Control and Prevention​ (CDC) release in order to guide the most injured patients into trauma​ centers? A. Determining patient​ priority, amount of time​ on-scene, and hospital transport decision B. ​Lights, sirens, and diesel C. Physiological​ determinants, anatomic​ criteria, and mechanism of injury D. ​Teamwork, timing, and transport

Physiological​ determinants, anatomic​ criteria, and mechanism of injury

Which of the following is indicated in the prehospital management of a prolapsed umbilical​ cord? A. Use your gloved hand to push the umbilical cord back up through the cervix. B. Encourage the mother to push forcefully in order to speed delivery. C. Immediately clamp the cord in two places and cut it between the clamps. D. Place the mother in a​ head-down position with pillows under her hips.

Place the mother in a​ head-down position with pillows under her hips.

You are dispatched to a private residence for a sick person. When you arrive you are told by the patient that due to the snow storm yesterday he missed his scheduled appointment at the dialysis center and is not feeling well. Your assessment does not reveal anything remarkable outside of the fact that he has missed his dialysis. Which of the steps below would NOT be part of your​ care? A. Place the patient in a supine position. B. When you obtain vital​ signs, obtain a blood pressure on an arm that does not have a fistula. C. Assess the ABCs. D. Administer oxygen at 15 lpm by nonrebreather mask.

Place the patient in a supine position.

Your patient is pregnant at 20​ weeks' gestation and has been thrown from a horse. She is complaining of back pain. Which of the following is the correct procedure for immobilizing her​ spine? A. Place the patient supine on the​ backboard, then put a pillow under the right side of the backboard. B. Use a short immobilization device and transport the patient in a sitting position. C. Place the patient on her left side on the backboard. D. Place the patient supine on the backboard.

Place the patient supine on the​ backboard, then put a pillow under the right side of the backboard.

What is the temporary organ of​ pregnancy, which functions to supply the developing fetus with oxygen and​ nutrients? A. Uterus B. Placenta C. Amnion D. Cervix

Placenta

Your patient is a 21-year-old male who has a gunshot wound to the chest. Which of the following is the highest priority in managing this patient? A. Placing an occlusive dressing over the wound B. Performing a rapid trauma assessment C. Placing a pressure dressing over the wound to control bleeding D. Placing the patient in the shock position

Placing an occlusive dressing over the wound

A suspected musculoskeletal injury of the shoulder is BEST managed by which of the following techniques? A. Placing the arm in a sling and using a triangular bandage to secure it to the body B. Using a longarm air splint C. Placing two long padded board splints on either side of the extremity, extending from the shoulder to the wrist D. Using an upper extremity traction splint

Placing the arm in a sling and using a triangular bandage to secure it to the body

Which of the following is the most serious complication of a fall injury with hip fracture in a​ 75-year-old patient? A. Pneumonia B. Collapsed lungs C. Bruised ribs D. Blood clots

Pneumonia

Which of the following lung conditions is the fourth leading cause of death among the​ elderly? A. Tuberculosis B. Chronic bronchitis C. Pneumothorax D. Pneumonia

Pneumonia

For which of the following reasons are​ newborns, infants, and the elderly more readily affected by the heat than other age​ groups? A. Inadequate salt intake B. Poor ability to regulate body temperature C. Faster metabolism D. Increased body mass

Poor ability to regulate body temperature

Which of the following may provide the EMT with an indication of physical neglect of an elderly​ patient? A. Poor hygiene B. Misuse of a​ person's personal belongings C. Evidence of sexual assault D. Injuries from a fall

Poor hygiene

It is important​ that, as an EMS​ provider, you avoid creating a situation where positional asphyxia could occur. Which of the following is positional​ asphyxia? A. Positional asphyxia is the name for the position someone assumes after she has been hit with a taser. B. Positional asphyxia is the position the person is seated in when you approach him. C. Positional asphyxia is a birth defect that causes psychiatric issues and is initiated because of the way the fetus was positioned in the uterus. D. Positional asphyxia is inadequate breathing or respiratory arrest caused by a body position that restricts breathing.

Positional asphyxia is inadequate breathing or respiratory arrest caused by a body position that restricts breathing.

A traction splint may be used on which of the following musculoskeletal injuries? A. Possible fracture of the cervical spine B. Possible fracture of the humerus C. Suspected multiple fractures of the femur, tibia, and fibula D. Possible fracture of the femur

Possible fracture of the femur

Your patient is a​ 35-year-old woman who was driving a minivan that was struck in the​ driver's side door by another vehicle. You notice that when you apply pressure to her sternum with your knuckles she extends her legs and flexes her arms and wrists. When giving your radio​ report, which of the following terms should you use to describe​ this? A. Posturing B. ​Battle's sign C. ​Tonic-clonic activity D. ​Cushing's reflex

Posturing

Your patient is a 55-year-old male who was found in the parking lot behind a tavern. He states that he was assaulted and robbed by three individuals. He is complaining of being "hit in the face and kicked and punched in his ribs and stomach." Your examination reveals contusions and swelling around both eyes, bleeding from the nose, a laceration of his upper lip, and multiple contusions of the chest, abdomen, and flanks. Which of the following should cause the greatest concern regarding the prehospital care of this patient? A. The presence of any defensive wounds the patient may have sustained B. The swelling around his eyes, which may be reduced by applying a cold pack C. Potential internal injuries D. Getting a description of the assailants

Potential internal injuries

Which of the following age groups typically do not like being separated from their​ parents? A. Adolescents B. Newborns C. School age D. Preschoolers

Preschoolers

Which of the following BEST describes the term crowning​? A. Delivery of the head during a breech​ birth, completing delivery B. Discharge of bloody mucus C. Presenting part of the baby being visible at the vaginal opening D. Complete dilation of the cervix

Presenting part of the baby being visible at the vaginal opening

How does carbon monoxide cause toxic​ effects? A. Prevents red blood cells from releasing carbon dioxide B. Prevents red blood cells from carrying oxygen C. Causes red blood cells to clump together D. Prevents the blood from clotting

Prevents red blood cells from carrying oxygen

Certain drugs are commonly referred as​ "blood thinners" or drugs that inhibit clotting. Which of the following would NOT be considered such a​ drug? A. Prilosec B. Lovenox​ (enoxaparin) C. Aspirin D. Coumadin​ (warfarin)

Prilosec

Using START triage​ guidelines, a patient on the scene of a​ mass-causality incident who has no pulse is deemed a​ ________ patient. A. Priority 1 B. Priority 0 C. Priority 2 D. Priority 3

Priority 0

Using START triage​ guidelines, a patient on the scene of a​ mass-causality incident who is not breathing​ initially, but begins breathing when the airway is​ open, is deemed a​ ________ patient. A. Priority 3 B. Priority 1 C. Priority 2 D. Priority 0

Priority 1

Using START triage​ guidelines, a patient on the scene of a​ mass-causality incident who is alert and has a capillary refill time of less than 2 seconds is deemed a​ ________ patient. A. Priority 1 B. Priority 0 C. Priority 2 D. Priority 3

Priority 3

With START​ triage, when you arrive on the scene you ask all the patients who can walk to move to one area. These patients are categorized​ as: A. Priority 3 walking wounded. B. red tagged. C. Priority 4 walking wounded. D. yellow tagged.

Priority 3 walking wounded.

You arrive on the scene where a school bus has been hit by a train. The bus was in its early stages of picking up students and you have 5 patients. Your closest unit is 30 minutes away. Triage this patient based on this​ scenario: Patient​ #1 is a​ 16-year-old female with the following vital​ signs: respirations​ 38, pulse​ 132, and altered mental status with cool clammy skin. A. Priority​ 2, yellow tag B. Priority​ 3, green tag C. Priority​ 1, red tag D. Priority​ 4, black tag

Priority​ 1, red tag

You arrive on the scene where a school bus has been hit by a train. The bus was in its early stages of picking up students and you only have 5 patients. Your closest unit is 30 minutes away. Triage this patient based on this​ scenario: Patient​ #4 is a​ 13-year-old female with the following vital​ signs: respirations​ 8, pulse​ 124, and nonresponsive with blood from the ears and nose. A. Priority​ 3, green tag B. Priority​ 4, black tag C. Priority​ 1, red tag D. Priority​ 2, yellow tag

Priority​ 1, red tag

You arrive on the scene where a school bus has been hit by a train. The bus was in its early stages of picking up students and you only have 5 patients. Your closest unit is 30 minutes away. Triage this patient based on this​ scenario: Patient​ #2 is an​ 8-year-old male with the following vital​ signs: respirations​ 10, pulse​ 102, ambulatory, and alert mental status with warm dry skin. A. Priority​ 3, green tag B. Priority​ 4, black tag C. Priority​ 1, red tag D. Priority​ 2, yellow tag

Priority​ 3, green tag

You arrive on the scene where a school bus has been hit by a train. The bus was in its early stages of picking up students and you only have 5 patients. Your closest unit is 30 minutes away. Triage this patient based on this​ scenario: Patient​ #5 is a​ 17-year-old male with the following vital​ signs: respirations​ 18, pulse​ 104, alert,​ crying, and wanting his​ mother; he is ambulatory. A. Priority​ 1, red tag B. Priority​ 3, green tag C. Priority​ 4, black tag D. Priority​ 2, yellow tag

Priority​ 3, green tag

You arrive on the scene where a school bus has been hit by a train. The bus was in its early stages of picking up students and you only have 5 patients. Your closest unit is 30 minutes away. Triage this patient based on this​ scenario: Patient​ #3 is a​ 38-year-old female with the following vital​ signs: respirations​ 0, pulse​ 0, and nonresponsive. A. Priority​ 3, green tag B. Priority​ 4, black tag C. Priority​ 1, red tag D. Priority​ 2, yellow tag

Priority​ 4, black tag

Which of the following is the BEST initial intervention for a patient who has inhaled a​ poison? A. Perform abdominal thrusts to clear the​ airway, followed by​ high-concentration oxygen. B. Transport in a position of comfort and provide oxygen for patients with a decreased level of consciousness. C. Provide plenty of fresh air and monitor pulse oximetry readings. D. Provide a patent​ airway, ventilation, and​ high-concentration oxygen.

Provide a patent​ airway, ventilation, and​ high-concentration oxygen.

Your patient is a​ 30-year-old female involved in a motorcycle crash. She was not wearing a helmet and struck her head on the pavement. She is unresponsive and has a blood pressure of​ 152/110 mmHg. Her pulse is 60 beats per minute and respirations are 8 breaths per minute and shallow. Which of the following is an appropriate​ intervention? A. Hyperventilate at a rate of 24 breaths per minute using supplemental oxygen. B. Apply a pressure dressing to her scalp lacerations. C. Elevate the legs. D. Provide cervical spine immobilization.

Provide cervical spine immobilization.

Your patient is a 20yearold college student who has fallen from a thirdlevel balcony onto a wooden deck below. The patient responds to verbal stimuli, is pale in color with moist skin, and has a very obvious deformity with protruding bone ends of his right forearm. Which of the following is the BEST sequence of intervention for this patient? A. Provide manual inline stabilization of the cervical spine along with assessment of breathing, pulse, and the presence of significant hemorrhage; apply highconcentration oxygen; perform a rapid trauma exam; immobilize to a long backboard; transport; and splint the extremity en route if time and resources allow. B. Provide immediate manual inline stabilization of the cervical spine; apply highconcentration oxygen by nonrebreather mask; perform a focused history and assessment; apply the cervical collar; apply a padded board splint, sling, and swathe to the forearm injury; and transport. C. Provide manual inline stabilization of the cervical spine along with assessment of breathing, pulse, and the presence of significant hemorrhage; apply highconcentration oxygen; perform a rapid trauma exam; immobilize to a long backboard; and check with medical control about the need to splint the forearm injury prior to transport. D. Open the airway; assess breathing; check the carotid pulse; splint the forearm injury; immobilize the patient to a long backboard; apply highconcentration oxygen; and transport.

Provide manual inline stabilization of the cervical spine along with assessment of breathing, pulse, and the presence of significant hemorrhage; apply highconcentration oxygen; perform a rapid trauma exam; immobilize to a long backboard; transport; and splint the extremity en route if time and resources allow.

While staffing an emergency services dispatch​ center, an Emergency Medical Dispatcher​ (EMD) receives a call from someone who is very upset and screaming that her friend was just shot. Which of the following would the EMD need to do in this​ situation? A. Provide prearrival medical instructions to the caller. B. Determine the location of the shooter. C. Notify the​ victim's family of the incident. D. Provide the caller with information on local hospitals.

Provide prearrival medical instructions to the caller.

Which of the following is the BEST defense for an EMT when facing allegations of misconduct from a mentally ill​ patient? A. Providing accurate and complete​ documentation, supported by a​ third-party witness B. Making sure you and your partner get your stories straight before talking to anyone C. Having a clean employment record and lack of a criminal background D. Hiring a background investigator to discredit the accuser

Providing accurate and complete​ documentation, supported by a​ third-party witness

Which of the following fractures is most common among elderly women due to​ falls? A. Distal femur B. ​Mid-shaft femur C. Proximal femur D. Pelvis

Proximal femur

Which type of wound has a small opening into the skin, but may be quite deep, and is often caused by instruments such as nails, ice picks, or pencils? a Avulsion b Laceration c Puncture d Incision

Puncture

Which of the following correctly states characteristics of blast injuries? a. Primary injuries occur because of the intense high pressure that hits the patients. b. Secondary injury is the result of projectiles such as debris hitting the patient. c. Tertiary injuries can include not just soft-tissue injuries but also fractures, avulsions, and amputations. d. Quaternary injuries involve the lower quarter of the body, which is almost always injured during a blast.

Quaternary injuries involve the lower quarter of the body, which is almost always injured during a blast.

Your patient is a​ 38-year-old male who has taken an overdose of several different medications and has consumed some household cleaning agents as well. On your​ arrival, the patient only responds to painful​ stimuli, and has a heart rate of 90 beats per minute with a respiratory rate of 12 breaths per minute. Medical control orders you to give the patient two to three glasses of milk to drink to dilute the contents of the stomach. Which of the following actions should you​ do? A. Question the order and explain why. B. Assist the patient in drinking as much of the two to three glasses of milk as he can. C. Assist the patient in drinking only one glass of milk to reduce the risk of aspiration. D. Ask medical control if you can substitute water for the milk since water will not damage the lungs if aspirated.

Question the order and explain why.

You are dispatched to a local​ fast-food restaurant for a​ "nature unknown" call. You arrive​ on-scene and find a​ 47-year-old male in front of the counter repeatedly singing​ "Happy Birthday" to himself. Police are​ on-scene and the scene is safe. Which of the following would be considered appropriate​ care? A. Quickly approach the patient and take charge. Tell him he has to come with you to be evaluated. B. Have the police take​ charge, restrain the​ patient, and have him brought into your ambulance. C. Quietly and carefully evaluate the situation and keep your emotions under control. Be as unhurried as you can. D. Have your partner join you in approaching the patient and explain to him that he has to stop singing or you will have him arrested.

Quietly and carefully evaluate the situation and keep your emotions under control. Be as unhurried as you can.

Which of the following is NOT a way in which poisons can access the​ body? A. Radiation B. Absorption C. Inhalation D. Injection

Radiation

You are caring for a multisystem trauma patient. Which of the following EMT interventions is most​ important? A. Splint all suspected musculoskeletal injuries B. Complete a complete secondary physical exam C. Rapid transport D. Trend vital signs

Rapid transport

Your patient is a 34yearold male complaining of pain "in his right side." He is pale and diaphoretic with a heart rate of 90 beats per minute, a respiratory rate of 28 breaths per minute, and a blood pressure of 132/80 mmHg. The patient is very agitated and anxious. Which approach is most appropriate? A. Insert an oropharyngeal airway. B. Try to determine the cause of his pain. C. Tell the patient that you cannot transport him unless he calms down and lies still. D. Reassure him that you will make him as comfortable as possible and get him to the hospital for additional care.

Reassure him that you will make him as comfortable as possible and get him to the hospital for additional care.

Which of the following actions is the MOST appropriate for the EMT to take when managing a patient with a behavioral​ emergency? A. Establish control of the situation by a show of force. B. Remain calm and reassure the patient. C. Find out if the​ patient's insurance covers psychiatric treatment. D. Quickly perform a​ head-to-toe exam and transport without delay.

Remain calm and reassure the patient.

Which one of the listed items below is NOT a function of the​ blood? A. Delivery of oxygen to the cells B. Removal and delivery of other waste products to organs that provide filtration and​ removal, such as the kidneys and liver C. Control of bleeding by clotting D. Removal of carbon monoxide from the cells

Removal of carbon monoxide from the cells

Which of the following is acceptable in the management of a patient stung by a honey​ bee? A. Leave stingers in place. B. Elevate the affected site above the level of the heart to reduce swelling. C. Soak the affected area in warm water. D. Remove jewelry from any affected limbs.

Remove jewelry from any affected limbs.

42. You are dispatched to an auto repair shop for an "accident." You arrive and are told by the owner that one of the workers apparently got his hand in the way while using a high-pressure grease gun and injected the grease into his hand. You find the patient seated in a chair with a coworker applying ice to the injured hand. Your examination reveals a minor round laceration in the hand. The patient is complaining of pain in the area but wants to let the wound take care of itself and go back to work. What should you do? a. The patient is correct. This type of wound will heal on its own and he can go back to work. b. Remove the ice from the wound, then elevate and splint the limb. Transport the patient high priority. c. Continue to apply cold, elevate and splint the limb, and transport the patient high priority. d. Tell the patient that if it does not get better within the next few days he should consult his own doctor

Remove the ice from the wound, then elevate and splint the limb. Transport the patient high priority.

You are on the scene of a possible overdose. You find a​ 30-year-old man pacing about his living room. There is evidence of illicit drug use and the apartment is in disarray. The patient seems agitated and nervous. Attempts at calming the patient should include which of the​ following? A. Repeat part of what the patient is saying to show that you are listening to him. B. Sit close to him with your arm around his shoulders to show that you are truly concerned. C. Stand still with your arms crossed to instill a sense of control and authority. D. Speak quickly to give the patient all of the important information without delay.

Repeat part of what the patient is saying to show that you are listening to him.

Your patient is a​ 28-year-old male who cut his thigh with a chain saw. Bleeding is significant and difficult to control. Which of the following is NOT part of the proper management of this​ patient? A. Use of a tourniquet B. Replacement of fluid level by giving the patient adequate amounts of water C. Using direct pressure to control the bleeding D. Administering oxygen

Replacement of fluid level by giving the patient adequate amounts of water

Which of the following is NOT the role of the first arriving EMT at the scene of a possible hazardous materials​ emergency? A. Rescuing any victims still in the area of the spill B. Recognizing a hazardous materials emergency C. Establishing a safe zone D. Requesting special resources to respond

Rescuing any victims still in the area of the spill

Which of the following signs is LEAST likely to indicate aspirin poisoning in a​ child? A. Vomiting B. Restlessness followed by drowsiness C. Sweating D. Hyperventilation or hypoventilation

Restlessness followed by drowsiness

Which abdominal quadrant contains the appendix? A. Right lower B. Right upper C. Left upper D. Left lower

Right lower

Which of the following pieces of equipment is acceptable for use in the prehospital stabilization of suspected cervical spine​ injuries? A. 2′′ ×4′′ wood cribbing B. Soft cervical collars C. Rigid cervical collars D. ​Five-pound sand bags

Rigid cervical collars

You have assisted in the delivery of a​ full-term infant by suctioning the mouth and nose as the head was delivered and again following complete delivery. The infant is not yet breathing. Which of the following is the correct sequence of​ interventions? A. Rub the​ infant's back, tap the bottom of his​ foot, begin​ bag-valve-mask ventilations, and check the pulse. B. Transport without further intervention. C. Begin​ bag-valve-mask ventilations, suction the airway with a rigid tonsil​ tip, and begin CPR. D. Perform a series of back blows and chest thrusts interposed with​ mouth-to-mouth ventilation.

Rub the​ infant's back, tap the bottom of his​ foot, begin​ bag-valve-mask ventilations, and check the pulse.

What source will provide first aid information about hazardous materials for employees at a work​ site? A. Shipping manifest B. Placard C. Bill of lading D. Safety Data Sheets

Safety Data Sheets

Heat cramps occur due to loss of which of the following​ substances? A. Salt B. Magnesium C. Water D. Water and potassium

Salt

You are dispatched to a psychiatric emergency for a​ 68-year-old male. Dispatch provides no other information​ and, when questioned by​ you, they do not have any more information. Although all steps are important in dealing with this​ situation, which step is the MOST​ important? A. Vital signs and SAMPLE history B. Primary assessment C. Secondary assessment D. Scene​ size-up

Scene​ size-up

Which of the following is recommended when caring for an amputated part? a Rinse away debris with saline solution and place the part in a container of ice. b Wrap the part in aluminum foil to preserve body temperature. c Seal the part in a plastic bag and place it in a pan of water cooled by an ice pack. d Always transport the amputated part with the patient.

Seal the part in a plastic bag and place it in a pan of water cooled by an ice pack.

During which stage of labor is the baby​ born? A. Third B. First C. Primary D. Second

Second

Which of the following is an accurate definition of a flail chest? A. Section of the chest wall that is unstable, leading to breathing problems B. Fracture of one rib in two or more consecutive places C. Fracture of at least four ribs in two or more places D. Lung that has been punctured by a fractured rib, resulting in a buildup of air

Section of the chest wall that is unstable, leading to breathing problems

Which of the following may help in minimizing the stressful effects of the disruption in circadian rhythms seen in EMS​ multiple-casualty incidents​ work? A. Keep your bedroom light on and warm. B. Eat a heavy meal just before bedtime. C. Seek help from a trained professional. D. Sleep as much as possible on days off.

Seek help from a trained professional.

Which of the following is the correct order of operations when transferring a stable patient from his or her house to the​ ambulance? A. Select the proper​ patient-carrying device, move the patient to the​ ambulance, package the patient for​ transport, and load the patient into the ambulance. B. Select the proper​ patient-carrying device, package the patient for​ transport, move the patient to the​ ambulance, and load the patient into the ambulance. C. Package the patient for​ transport, select the proper​ patient-carrying device, move the patient to the​ ambulance, and load the patient into the ambulance. D. Package the patient for​ transport, move the patient to the​ ambulance, select the proper​ patient-carrying device, and load the patient into the ambulance.

Select the proper​ patient-carrying device, package the patient for​ transport, move the patient to the​ ambulance, and load the patient into the ambulance.

You are called to a nursing home for an 85yearold patient complaining of abdominal pain. The patient has a history of dementia and cannot describe the pain to you. The nurse states the patient has been vomiting dark coffee ground emesis for about an hour. His blood pressure is 90/40, pulse 100, and respiratory rate of 24. In what position should you transport the patient? A. Supine with knees bent B. Trendelenburg C. SemiFowler's D. Supine

SemiFowler's

Which of the following is controlled by the left side of the​ brain? A. Sensation in the left arm B. Movement of the left arm C. Sensation of the right leg D. Movement of both lower extremities

Sensation of the right leg

Messages from the body to the brain are carried by which of the following types of​ nerves? A. Motor B. Cranial C. Skeletal D. Sensory

Sensory

Which of the following measures can the EMT take to create a safer extrication​ scene? A. Setting up road flares every 10 feet for a safe distance to divert traffic away from the collision B. Setting up road flares around hazards such as spilled fuel C. Dropping road flares from the ambulance as you are approaching the scene D. Both B and C

Setting up road flares every 10 feet for a safe distance to divert traffic away from the collision

Which of the following is NOT a contraindication for the use of a traction splint? A. Injury to the lower third of the leg that would interfere with the ankle hitch B. Avulsion or partial amputation where traction could separate the extremity C. Severe swelling and redness at a midshaft femur D. Pelvis, hip, or knee injury

Severe swelling and redness at a midshaft femur

Which of the following is NOT a legitimate assessment technique for a pediatric​ patient? A. Tap the child to assess pain response. B. Shake the child to assess alertness. C. Pinch the child to assess pain response. D. Shout to elicit a response to a verbal stimulus

Shake the child to assess alertness.

Which of the following is true concerning trauma in the pregnant​ woman? A. She may lose up to​ 35% of her blood volume before exhibiting signs of shock. B. The​ mother's body will preferentially protect the life of the fetus over that of the mother. C. The increase in blood volume during pregnancy makes shock an unlikely cause of death. D. She may lose up to​ 15% of her blood volume before exhibiting signs of shock.

She may lose up to​ 35% of her blood volume before exhibiting signs of shock.

In which of the following ways does the body produce heat in response to being​ cold? A. Burning fewer calories B. Excreting more urine C. Shivering D. Increasing the respiratory rate

Shivering

Which of the following signs would you NOT expect to see in a patient suffering from severe​ hypothermia? A. Skin cool to touch B. Shivering C. Drowsiness D. Numbness

Shivering

Which of the following pathologies make patients at high risk for acute renal​ failure? A. Hypertension B. Uncontrolled diabetes C. Shock D. Polycystic kidney disease

Shock

You are evaluating an​ 18-month-old child who presents with a history of​ fever, diarrhea, and vomiting. Her radial pulse is weak in comparison with her carotid pulse. Her respirations are 30 with adequate depth. Her skin is​ pale, cool, and​ clammy, and she cries without tears. What is the primary problem for which you will treat this​ patient? A. Mild dehydration B. Hypoglycemia C. Respiratory failure D. Shock

Shock

An elderly patient who is experiencing a myocardial infarction is more likely to complain of which of the following symptoms than a younger patient​ would? A. Shortness of breath with chest pain B. No chest pain or shortness of breath C. Shortness of breath without chest pain D. A rash caused by shingles

Shortness of breath without chest pain

Which one of the following is NOT a key decision for the EMT when faced with a multisystem or multiple trauma​ patient? A. Should I transport to a trauma​ center? B. Should I allow police to interview the patient​ on-scene? C. Do I need to minimize​ on-scene time? D. Is the patient seriously​ injured?

Should I allow police to interview the patient​ on-scene?

Which of the following is a surgically implanted drainage device that runs from the brain to the​ abdomen? A. Shunt B. Tracheostomy tube C. Central intravenous line D. Gastrostomy tube

Shunt

Which of the following statements regarding elder abuse is​ correct? A. Signs of elder abuse may be difficult to detect. B. Abused elderly patients will always have physical signs of abuse. C. Signs of elder abuse are usually obvious. D. EMTs should ask elderly patients to point out their abuser.

Signs of elder abuse may be difficult to detect.

Which of the following must be kept in mind when considering the severity of external​ bleeding? A. A younger person can tolerate more blood loss than an adult. B. Signs of shock do not appear until a large amount of blood has been lost. C. The amount of blood loss is easily estimated by the amount of blood visible on the ground. D. All of the above

Signs of shock do not appear until a large amount of blood has been lost.

Which of the following is NOT a phase of the extrication​ process? A. Terminating the rescue B. Gaining access to the patient C. Disentanglement D. Simplification

Simplification

Why is it important to keep patients at rest when they are​ hypothermic? A. Since the blood is coldest in the​ extremities, exercise or unnecessary movement could quickly circulate the cold blood and lower the core body temperature. B. Since the blood is coldest in the​ extremities, exercise or unnecessary movement could quickly circulate the cold blood and drop the blood pressure. C. Since the blood is warmest in the​ extremities, exercise or unnecessary movement could quickly circulate the warm blood and raises the core body temperature. D. None of the above

Since the blood is coldest in the​ extremities, exercise or unnecessary movement could quickly circulate the cold blood and lower the core body temperature.

In which of the following situations would a person lose heat by​ conduction? A. Sitting on cold metal bleachers at a football game B. Breathing C. Wearing wet clothing in windy weather D. Going outside without a coat during a cold but calm day

Sitting on cold metal bleachers at a football game

Which of the following is NOT a form of elder abuse or​ neglect? A. Financial B. Social C. Physical D. Psychologica

Social

Which of the following contributes MOST significantly to the occurrence of ambulance​ crashes? A. Size of the vehicle B. Weather conditions C. Time of day D. Speed of the vehicle

Speed of the vehicle

The bony bumps you feel along the center of a​ person's back are known as which of the​ following? A. Vertebrae B. Spinous process C. Foramen magnum D. Transverse processes

Spinous process

Which of the following organs may seriously be damaged in sickle cell​ anemia, and often subsequently leads to severe​ infections? A. Liver B. Spleen C. Lungs D. Kidneys

Spleen

Limiting time spent at a scene can be especially important if the mechanism of injury suggests that the patient could go into shock. In order to keep the time at the scene to a​ minimum, which of the following assessments or treatments should not be performed on the​ scene? A. Rapid trauma exam B. Immobilization C. ABCs with spinal precautions D. Splinting swollen extremities

Splinting swollen extremities

What is the term for when the fetus and placenta deliver before the 28th week of​ pregnancy? A. Induced abortion B. Stillbirth C. Eclampsia D. Spontaneous abortion

Spontaneous abortion

Which of the following is a characteristic of arterial​ bleeding? A. Steady flow B. Spurting under pressure C. Dark red color D. Both B and C

Spurting under pressure

43. You are dispatched to the local elementary school for an injured student. Upon arrival you find that two 7-year-olds got into a fight and one of them jabbed a pencil in the other's cheek. The pencil is still sticking out of the child's cheek. When you examine the patient, you cannot see the end of the pencil that went through the cheek, as it appears to be stuck in the palate. There is not significant bleeding, and the child is not having any difficulty breathing. The child is very upset and wants you to pull the pencil out. What should you do? a. Stabilize the object, but do not try to remove it. b. Remove the object and put a bandage on the outside of the cheek. c. Call the parents and see what they want you to do. d. Let the child carefully pull the object out.

Stabilize the object, but do not try to remove it.

Your patient is a​ 37-year-old male sitting on the kitchen floor. His wife is attempting to talk with​ him, but he does not look at her or answer. He is fidgeting and has apparently thrown some dishes against the wall. What should be your first​ consideration? A. Ask the wife to step into the other room so you can speak with the patient alone. B. Quickly check the​ patient's carotid pulse. C. Ask the wife to step outside with you so you can get information from her. D. Stay a safe distance away.

Stay a safe distance away.

Which of the following is a desirable characteristic of dressings used in the prehospital management of most open wounds? A Absorbent B Occlusive C Adherent D Sterile

Sterile

Which of the following is a desirable characteristic of dressings used in the prehospital management of most open wounds? A. Occlusive B. Adherent C. Nonabsorbent D. Sterile

Sterile

Which of the following bones is found in the chest? A. Metacarpals B. Ilium C. Sternum D. Tarsals

Sternum

Which of the following is NOT a role of dialysis in a renal failure​ patient? A. Stimulate blood cell production B. Remove fluid from the body C. Filter and detoxify the blood D. Remove waste from the body

Stimulate blood cell production

You are inserting an OPA into a small​ child's airway. What is the preferred technique that you should​ use? A. 180 degree reverse twist B. Straight insertion technique with a tongue depressor C. 360 degree reverse twist D. The same technique as with adults

Straight insertion technique with a tongue depressor

Which of the following is an injury to the musculature of an extremity? A. Sprain B. Strain C. Fracture D. Luxation

Strain

When a patient or bystander at the scene of an emergency displays​ fear, anger, or​ grief, this is best described as which of the​ following? A. Catharsis B. ​Post-traumatic stress disorder C. Neurosis D. Stress reaction

Stress reaction

Which of the following is NOT a cause of abdominal pain? A. Diabetes B. Food poisoning C. Heart attack D. Stroke

Stroke

Which of the following BEST explains the reason for minimizing​ scene-time for the trauma patient with significant hemorrhage or the potential for significant​ hemorrhage? A. The clock for the​ "golden hour" of trauma begins at the time of your arrival. B. There is nothing the EMT can do for a patient in shock. C. Studies have indicated that trauma patients who receive surgery within 1 hour of injury have better chances of survival. D. It gives the EMT less opportunity to make mistakes in the​ patient's care.

Studies have indicated that trauma patients who receive surgery within 1 hour of injury have better chances of survival.

Which of the following layers of the skin is the most important in insulating the body against heat loss? A. Parietal layer B. Subcutaneous layer C. Epidermis D. Subdural layer

Subcutaneous layer

When blood accumulates between the brain and the dura​ mater, what is the​ result? A. Epidural hematoma B. Epidural contusion C. Subdural hematoma D. Subdural contusion

Subdural hematoma

Which of the following is an injury that commonly occurs in alcoholic patients with even minor falls and blows to the​ head? A. Intracerebral hematoma B. Subdural hematoma C. Skull fracture D. Concussion

Subdural hematoma

What is the correct terminology for a wound in which a vacuum has been created within the chest, drawing air into the thorax with each breath? A. Closed tension pneumothorax B. Sucking chest wound C. Gurgling chest wound D. Open chest wound

Sucking chest wound

You respond to a boat dock for a diving injury. You find a​ 24-year-old male patient unresponsive with frothy blood in the mouth and lung sounds absent on the right side. The patient is breathing 28 times a minute. The​ patient's friends state they were diving when he unexpectedly came out of the water complaining of chest pains and then collapsed. What is your first​ action? A. Suction the airway. B. Place the patient on​ high-concentration oxygen by​ bag-valve mask. C. Insert an oropharyngeal airway. D. Place the patient on​ high-concentration oxygen by nonrebreather mask.

Suction the airway.

You respond to a professional volleyball tournament for a potential heatstroke. Upon arrival you find a​ 28-year-old male patient seated inside the​ air-conditioned first aid trailer. He is alert and in obvious distress. He is complaining of severe cramping of his arms and legs. His vital signs are stable. What is the most likely cause of the​ cramping? A. Sweating too much B. Cooling off too quickly C. Drinking too many sports drinks D. Heat exhaustion

Sweating too much

41. You are dispatched to the local high school for a "person struck with a baseball." You arrive on the scene and find a 16-year-old male sitting on the bench. Apparently he was the pitcher and was struck in the abdominal area by a line drive ball that was hit very hard. He states that nothing is hurting except he has some mild pain in the area where he was struck. He is upset that the ambulance was called and wants to go back into the game and continue pitching. Your exam reveals nothing remarkable except mild pain when you palate the injured area. Vital signs are normal. What is the next step? a. Have the coach apply cold to the area and let the pitcher back in the game if he does not get worse in the next 15 minutes. b. Have the patient walk to the ambulance and take him to the ED. No other care needs to be provided, as there is really no significant sign of any injury. c. Take appropriate Standard Precautions, apply high-concentration oxygen by nonrebreather mask, and transport the patient ASAP, carefully monitoring the patient during transport. d. Tell the coach the patient is okay and he can go back into the ball game.

Take appropriate Standard Precautions, apply high-concentration oxygen by nonrebreather mask, and transport the patient ASAP, carefully monitoring the patient during transport.

You encounter an accident on a busy intercity street while on duty. Calling into​ dispatch, you make note that the occupants of both vehicles are​ outside, and you request additional units to proceed​ non-emergently. You approach an elderly male who is rubbing his back and left shoulder. During secondary assessment of past medical​ history, you make note of several important​ details: the patient is on high blood pressure​ medications, and has had a heart attack in the past. He is complaining of midline thoracic pain on palpation of his spine and left shoulder​ pain, which may have been from the seat​ belt, but is refusing care and transport. The patient did not lose consciousness. Based on this​ information, which transport decision would be most appropriate for this​ patient? A. Call for ALS intercept. B. Allow the patient to sign a refusal. C. Take the patient to a trauma center. D. Take the patient to a local community hospital.

Take the patient to a local community hospital.

You are on the scene where a​ 23-year-old female has attempted suicide by cutting her forearms with a razor. She is sitting in the front yard and has lost a large amount of blood. She is conscious and holding a small razor​ blade, and tells you to get away from her. Which of the following should NOT be​ done? A. Keep bystanders away from the patient. B. Use Body Substance Isolation​ (BSI). C. Call law enforcement for assistance. D. Take the razor blade away from her by force.

Take the razor blade away from her by force.

When grossly decontaminating a patient who is not wearing personal protective equipment in a hazardous materials​ incident, which of the following techniques should be​ used? A. Scrubbing with a​ long-handled brush B. Sponging with a dilute solution of water and white vinegar C. Taking a​ low-pressure decontamination shower D. Using a high pressure water jet system

Taking a​ low-pressure decontamination shower

Which of the following describes the proper application of an occlusive dressing for an open chest wound? A. Tape the occlusive dressing on two sides only to create dual flaps for relief pressure build-up. B. Tape the dressing securely on three sides. C. Use a porous material such as a 4double prime′′ by 4double prime′′ gauze pad. D. Trim the dressing so that it is the exact size of the wound.

Tape the dressing securely on three sides.

You respond to an abdominal pain call. Your partner suspects that the patient is having an abdominal aortic aneurysm (AAA). What type of pain is the patient most likely experiencing? A. Tearing B. Referred C. Visceral D. Parietal

Tearing

Which one of the following statements is NOT true? A. Muscles are the tissues or fibers that cause movement of body parts or organs. B. Cartilage is connective tissue that covers the outside of the bone end and acts as a surface for articulation. C. Tendons are bands of connective tissue that bind the ligaments to muscles. D. Ligaments are connective tissues that connect bone to bone.

Tendons are bands of connective tissue that bind the ligaments to muscles.

Which one of the following definitions is NOT true? A. Another name for manual traction is tension. B. A sprain is the stretching and tearing of ligaments. C. Tendons connect muscles to ligaments. D. Joints are places where bones articulate.

Tendons connect muscles to ligaments.

A patient with jugular vein distention (JVD) is most likely suffering from which of the following injuries? A. Pneumothorax B. Tension pneumothorax C. Hemopneumothorax D. Hemothorax

Tension pneumothorax

Which of the following patients is at greatest risk of respiratory failure and should be carefully monitored for ventilatory status throughout treatment and transport? A. Flail chest B. Abdominal evisceration C. Hemopneumothorax D. Tension pneumothorax

Tension pneumothorax

Which of the following is true in regards to touching a patient who is being shocked with an automatic implanted cardiac defibrillator​ (AICD)? A. The EMT cannot feel the shock of an AICD when delivered to the patient. B. It depends on the AICD model as to whether or not the EMT can feel the shock. C. The EMT will always feel the​ shock, but it is not very painful. D. The EMT will always feel the​ shock, and it is very dangerous to the EMT as well.

The EMT cannot feel the shock of an AICD when delivered to the patient.

Which of the following should the EMT keep in mind when evaluating a patient for a possible behavioral​ emergency? A. The EMT must determine the cause of the behavior before making a transport decision B. The behavior should be evaluated outside of the situation at hand C. The behavior should be evaluated based on the situation at hand D. Uncommon emotional reactions should always be considered a behavioral emergency

The behavior should be evaluated based on the situation at hand

Which of the following statements regarding the bones of infants and children is​ TRUE? A. The bones are harder and more brittle than in adults. B. The bones are more flexible than in adults. C. The bones are more easily fractured than in adults. D. Traumatic forces are more likely to injure bones than organs.

The bones are more flexible than in adults.

Which of the following is NOT true concerning abdominal pain in geriatric patients? A. Medications may mask signs of shock associated with an abdominal complaint. B. The elderly person may not be able to give a specific description of the pain. C. The causes of abdominal pain in the elderly are rarely serious. D. The elderly have a decreased ability to perceive pain.

The causes of abdominal pain in the elderly are rarely serious.

Regarding drowning in​ adults, which of the following statements is true​? A. Colder water improves survival chances in saltwater drowning but not in freshwater drowning. B. Water temperature makes no difference in the chances of survival in either saltwater or freshwater drowning. C. The colder the​ water, the better the chances of survival in either saltwater or freshwater drowning. D. The warmer the​ water, the better the chances of survival in saltwater drowning.

The colder the​ water, the better the chances of survival in either saltwater or freshwater drowning.

Which of the following is true concerning a stillborn​ baby? A. Resuscitation must always be attempted. B. Infants born in cardiopulmonary arrest should not be resuscitated. C. The death may occur weeks before delivery. D. The parents should never be allowed to see the​ baby, especially if it has begun to deteriorate.

The death may occur weeks before delivery.

Which of the following statements regarding pediatric burn patients is​ TRUE? A. They should always be transported immediately to a burn center. B. They cannot be assessed in the field. C. Their burns should be left uncovered. D. The extent of their burns may be estimated with The Rule of Nines.

The extent of their burns may be estimated with The Rule of Nines.

Which of the following is of concern in a patient who received burns to his hand when he grabbed a live electrical wire? A. The extent of tissue damage may be much greater than it appears on the surface. B. The burning will continue for hours, perhaps days. C. The patient remains an electrocution hazard to rescuers for several minutes after being removed from the source of electricity. D. Toxic gases were inhaled.

The extent of tissue damage may be much greater than it appears on the surface.

Your patient is a​ 26-year-old female with a history of ectopic pregnancies. She has called 911 for extreme abdominal pain and she is concerned that she has another ectopic pregnancy. Her vital signs are pulse​ 118, respirations​ 20, and blood pressure​ 100/76. Which of the following is true regarding ectopic​ pregnancies? A. The fertilized egg usually implants in the wall of the uterus. B. It usually occurs during the second trimester. C. It may cause painless bright red bleeding. D. The fallopian tubes cannot stretch to accommodate a fetus.

The fallopian tubes cannot stretch to accommodate a fetus.

You have just delivered a​ full-term baby girl and she is doing well. You have dried her off and wrapped her in a warm blanket. You are preparing to cut the umbilical cord. Which of the following is normally true regarding cutting the umbilical​ cord? A. You should hold the baby above the level of the mother when cutting the cord. B. The infant must be breathing on his or her own before you cut the cord. C. The cord should be cut immediately following delivery. D. You must try to cut the cord before it stops pulsating.

The infant must be breathing on his or her own before you cut the cord.

Which of the following is true concerning prehospital delivery of​ twins? A. This is a true emergency and cannot be managed outside the hospital setting without additional help. B. There are always two placentas. C. The umbilical cord of the first infant must not be cut until the second infant is born. D. The infants will probably be smaller than a single infant and need attention in keeping them warm.

The infants will probably be smaller than a single infant and need attention in keeping them warm.

Regarding ambulance warning​ devices, which of the following statements is NOT​ true? A. The large lights on the outermost corners of the ambulance box should blink alternately. B. ​Four-way flashers should not be used as emergency lights. C. The use of strobes is controversial. D. In most states it is illegal to drive at night with only one headlight.

The large lights on the outermost corners of the ambulance box should blink alternately.

46. You are examining a 48-year-old patient who has been burned. You decide to use the rule of palm to measure the extent of the burn. What does this mean? a. The palm of your hand represents 1 percent of the body's surface area. b. The palm of the patient's hand equals about 1 percent of the body's surface area. c. The palm of the patient's hand equals 9 percent of the body's surface area. d. You can only use the rule of palm for children, so you would use the rule of nines instead

The palm of the patient's hand equals about 1 percent of the body's surface area.

Who is the best source of patient history information for an older adolescent​ patient? A. The​ patient's friends B. The school nurse C. The parents D. The patient

The patient

You are called to a nursing home for an 85yearold patient complaining of abdominal pain. The patient has a history of dementia and cannot describe the pain to you. The nurse states the patient has been vomiting dark coffee ground emesis for about an hour. His blood pressure is 90/40, pulse 100, and respiratory rate of 24. Why should the EMT use an oxygen mask with caution? A. The patient may hyperventilate. B. The oxygen will cause the patient to start vomiting again. C. The patient may go unresponsive. D. The patient cannot clear any additional emesis.

The patient cannot clear any additional emesis.

Which of the following situations generally allows the EMT to transport a patient with a behavioral emergency against his or her​ wishes? A. The​ patient's personal physician gives you permission. B. The patient is a threat to himself or others. C. A family member gives consent and is willing to accompany the patient. D. The​ patient's insurance company agrees to​ pay, even though the patient has not given consent.

The patient is a threat to himself or others.

Which of the following situations may warrant the use of helicopter transport from the scene to a medical​ facility? A. The patient is in cardiac arrest. B. The distance to the medical facility is 15 minutes away. C. The patient is located in a remote area. D. The patient may go into shock shortly.

The patient is located in a remote area.

You are treating a​ 61-year-old who is a chronic alcohol abuser. He is complaining of​ "snakes slithering around his​ ankles," and he tells you that he had only two drinks tonight. You notice that he is​ sweating, trembling, and anxious. Before you can complete your​ assessment, he begins to have a seizure. Which of the following is the most likely cause of this​ situation? A. The patient drank mouthwash instead of beverage alcohol. B. The patient could not find an alcoholic beverage and drank antifreeze instead. C. The patient is suffering from acute alcohol poisoning. D. The patient is suffering from alcohol withdrawal.

The patient is suffering from alcohol withdrawal.

You're an off-duty EMT who encounters a patient sitting behind the wheel of a vehicle that ran off the road along an isolated county road. It appears the patient was unrestrained, or not wearing a seat belt, and struck the steering wheel with his chest. On assessment, you notice a paradoxical motion to the patient's chest on inspiration and expiration. When you radio for dispatch of an ambulance, which of the following pieces of information would you be sure to include? A. The patient is showing signs of an open chest injury. B. The patient may have a flail chest. C. The patient may have an abdominal evisceration. D. The patient is showing signs of abdominal bleeding

The patient may have a flail chest.

You have responded to a local pool for a drowning patient. Lifeguards have pulled the patient out of the pool prior to your arrival. You find a​ 22-year-old male patient unresponsive with agonal respirations and a weak carotid pulse. Bystanders state the patient was not using the diving board and was found in the shallow end. Why are cervical spine precautions​ necessary? A. The patient may have hit his head on the side wall while swimming laps. B. The patient may have suffered a cervical spine injury when pulled out of the water. C. The patient may have been using the diving board after all and injured his cervical spine. D. The patient may have suffered a spinal injury by jumping in shallow water.

The patient may have suffered a spinal injury by jumping in shallow water.

Which of the following statements concerning a concussion is true​? A. It is a bruising of the brain tissue. B. It is accompanied by recurrent episodes of unconsciousness. C. The patient may not have any symptoms of the injury. D. All of the above

The patient may not have any symptoms of the injury.

You are called to a nursing home for an 85yearold patient complaining of abdominal pain. The patient has a history of dementia and cannot describe the pain to you. The nurse states the patient has been vomiting dark coffee ground emesis for about an hour. His blood pressure is 90/40, pulse 100, and respiratory rate of 24. Aside from the airway, what is your greatest concern? A. The patient will go into hypovolemic shock. B. The patient will have a stroke. C. The patient will become combative. D. The patient will have a myocardial infarction

The patient will go into hypovolemic shock.

Which of the following BEST describes placenta​ previa? A. The placenta prematurely separates from the uterine wall. B. The placenta is implanted over the opening of the cervix. C. The umbilical cord is the presenting part. D. The pregnancy is lost before the 20th week of gestation.

The placenta is implanted over the opening of the cervix.

Your patient is a 55-year-old male who was found in the parking lot behind a tavern. He states that he was assaulted and robbed by three individuals. He is complaining of being "hit in the face and kicked and punched in his ribs and stomach." Your examination reveals contusions and swelling around both eyes, bleeding from the nose, a laceration of his upper lip, and multiple contusions of the chest, abdomen, and flanks. Which of the following should cause the greatest concern regarding the prehospital care of this patient? A. Reducing the swelling around his eyes by applying a cold pack B. The possibility of a pneumothorax C. Getting a description of the assailants D. The presence of any defensive wounds the patient may have sustained

The possibility of a pneumothorax

55. Which of the following is a description of the rule of nines for an adult? a. The rules of nines assigns 9 percent to the head and neck, each upper extremity, the chest, the abdomen, the upper back, the lower back and buttocks, the front of each lower extremity, and the back of each lower extremity, then 1 percent to the genital region. b. The rules of nines assigns 9 percent to the head and neck, as well as each upper extremity; 10 percent to the chest, the abdomen, and the upper back; 8 percent to the lower back and buttocks, the front of each lower extremity, and the back of each lower extremity, then 1 percent to the genital region. c. The rules of nines assigns 9 percent to the head and neck, each upper extremity, the chest, the abdomen, the upper back, and the lower back and buttocks; 8 percent to the front of each lower extremity; and 10 percent for the back of each lower extremity, then 1 percent to the genital region. d. The rules of nines assigns 9 percent to the head and neck; 6 percent to each upper extremity; 12 percent to the chest and abdomen; 9 percent to the upper back, the lower back and buttocks, the front of each lower extremity, and the back of each lower extremity, then 1 percent to the genital region.

The rules of nines assigns 9 percent to the head and neck, each upper extremity, the chest, the abdomen, the upper back, the lower back and buttocks, the front of each lower extremity, and the back of each lower extremity, then 1 percent to the genital region.

Which of the following is a true statement regarding the skin's status in the case of a closed chest injury? A. The skin is not penetrated. B. Internal contusions and lacerations cannot occur. C. The skin may be penetrated and occluded. D. The skin is penetrated.

The skin is not penetrated.

Which of the following BEST describes a partial thickness burn? a The skin is red, but dry and painful. b The skin is charred or blackened and lacks sensation. c The skin is white and dry with no sensation of pain. d The skin is red and moist with blister formation.

The skin is red and moist with blister formation.

You are getting ready to transport an unresponsive​ 25-year-old female patient. She was hit by a vehicle while crossing the street. She is 26 weeks pregnant. You are 10 minutes away from the nearest​ facility, 15 minutes away from a Level I Trauma​ Center, and 15 minutes away from a hospital that specializes in​ high-risk obstetrics. You should transport the patient to which​ hospital? A. The trauma​ center: the fetus will need specialized neonatology surgeons B. The​ high-risk obstetric​ hospital: the fetus will need specialized neonatology surgeons C. The nearest​ facility: she is unresponsive and unstable D. The trauma​ center: the patient will need specialized trauma surgeons

The trauma​ center: the patient will need specialized trauma surgeons

Which of the following statements about a pediatric patient with a tracheostomy tube on a home ventilator is​ TRUE? A. There is a high risk of infection. B. EMTs are not permitted to care for such patients. C. Infection is rare in such patients. D. Parents of such patients are typically poor historians.

There is a high risk of infection.

Which of the following observations may the EMT use to rule out a spinal injury in a trauma​ patient? A. Patient denies pain in his spine. B. There is a lack of mechanism of injury. C. Patient is able to walk at the scene. D. There is a lack of numbness and paralysis of the extremities.

There is a lack of mechanism of injury.

Which of the following is a common sign of a​ pre-delivery emergency? A. The placenta fails to deliver within 20 minutes of the​ infant's birth. B. The amniotic sac ruptures. C. Contractions become more intense and closer together. D. There is profuse vaginal bleeding.

There is profuse vaginal bleeding.

Which of the following statements concerning people who have made prior suicide attempts is true​? A. They are usually just making a cry for help but do not want to die. B. They are at an increased risk for a subsequent successful suicide. C. They should be allowed to sign a release if their injuries are not serious. D. They do not require psychological counseling like a person who is truly depressed.

They are at an increased risk for a subsequent successful suicide.

What is the purpose of the red blood​ cells? A. They are responsible for the removal of oxygen from the cells. B. They are critical in response to infection and are mediators of the​ body's immune response. C. They are crucial in the formation of clots. D. They are responsible for the delivery of oxygen to the cells.

They are responsible for the delivery of oxygen to the cells.

Why should an EMT utilize a local poison control center in managing a poisoning​ patient? A. It prevents the EMT from having to do training on poisons. B. They can activate a specialty team to come and treat the patient. C. They can assist in finding out important information about the poison. D. It places any legal liability on the poison control center.

They can assist in finding out important information about the poison.

Why are infants and young children more prone to hypothermia than​ adults? A. They have less fat than adults. B. They are unable to recognize how cold they are. C. Their immune system is not fully developed. D. They shiver more than adults.

They have less fat than adults.

Which of the following is TRUE about​ infants? A. They typically breathe through the mouth. B. The chest wall is more rigid than that in adults. C. They typically breathe through the nose. D. Hyperextension of the neck is needed to open the airway.

They typically breathe through the nose.

Which of the following is responsible for most of the signs and symptoms of early​ shock? A. Constriction of the peripheral blood vessels B. Dilation of the peripheral blood vessels C. Increased respirations of the patient D. The​ body's attempts at compensation for blood loss

The​ body's attempts at compensation for blood loss

You are dispatched to a private residence for a sick person. You arrive and find a​ 47-year-old male patient who recently completed his peritoneal dialysis and complains of severe abdominal pain that is worsened by movement. One of the more serious complications of this type of dialysis is a bacterial infection within the peritoneal cavity. What would be the​ tell-tale sign that this is what is happening in this​ case? A. The​ patient's dialysis fluid appears cloudy when it is drained from the peritoneal cavity rather than its normal clear appearance. B. The​ patient's dialysis fluid appears clear when it is drained from the peritoneal cavity rather than its normal cloudy appearance. C. The patient has trouble urinating. D. The patient has swollen ankles.

The​ patient's dialysis fluid appears cloudy when it is drained from the peritoneal cavity rather than its normal clear appearance.

Which of the following BEST describes the events that occur during the first stage of​ labor? A. Expulsion of the fetus through the birth canal B. Expulsion of the placenta C. Thinning and dilation of the cervix D. Rupture of the amniotic sac

Thinning and dilation of the cervix

Which of the following statements concerning forcible restraint of patients is true​? A. This is necessary only with mentally ill individuals. B. This is normally considered to be within the jurisdiction of law enforcement. C. It causes irreparable emotional harm to most patients. D. It is easily accomplished by two EMTs with a good plan.

This is normally considered to be within the jurisdiction of law enforcement.

To which of the following sections of the spine are the ribs​ attached? A. Sacral B. Thoracic C. Cervical D. Lumbar

Thoracic

How many times a week do the majority of American hemodialysis patients receive​ treatment? A. Seven B. Five C. One D. Three

Three

What is the purpose of a tracheostomy in an ill​ patient? A. To assist the patient with breathing B. To assist the patient with feedings C. To assist the patient with sleeping D. To assist the patient with waste removal

To assist the patient with breathing

Which of the following is a purpose for carrying sterilized aluminum foil on an​ ambulance? A. To control major arterial bleeding B. To care for amputated parts C. To keep a newborn warm D. None of the above

To keep a newborn warm

Which of the following is the most important reason for controlling external​ bleeding? A. To prevent the spread of infectious diseases B. To make​ clean-up of the ambulance and your equipment easier after the call C. To prevent the patient from becoming upset at the sight of blood D. To prevent hypoperfusion

To prevent hypoperfusion

Which of the following is the primary reason why ambulances should have two oxygen supply​ systems? A. To have a backup system in the event of failure of the primary system B. To provide oxygen to the patient in the event of vehicle breakdown C. To provide oxygen to two patients at one time D. To ensure a supply of oxygen for​ long-distance transports

To provide oxygen to two patients at one time

A physical examination of an infant or small child should generally follow which​ order? A. Front to back B. Head to toe C. Toe to head D. Extremities to trunk

Toe to head

Which of the following types of bandages should NOT be used by the EMT providing wound care? A. Triangular bandage B. Tourniquet C. Self-adherent roller gauze D. Adhesive tape

Tourniquet

Which of the following describes any substance produced by a living organism that is poisonous to human​ beings? A. Acid B. Antibody C. Toxin D. Antigen

Toxin

Which of the following is an unreliable sign for determining the presence of a tension pneumothorax? A Distended neck veins B Signs and symptoms of shock C Shortness of breath D Trachea that shifts to the side opposite the injury

Trachea that shifts to the side opposite the injury

Your patient is a​ 24-year-old male who is severely depressed. He tells you that he​ can't "handle the​ pressure" anymore and that he wants to die. He is refusing transport. Which of the following is the most appropriate decision regarding this​ patient's care? A. Respect the​ patient's wishes; he is​ alert, oriented, and capable of giving consent. B. Transport the patient against his will with the assistance of law enforcement. C. Contact the​ patient's family to find out what their wishes are. D. Leave the scene and allow law enforcement to handle the situation.

Transport the patient against his will with the assistance of law enforcement.

You respond to a boat dock for a diving injury. You find a​ 22-year-old female patient unresponsive with frothy blood in the mouth and lung sounds absent on the right side. The patient is breathing 28 times a minute. The​ patient's friends state they were diving when she unexpectedly came out of the water complaining of chest pains and then collapsed. The nearest hospital is 25 minutes away and the nearest specialty resource center with a hyperbaric chamber is 30 minutes away. What is the best decision when determining​ transport? A. Transport the patient by air medical to the nearest facility. B. Transport the patient by ground to the specialty center. C. Transport the patient to the nearest facility. D. Remain on scene until ALS personnel arrive.

Transport the patient by ground to the specialty center.

You are stabilizing a patient who has just been stabbed in the chest to the right of the mediastinum. After placing the patient on supplemental oxygen, his shortness of breath resolves. You also cover the wound with an occlusive dressing. The patient is asymptomatic at the time you're making the decision to transport. Which of the following BEST encapsulates the correct strategy for transport? A. Transport the patient non-emergently because he's complaint free. B. The patient does not necessarily need transport, so allow him to refuse if he wants. C. Begin transport non-emergently and upgrade if the patient's condition deteriorates. D. Transport the patient emergently because of the high index of suspicion for a serious injury.

Transport the patient emergently because of the high index of suspicion for a serious injury.

Your patient is a 17yearold with a history of asthma. She is complaining of pain in her lower abdomen. Assessment reveals that her breath sounds are clear and equal, she has a respiratory rate of 28 breaths per minute, a heart rate of 96 beats per minute, and a blood pressure of 112/74 mmHg. Which of the following is the MOST appropriate next step? A. Insert a nasopharyngeal airway in the patient's nose. B. Transport the patient for further evaluation. C. Have the patient breathe into a paper bag to rebreathe some of her carbon dioxide. D. Assist the patient in the administration of her metereddose inhaler.

Transport the patient for further evaluation.

When transporting patients from the scene of a​ mass-casualty incident, who should be responsible for contacting the receiving​ hospital? A. Triage officer B. Transportation officer C. Transporting EMT D. Incident commander

Transportation officer

Who is the person that is in charge of tracking the​ patient's final​ destination? A. Staging officer B. Incident Commander C. Treatment officer D. Transportation officer

Transportation officer

What is the number one cause of death in pediatric​ patients? A. Trauma B. Diabetes C. Sepsis D. Anaphylaxis

Trauma

Which of the following is the MOST common cause of death in​ children? A. Trauma B. Infectious disease C. Poisonings D. Environmental emergencie

Trauma

Your patient was working on a car when it fell off the jack and trapped him between the tire and ground. His face is very blue and his eyes are bloodshot. Which of the following has the patient most likely suffered? A. Traumatic asphyxia B. Pneumothorax C. Hemothorax D. Flail chest

Traumatic asphyxia

Which of the following measures is NOT appropriate for a patient with a significant isolated head​ injury? A. Treat for shock by elevating the foot of the backboard. B. Control bleeding from head wounds. C. Try to keep the patient from being agitated. D. Keep the patient from becoming overheated.

Treat for shock by elevating the foot of the backboard.

While treating a pregnant trauma​ patient, which of the following is the BEST way to keep the fetus​ alive? A. Transport the patient to the closest pediatric center. B. Treat the patient as any other trauma patient. C. Do an emergency cesarean section of the fetus. D. Do not put the patient on a spine board to prevent supine hypotensive syndrome.

Treat the patient as any other trauma patient.

Which of the following methods should the EMT use first to attempt to access a patient in a vehicle while awaiting arrival of a rescue​ crew? A. Use a heavy tool to break the window closest to the patient. B. Use a​ spring-loaded punch to break the rear window. C. Try all of the​ vehicle's doors to see if they will open. D. Use a​ pry-bar to release the Nader pin in the door latch.

Try all of the​ vehicle's doors to see if they will open.

If the​ baby's umbilical cord is noted to be wrapped around his neck after the head is​ delivered, which of the following should be​ done? A. Clamp the cord in two​ places, but do not cut it until the baby is delivered. B. Immediately cut the cord before delivering the baby. C. Try to slip the cord over the​ baby's head and shoulder. D. Transport emergently without further intervention.

Try to slip the cord over the​ baby's head and shoulder.

Which of the following serious health problems is related to​ homelessness? A. Heart disease B. Liver disease C. Hypertension D. Tuberculosis

Tuberculosis

You are called at​ 6:00 a.m. to a private residence where the parents found their​ 9-month-old infant in the crib and not breathing. When you examine the​ baby, you note that rigor mortis is present. What should you​ do? A. Call ALS to the scene to confirm the rigor mortis and talk to the parents about it. B. Transport the infant immediately and indicate to the parents that signs are hopeful. C. Turn your attention to supporting the parents. D. Tell the parents that you will save their child.

Turn your attention to supporting the parents.

The U.S. Department of Transportation has issued specifications for all of the following types of ambulances ​except: A. Type IV. B. Type I. C. Type III. D. Type II.

Type IV.

Which of the following personal protective items must be used by EMS personnel caring for patients in a hazardous materials​ emergency? A. HEPA mask B. Chemical permeable gloves C. Tyvek or other chemical impermeable coveralls D. Structural firefighting gear

Tyvek or other chemical impermeable coveralls

Which of the following findings would be helpful in differentiating between a closed head injury and a behavioral​ emergency? A. Combativeness B. Irrational behavior C. Confusion D. Unequal pupils

Unequal pupils

You are on the scene of a riot following a decision in a controversial court case. A large group of people have set fire to​ vehicles, broken​ windows, and assaulted people in the area. Which of the following would be the BEST method of incident​ command? A. Unified command B. Singular command by EMS C. Singular command by law enforcement D. Singular command by fire service

Unified command

Which patient is the most​ unstable? A. Unresponsive patient with hypotension B. Unresponsive patient with tachycardia C. Patient who only responds to verbal stimuli D. Patient responsive to painful stimuli only

Unresponsive patient with hypotension

Which of the following would LEAST likely indicate that an emergency is due to drugs or​ alcohol? A. Hospital discharge order with a pain prescription B. Open sores and scars to the upper arms C. Empty liquor bottles at the scene D. Unresponsive patient with unequal pupils

Unresponsive patient with unequal pupils

The maxillae form which of the following​ structures? A. Forehead B. Lower jaw C. Cheek bones D. Upper jaw

Upper jaw

Which of the following are desirable characteristics of the​ "safe or cold​ zone" in a hazardous materials​ incident? A. ​Upwind, uphill from the spill B. ​Downwind, uphill from the spill C. ​Downwind, downhill from the spill D. ​Upwind, on the same level as the spill

Upwind, on the same level as the spill

What is one of the most common diseases to affect the renal and urinary​ system? A. Kidney stones B. Acute renal failure C. ​End-stage renal disease D. Urinary tract infections

Urinary tract infections

Which of the following is recommended in situations in which a tourniquet must be​ used? A. Use a material that is wide and thick. B. Apply a bulky dressing and bandage over the tourniquet. C. Remove the tourniquet as soon as bleeding is controlled to minimize further damage to the limb. D. Apply the tourniquet over the elbow or knee.

Use a material that is wide and thick.

Your patient is a​ 16-year-old male who ran his truck off of the road and into a ditch. He was driving just fast enough to cause the air bag to deploy when he hit the ditch. He is complaining of some neck and shoulder pain. His vital signs are respirations​ 20, pulse​ 110, blood pressure​ 116/80, and pupils equal and reactive. How should you manage this​ patient? A. Perform a rapid extrication and spinal immobilize him on a long spine board. B. Use a short spine board or vest​ device, then transfer him to a long spine board. C. Apply a cervical​ collar, and then rotate him to a long spine board. D. Place him on​ oxygen, and then slowly move him to the stretcher.

Use a short spine board or vest​ device, then transfer him to a long spine board.

Which of the following is required in the management of all open soft-tissue injuries? a Provision of high-concentration oxygen b Use of Standard Precautions by the EMT c Cervical spine precautions d Application of a disinfectant solution

Use of Standard Precautions by the EMT

If the​ baby's head is delivered with the amniotic sac still​ intact, which of the following should be done​ first? A. Call medical control before taking action. B. Use the scissors in the obstetrics kit to cut the sac away from the​ baby's head. C. Use your fingers to tear the sac away from the​ baby's face. D. Leave the sac intact until the entire body is delivered.

Use your fingers to tear the sac away from the​ baby's face.

Which of the following is the MOST effective way of controlling external​ bleeding? A. Running cold water over the wound B. Using an ice pack C. Using direct pressure with a dressing D. Elevating the affected part

Using direct pressure with a dressing

Which of the following types of vessels have valves to maintain​ one-way blood​ flow? A. Arterioles B. Arteries C. Veins D. Capillaries

Veins

Your patient has had his throat slashed during a robbery attempt. You are concerned since it is apparent that the vessels in his neck have been lacerated. A breach in which of the following vessels would be most likely to lead to an air​ embolism? A. Arterioles B. Arteries C. Veins D. Capillaries

Veins

The purpose of a constricting band after a venomous snake bite is to impede the flow of which of the​ following? A. Lymph B. Arterial blood C. Venom D. Both A and C

Venom

A patient uses a home ventilator connected to his tracheostomy tube to breathe. Which of the following is the correct procedure to use when moving this patient to the​ ambulance? A. Ventilate the patient with​ mouth-to-mask ventilations at a normal rate and volume while the patient is disconnected from the ventilator. B. Do​ nothing; there is no need to ventilate the patient between the house and the ambulance. C. Ventilate the​ patient's nose and mouth with a​ bag-valve mask at a normal rate and volume while the patient is disconnected from the ventilator. D. Ventilate the​ patient's tracheostomy tube via a connected​ bag-valve mask at a normal rate and volume while the patient is disconnected from the ventilator.

Ventilate the​ patient's tracheostomy tube via a connected​ bag-valve mask at a normal rate and volume while the patient is disconnected from the ventilator.

Rough handling of a patient with severe hypothermia may result in which of the​ following? A. Heart attack B. Ventricular fibrillation C. Seizures D. Rewarming shock

Ventricular fibrillation

Which of the following groups of people makes up the largest portion of the homeless​ population? A. Children B. Females C. Families D. Veterans

Veterans

Reducing the pain of a marine animal sting can be accomplished by rinsing the affected area with which of the​ following? A. Sterile saline solution B. Gasoline or kerosene C. Vinegar D. Cold water

Vinegar

Cleaning​ fluid, glue, and model cement are common examples of which of the​ following? A. Volatile chemicals B. Aromatic spirits C. Narcotics D. Hallucinogens

Volatile chemicals

Your patient is an​ 18-year-old male who fell into a​ neighbor's swimming pool but was pulled out by bystanders. The neighbors report that the patient wandered into their yard and that he said he had a​ "funny numb​ feeling" in his head and his chest hurt before stumbling into the pool. As you are performing​ CPR, you notice that the mucous membranes in his mouth and nose are swollen. Which of the following would account for the​ patient's behavior and current​ condition? A. Methamphetamine B. Carbon monoxide poisoning C. GHB D. Volatile chemicals

Volatile chemicals

When responding to a​ multiple-casualty incident, staging refers to which of the​ following? A. Keeping a log of all patients transported B. Setting up a triage center from the back of your ambulance C. Staying on station until called upon to receive a patient D. Waiting in a designated location until called upon by the designated sector officer

Waiting in a designated location until called upon by the designated sector officer

Where is the decon corridor located where​ patients, personnel, and equipment are​ decontaminated? A. Cold zone B. Cool zone C. Warm zone D. Hot zone

Warm zone

Disaster plans are a predefined set of instructions that should be written to address the events that are conceivable for a particular location and which other three​ parts? A. Well​ publicized, realistic, and rehearsed B. ​Written, multijurisdictional, and rehearsed C. ​Written, realistic, and rehearsed D. Well​ publicized, multijurisdictional, and rehearsed

Well​ publicized, realistic, and rehearsed

Which of the following questions is inappropriate when taking the history of a female patient with abdominal pain? A. Are you having vaginal bleeding or discharge now? B. Have you had sexual intercourse since your last menstrual period? C. If you are menstruating, is the flow normal? D. What is your sexual orientation?

What is your sexual orientation?

You are called to the scene of a​ 21-year-old female patient who is unresponsive. Her boyfriend suspects that she tried to commit suicide by overdosing on her depression medications. The patient is unresponsive to painful​ stimuli, with agonal​ respirations, and has vomited. She has a weak carotid pulse. After securing the airway and providing oxygen by​ bag-valve mask, what is the MOST important question the EMT can ask regarding the​ medication? A. What medication did she​ take? B. Why is she on that particular depression​ medication? C. How many pills did she​ take? D. Were the medications​ new?

What medication did she​ take?

At which point may manual stabilization of the cervical spine be​ terminated? A. After a short immobilization device has been applied B. When the patient is secured to a long backboard C. After a cervical collar has been applied D. Only when directed to do so by medical control

When the patient is secured to a long backboard

Which of the following should NOT be included in documentation of an incident involving a patient with a behavioral or psychiatric​ emergency? A. Your actions B. Description of the​ patient's behavior C. Whether or not you think the patient is mentally ill D. Your observations of the​ patient's surroundings at the scene

Whether or not you think the patient is mentally ill

45. You are dispatched to a local industrial plant for an "electrical injury." You arrive on-scene and find a 46-year-old male lying supine in front of an electrical panel. You are told he was attempting to make a repair and somehow received an electrical shock and was thrown to the ground. The scene is safe and the electricity is off. Your initial exam reveals a conscious person, breathing adequately. Vital signs are normal and there are no obvious signs of burns. Coworkers state that he was unconscious until your arrival. What is the next step? a. Load and go—conduct any other care in the ambulance en route due to the potential seriousness of the burns. b. Put the patient on oxygen and help him into your ambulance. c. Stay with the patient awhile, and if he does not appear to be in distress, encourage him to refuse care. d. While on the scene, rapidly do a complete assessment, provide oxygen, provide care for potential spine injuries, and transport as soon as possible after the exam.

While on the scene, rapidly do a complete assessment, provide oxygen, provide care for potential spine injuries, and transport as soon as possible after the exam.

The term blanching​, used to describe localized cold​ injury, means the tissue has turned to which of the following​ colors? A. White or lighter B. Blue or purple C. Red D. Black

White or lighter

Your patient is a conscious​ 16-year-old female who has ingested an unknown number of sleeping pills. Which of the following questions is LEAST pertinent to the care of this​ patient? A. Has anyone tried to treat you with​ anything? B. How much do you​ weigh? C. Over what period of time did you take the​ pills? D. Who do these sleeping pills belong​ to?

Who do these sleeping pills belong​ to?

Your patient is a​ 7-year-old male who ate an unknown type of mushroom on a dare by his friends. You should ask all of the following questions except​: A. Why did you eat the​ mushroom? B. When did you eat the​ mushroom? C. How much did you​ eat? D. Are there any more of the​ mushrooms?

Why did you eat the​ mushroom?

Which of the following is NOT used to calculate a​ patient's GCS? A. Eye movement B. Verbal response C. Work of breathing D. Motor response

Work of breathing

47. You are treating the amputation of three fingers on a 40-year-old male. The fingers were torn off while he was cleaning his snow blower. You have stopped the bleeding. What should you do with the amputated fingers? a. Fingers cannot be reattached so you can discard them in red bag trash. b. Because it is cold out, you merely need to wrap them in a 5 x 9 dressing and give them to the staff at the ED. c. Wrap them in a sterile dressing, put them in a plastic bag, and keep them cool. d. Place the fingers directly on ice or use cold packs, as they must be kept very cold.

Wrap them in a sterile dressing, put them in a plastic bag, and keep them cool.

Which of the following is appropriate when caring for a premature​ baby? A. Deeply suctioning the airway with a rigid​ tonsil-tip B. Applying​ high-concentration oxygen with a neonatal nonrebreather mask C. Encouraging family members to hold the infant D. Wrapping the infant in a​ blanket, covering his​ head, and keeping the temperature in the ambulance between 90degrees°F and 100degrees°F

Wrapping the infant in a​ blanket, covering his​ head, and keeping the temperature in the ambulance between 90degrees°F and 100degrees°F

Which of the following is the easiest way for an EMT to communicate with a patient with drastic hearing​ loss? A. Use a​ TDD/TYY telephone. B. Speak loudly and directly into the​ patient's ear. C. Write your questions using paper and pen. D. Slow down your speech so that the patient can read your lips.

Write your questions using paper and pen.

Patients with a history of chronic sickle cell anemia​ (SCA) may present with which of the following signs and​ symptoms? A. Pale appearance and bradypnea on exertion B. Yellowing of the skin and dyspnea on exertion C. Flushed appearance and shortness of breath D. Bluish discoloration of the skin and shortness of breath

Yellowing of the skin and dyspnea on exertion

You have transported a stable patient with complaints of having abdominal pain for the last three weeks to the hospital. On​ arrival, you notice the emergency department is very busy and there are no empty beds to be found. You have attempted to get the attention of a hospital staff member to transfer the​ patient, but all are currently helping other patients. Your partner tells you to take the patient to the waiting room and leave him since the dispatcher of your service is holding several serious calls. What could possibly happen to you if you leave the patient in this​ fashion? A. ​Nothing; there are other people who are in need of an ambulance. B. You could possibly be charged with false imprisonment. C. You could possibly be charged with abandonment. D. ​Nothing, provided that your supervisor authorizes this action.

You could possibly be charged with abandonment.

49. You are caring for a 23-year-old female who fell off of a bicycle and sustained a severe laceration on the inside of her upper thigh. Her slacks are torn, and you can see most of the wound. What is the next step? a. You can treat the wound without total exposure as exposing a wound in that area of the body could be embarrassing. b. You need to expose the wound completely because you need to clear away any embedded particles and debris from the wound. c. You need to expose the wound completely, control bleeding, clean the surface by simply removing large pieces of foreign matter if any, and dress and bandage the wound. d. You need to make sure the wound is very clean before trying to control bleeding by exposing the wound completely and cleaning it with alcohol.

You need to expose the wound completely, control bleeding, clean the surface by simply removing large pieces of foreign matter if any, and dress and bandage the wound.

Most cases of accidental poisoning involve which of the​ following? A. Infants B. Young children C. Adolescents D. Adults

Young children

You are called for a possible imminent delivery. Your patient is a​ 15-year-old girl who is in your estimation about 8 months pregnant. She tells you that she has been hiding her pregnancy. You are especially concerned about problems she might have during the delivery. Which of the following are findings that may indicate the need for neonatal​ resuscitation? A. Patient with a blood pressure of​ 130/82 B. Mother who has had five previous births C. Patient whose water has already broken D. Young mother who has not had prenatal care

Young mother who has not had prenatal care

As an adult​ ages, the body experiences changes that​ cause: A. a decreased compensatory ability. B. disease processes to take place. C. an inability to maintain homeostasis. D. an increased blood pressure.

a decreased compensatory ability.

Failure to formally transfer the patient to medical staff in the emergency department could place the EMT in danger of being held liable​ for: A. patient endangerment. B. false imprisonment. C. abandonment. D. unlawful consent.

abandonment.

A nasopharyngeal airway should​ be: A. about the thickness of the​ patient's little finger. B. the length of the​ patient's nose. C. the width of the tragus of the ear. D. able to reach from one of the​ patient's earlobes to the other.

about the thickness of the​ patient's little finger.

A wound in which the epidermis is scraped away with minimal bleeding, such as commonly occurs when a child falls on his knees on a sidewalk, is called a(n): a abrasion. b contusion. c avulsion. d evisceration.

abrasion.

While assessing the mental status of an elderly patient who has been retired for several​ years, you notice he has difficulty answering your questions. He seems depressed. Based on this​ information, you​ should: A. consider the need for advanced provider care. B. suspect a​ life-threatening neurological event. C. consider his mental status as being decreased. D. acknowledge the patients fears and try to put them in perspective.

acknowledge the patients fears and try to put them in perspective.

Applying an external source of heat to the​ patient's body to rewarm him is called​ ________ rewarming. A. active B. central C. peripheral D. endogenous

active

For a pediatric patient with complications related to a tracheostomy​ tube, your care will primarily focus on​ addressing: A. trauma. B. dehydration. C. airway problems. D. malnutrition.

airway problems.

When assessing the mental status of a​ child, a well child will likely​ be: A. drowsy. B. unresponsive. C. inattentive. D. alert.

alert.

When interviewing the elderly​ patient, the EMT can increase the accuracy of the information he or she gains​ by: A. allowing sufficient time for the patient to answer. B. speaking louder than normal. C. having his or her partner assist with asking questions. D. asking multiple questions at the same time.

allowing sufficient time for the patient to answer.

In order for the pediatric​ patient's parents to be​ calm, you​ should: A. demonstrate your authority. B. promise them that everything will be fine. C. also be calm. D. separate them from the child.

also be calm.

The KKK standards deal​ with: A. minimum standard medical protocols. B. training and education of field personnel. C. air evacuation of trauma victims. D. ambulance safety and design.

ambulance safety and design.

Signs and symptoms of alcohol abuse include all of the following ​except: A. confusion. B. an acetone or ketone odor to the breath. C. nausea and vomiting. D. swaying and unsteadiness of movement.

an acetone or ketone odor to the breath.

When an elderly patient complains of abdominal​ pain, the EMT should consider this symptom to​ be: A. ​cardiac-related until proven otherwise. B. an indication of a potentially serious problem. C. ​non-life-threatening unless accompanied by other symptoms. D. related to a gastrointestinal problem

an indication of a potentially serious problem.

When palpating the pulse of an elderly​ patient, you notice that there is an irregular pulsation without any kind of pattern or cycle to it. This is​ called: A. pulsus paradoxus. B. pulsus alternans. C. an irregularly irregular pulse. D. heart failure.

an irregularly irregular pulse.

54. The meatus is: a. an orifice of the urethra. b. in a male, the area between the scrotum and the anus. c. the tube that carries urine from the kidney to the bladder. d. the canal through which urine is discharged from the bladder to the exterior of the body.

an orifice of the urethra.

With the exception of the ________, most abdominal organs are not able to sense tearing sensations. A. aorta B. liver C. ovaries D. colon

aorta

While palpating the radial pulses of a patient who was involved in a motor vehicle crash, you notice a difference in the strength of the pulses bilaterally. This is a finding that you suspect may be associated with: A. commotio cordis. B. aortic dissection. C. tension pneumothorax. D. flail chest.

aortic dissection.

You are called to a 25 year old male complaining of right lower quadrant (RLQ) pain. His other symptoms are nausea and vomiting, fever, and decreasing pain in the umbilicus area. As an EMT, you feel this patient might have: A. cholecystitis. B. pancreatitis. C. appendicitis. D. peritonitis.

appendicitis.

You are called to the scene of a 16yearold female patient complaining of severe lower quadrant abdominal pain. The patient states she is sexually active. Upon palpation, you observe rebound tenderness in the right lower quadrant. You suspect: A. ectopic pregnancy. B. renal colic. C. appendicitis. D. cholecystitis.

appendicitis.

A teenage male has fallen onto a railing while skateboarding. He complains of right-sided chest pain and moderate dyspnea. Exposure of the chest reveals a section of his ribs that is moving opposite of the rest of the ribs. You should: A. apply a bulky dressing over the section. B. roll the patient over onto his right side. C. cover the wound with an occlusive dressing. D. begin positive pressure ventilation.

apply a bulky dressing over the section.

One of the most effective and widely available drugs to prevent the aggregation of platelets​ is: A. anemia. B. hemoglobin. C. plasma. D. aspirin.

aspirin.

A study of emergency collisions demonstrated that the majority of collisions occurred A. at night. B. during turning. C. at intersections. D. during inclement weather.

at intersections.

The part of the skeleton that is made up from the skull and spinal column is called the: A. torso. B. boney skeleton. C. axial skeleton. D. appendicular skeleton.

axial skeleton.

You respond to a landing zone to pick up a skier who was lost in the woods for 36 hours and found by the search and rescue helicopter. Your patient is a​ 19-year-old male patient. He is alert and oriented to​ time, person,​ place, and event. He is covered in blankets and shivering. He complains of not being able to feel his fingers. Physical exam reveals that the fingers of both extremities have a waxy appearance and feel​ "frozen" on the surface. After treating the patient for potential hypothermia and rapid​ transport, your next action is​ to: A. bandage the digits. B. keep the digits frozen on ice until they can be properly rewarmed at the hospital. C. actively rewarm the digits. D. gently warm the digits by slowly massaging them.

bandage the digits.

When resuscitating a hypothermic patient in cardiac​ arrest, resuscitation attempts must continue until the patient​ has: A. developed rigor mortis. B. been ventilated for at least 30 minutes with an oropharyngeal airway​ (OPA) in place. C. been defibrillated a total of nine times. D. been rewarmed.

been rewarmed.

Your patient is a 17-year-old male baseball player found pulseless and apneic after being struck in the chest by a baseball 6 minutes ago. He is surrounded by other players and staff but no one is providing care. You should: A. attach the AED and analyze. B. place him on a backboard. C. begin chest compressions. D. elevate the patient's legs.

begin chest compressions.

Your unconscious infant has a complete airway obstruction. You​ should: A. alternate back blows and chest thrusts. B. administer abdominal thrusts. C. begin chest compressions. D. sweep out the obstruction with a gloved finger.

begin chest compressions.

You are treating a patient with paradoxical motion on the left side of the chest. He is breathing shallow at a rate of 4 breaths per minute. You should: A. roll the patient onto his left side. B. apply a bulky dressing to the chest. C. administer 15 lpm oxygen via mask. D. begin positive pressure ventilation.

begin positive pressure ventilation.

You are dispatched to an unconscious hemodialysis patient. On arrival to the dialysis​ clinic, the patient is​ unresponsive, apneic, and pulseless. You secure the​ ABCs, begin​ ventilation, and initiate chest compressions.​ However, the​ patient's cardiac arrest rhythm is continuously unresponsive to defibrillations with your AED. The best approach is​ to: A. begin transporting and contact an ALS intercept. The​ patient's dysrhythmia may be related to kidney failure. B. call online medical control to terminate the code. The probability of survival is too low to continue resuscitation efforts. C. continue defibrillating as the monitor recommends. The myocardium becomes increasingly responsive to successive defibrillations. D. run a​ "no-electricity" code in which compressions and ventilation are continued. No additional defibrillations are necessary.

begin transporting and contact an ALS intercept. The​ patient's dysrhythmia may be related to kidney failure.

A patient has a dry powder poison covering both upper extremities and chest. You​ should: A. flush the area with large amounts of water and then brush the remaining poison off. B. immediately wash the powder off to prevent further exposure. C. cover the​ patient's arms with a burn sheet to preserve the powder for the emergency room. D. brush the powder off and irrigate the contaminated area with large amounts of water.

brush the powder off and irrigate the contaminated area with large amounts of water.

You are called to a farm for a possible organophosphate poisoning of one of the workers. Your first action is​ to: A. decontaminate the patient. B. call for specialized hazardous materials units. C. call for an ALS unit. D. bag the patient before he dies.

call for specialized hazardous materials units.

It is a chilly fall morning and you are called to an RV campground for three patients who are complaining of​ headache, dizziness, and nausea. Your primary assessment reveals that they are cyanotic and have an altered mental status. You​ suspect: A. snake bite. B. bee sting. C. food poisoning. D. carbon monoxide poisoning.

carbon monoxide poisoning.

The pathophysiology of ________ is one in which the pericardial sac fills with blood to the point where the chambers of the heart no longer fill adequately, usually secondary to trauma. A. cardiac tamponade B. hemopneumothorax C. pericardial effusion D. commotio cordis

cardiac tamponade

Your patient has been injured by a fall down a flight of steps. He is alert and oriented but complains of back and neck pain. You spinal immobilize him on a long spine board with a cervical collar on his neck as a precaution because you know that​ the: A. lumbar area is rarely injured due to the rib support. B. thoracic spine is especially vulnerable to injury. C. cervical spine is the most vulnerable part of the spine. D. coccyx is easily dislocated.

cervical spine is the most vulnerable part of the spine.

You respond to a 75yearold female who is complaining of epigastric pain that feels like heartburn. The pain radiates to the right shoulder. Her vital signs are stable and she has a previous history of a myocardial infarction. She has prescription nitroglycerin tablets. She is most likely suffering from: A. cholecystitis. B. renal colic. C. Mittelschmerz. D. abdominal aortic aneurysm.

cholecystitis.

You encounter a patient who has been having diffuse abdominal pain for one week. The patient has a pulse rate of​ 86, a blood pressure of​ 140/90, and a respiratory rate of 20.​ However, the patient appears​ pale, and complains of generalized weakness and shortness of breath during everyday activities. You suspect the patient is​ experiencing: A. sickle cell anemia. B. acute anemia. C. anxiety disorder. D. chronic anemia.

chronic anemia.

A young female was injured in an automobile​ wreck, striking her head on the windshield. She is unconscious and her breathing is irregular. She is bleeding from a scalp​ wound, but your assessment shows that her cranium is intact. Her vital signs are pulse​ 68, blood pressure​ 148/90, and her pupils are unequal. You should suspect​ a(n): A. contusion. B. open head injury. C. concussion. D. closed head injury.

closed head injury.

Your patient has a crush injury to the arm. The body can no longer perfuse the tissues against the pressure. This condition is known as: A. perfusing syndrome. B. crushing syndrome. C. compartment syndrome. D. fracture syndrome.

compartment syndrome.

Rough handling or agitation of a pediatric patient with epiglottitis could lead​ to: A. improved cardiac output. B. febrile seizures. C. elevated oxygen saturation. D. complete airway obstruction.

complete airway obstruction.

The mechanism of injury in which a patient's chest has struck an immovable object, such as a steering wheel, may most accurately be described as a(n): A. penetrating injury. B. coup contrecoup injury. C. compression injury. D. open trauma injury.

compression injury.

The use of a​ made-up experience to replace lost memories with imaginary ones is​ called: A. hysteria. B. denial. C. confusion. D. confabulation.

confabulation.

While interviewing your​ patient, she tells you that her husband will be back from the store shortly and can get you a list of her medications. Her daughter tells you that the​ patient's husband died 15 years ago. This situation is known​ as: A. confabulation. B. imagination. C. hallucination. D. contemplation.

confabulation.

Chloe was born with a cardiac problem that she will have to manage the rest of her life. This disease is considered to​ be: A. genetic. B. acquired. C. autoimmune. D. congenital.

congenital.

You respond to a 75yearold female who is complaining of epigastric pain that feels like heartburn and radiates to the right shoulder. Her vital signs are stable and she has a previous history of myocardial infarction. She has prescription nitroglycerine tablets. An ALS unit is en route. After performing a physical examination and applying oxygen by nasal cannula, you should: A. contact medical control regarding the administration of the patient's nitroglycerin. B. cancel the ALS unit: this is just gallstones and BLS can transport. C. transport the patient in the Trendelenburg position for shock. D. apply the AED and prepare for imminent cardiac arrest from a myocardial infarction.

contact medical control regarding the administration of the patient's nitroglycerin.

An injury in which the epidermis remains intact, but blood vessels and cells in the dermis are injured, is called a(n): a contusion. b abrasion. c concussion. d avulsion.

contusion.

Your patient has an open abdominal wound with intestines protruding. You should: A. cover the organs with moist sterile dressings. B. cover the entire abdomen with a blanket. C. leave them alone and transport rapidly. D. attempt to place the organs back into the abdomen.

cover the organs with moist sterile dressings.

You have covered an open chest wound with your gloved hand and the patient's breathing has improved. In order to free your hand to provide further care, you should: A. cover the wound with a sterile dressing. B. remove your hand to see if the wound has closed. C. cover the wound with an occlusive dressing. D. apply a bulky dressing over the wound.

cover the wound with an occlusive dressing.

To conserve your pediatric​ patient's body​ heat, you​ should: A. keep the​ patient's head and torso exposed. B. cover the​ patient's head and torso. C. keep the​ patient's head covered but not the torso. D. cover the​ patient's torso but not the head.

cover the​ patient's head and torso.

The strong white fibrous material called the periosteum: A. surrounds the heart. B. covers the bones. C. protects the perineum. D. surrounds the abdominal cavity.

covers the bones.

The sound or feeling of the ends of broken bones rubbing together is called: A. crapilation. B. breakilation. C. crepitus. D. krepitus.

crepitus.

An injury caused by heavy pressure to the tissues, such as when an extremity is trapped under a fallen tree, that results in damage to muscle cells and the accumulation of waste products in the tissue is called a(n): a crush injury. b evisceration. c contusion. d abrasion.

crush injury.

When informing someone of the death of a loved one due to a​ multiple-casualty incident, you should use the​ term: A. expired. B. sickly. C. dead. D. passed away.

dead.

The term gangrene means​ tissue: A. damage. B. death. C. discoloration. D. infection.

death.

The elderly patient will most likely experience​ ________ as he or she continues to age. A. decreased chewing difficulty B. increased food absorption C. increased stomach acid secretion D. decreased intestinal tract movement

decreased intestinal tract movement

You are palpating the abdomen of a motor vehicle collision patient when you feel a pulsing mass. You should: A. defer further abdominal palpation. B. apply cold packs to the abdomen. C. ask your partner to verify your findings. D. bind the abdomen with an elastic bandage

defer further abdominal palpation.

A segment of the body associated with a specific nerve coming from the spinal​ cord, along which shingles can​ appear, is called​ a: A. myelin sheath. B. motor unit. C. dermatome. D. zoster.

dermatome.

A condition that interferes significantly with a​ person's ability to engage in activities of daily living is referred to as​ a: A. special need. B. physical handicap. C. disability. D. developmental disability.

disability.

All of the following are general rules in dealing with a psychiatric patient except​: A. make eye contact with the patient. B. listen to the​ patient, showing you are listening by repeating part of what the patient says back to him. C. speak slowly and​ clearly, and use a calm and reassuring tone. D. do not be​ judgmental; instead, show​ pity, as he needs to know that you understand him.

do not be​ judgmental; instead, show​ pity, as he needs to know that you understand him.

As it pertains to hazardous​ materials, the Occupational Safety and Health Administration requires employers​ to: A. document the appropriate level of training for each employee. B. train employees to handle any type of hazardous materials incident. C. ensure there is at least one person​ on-site trained at the technician level. D. have knowledge of all hazardous materials on the premises.

document the appropriate level of training for each employee.

The most common form of a hemostatic agent is​ hemostatic: A. powders. B. tourniquet. C. granules. D. dressings.

dressings.

Police escorts of ambulances driving at emergency status are typically discouraged​ because: A. there is no guarantee that the police officer knows where the call is located. B. police officers are not accustomed to driving an ambulance. C. drivers who pull over for the police officer often pull back in front of the ambulance. D. ambulances do not move as fast as police cruisers.

drivers who pull over for the police officer often pull back in front of the ambulance.

You are called for a woman with severe abdominal pain. During your assessment and​ interview, she tells you that she is sexually active and there is a chance she could be pregnant. Her vital signs are pulse​ 122, respirations​ 22, blood pressure​ 96/62, and skin cool and pale. You should immediately​ suspect: A. internal bleeding. B. placenta previa. C. ectopic pregnancy. D. appendicitis.

ectopic pregnancy.

You have delivered a newborn and wrapped the baby in a dry blanket. During your reassessment of the​ mother, you note continued moderate vaginal bleeding. Care for this bleeding may involve all of the following EXCEPT​: A. elevating the​ mother's feet. B. massaging the uterus to control bleeding. C. encourage the mother to use the bathroom if possible. D. Transport the mother has a​ high-priority patient.

encourage the mother to use the bathroom if possible.

Because pediatric patients are abdominal​ breathers: A. evaluating their breathing is different than evaluating an adult. B. they are less susceptible to abdominal injury. C. their lungs are less easily damaged from trauma. D. they are less susceptible to fatigue of the muscles involved in respiration.

evaluating their breathing is different than evaluating an adult.

Your patient is exhibiting bizarre and aggressive​ behavior; he starts shouting and becomes violent. He has extra strength and appears insensitive to pain. Drug paraphernalia is visible​ on-scene. This behavior is known​ as: A. chemical delirium. B. sensory excitement. C. excited delirium. D. intoxication reaction.

excited delirium.

EMTs may come across a situation where a patient begins to act extremely agitated or psychotic. Elevated temperature and sometimes alcohol or drug intoxication may be present. The patient will soon cease​ struggling, and often within minutes the patient develops inadequate or absent respirations and subsequently dies. It is important for the EMT to be alert for this sequence of events in a patient who exhibits this behavior and monitor the patient constantly throughout the call. This condition is​ called: A. excited psychosis. B. bipolar disorder. C. psychotic delirium. D. excited or agitated delirium.

excited or agitated delirium.

The term for a baby developing inside the​ mother's womb after week 8​ is: A. neonate. B. chorion. C. infant. D. fetus.

fetus.

Your pregnant patient is in active labor. You note that her contractions are getting closer together and much stronger as the baby moves deeper into the birth canal. The muscle contraction mechanism that moves the baby toward birth is the same mechanism that​ moves: A. fluid into and out of the cell. B. food from the esophagus to the stomach. C. air down the bronchial tubes to the alveoli. D. sweat to the surface of the skin.

food from the esophagus to the stomach.

Your patient is a 3yearold girl who is unable to move her elbow after her mother picked her up by the forearm. Proper splinting of this injury would be to immobilize from the ________ to the ________. A. wrist; shoulder B. fingertips; shoulder C. forearm; humerus D. wrist; elbow

forearm; humerus

You have arrived on the scene of a child with reported difficulty breathing. The patient is a​ four-year-old male who is sitting on the​ floor, putting together a puzzle with his mother. One of the FIRST things you notice is that the child is grunting with each expiration. This finding is part of​ the: A. circulation assessment. B. patient history. C. mental status assessment. D. general impression.

general impression.

You have been called for a​ 32-year-old female who is in active labor. During your assessment and​ interview, you note that she is 37 weeks​ along, this is her first​ child, and her contractions are 5 minutes apart. She also tells you that her pregnancy is considered​ "high risk." Your first concern should​ be: A. completing a thorough secondary exam. B. calling for a​ back-up unit. C. getting to the hospital. D. assembling your delivery kit.

getting to the hospital.

You are the first on the scene of a​ two-car vehicle collision. Your patient is a front passenger who is unresponsive inside the vehicle. The patient is trapped and the vehicle is on fire. The fire department is still en route. You​ should: A. use your fire extinguisher to put out the fire. B. have the fire department rescue the patient. C. perform an urgent move. D. perform an emergency move.

have the fire department rescue the patient.

If a parent of your critically ill or injured pediatric patient is interfering with patient​ care, you​ should: A. contact medical direction for advice. B. stop patient care and calm the parent down. C. refuse to treat the child further until the parent calms down. D. have the parent removed from the​ patient-care area.

have the parent removed from the​ patient-care area.

You respond to a 65yearold patient complaining of abdominal pain. Your physical exam reveals a nonpulsating mass in the lower left quadrant. You suspect: A. hernia. B. splenic rupture. C. appendicitis. D. aortic abdominal aneurysm.

hernia.

The opiate triad includes all of the following ​except: A. pinpoint pupils. B. hypertension. C. respiratory depression. D. coma.

hypertension.

Applying a wet burn sheet to a pediatric burn patient increases the risk​ of: A. hypoxia. B. increased pain. C. fluid retention. D. hypothermia.

hypothermia.

Wind and wet clothing increase the pediatric​ patient's risk​ of: A. hypothermia. B. hypoxia. C. hyperthermia. D. hypotension.

hypothermia.

Your pediatric patient accidentally ingested a medication that causes widespread vasodilation. This increases the​ patient's risk​ of: A. hypertension. B. hyperthermia. C. hypothermia. D. hemorrhage.

hypothermia.

You respond to a construction site on a very hot summer day for a person​ "not acting​ right." You find a​ 44-year-old male patient seated in a work vehicle with the air conditioner on maximum. Coworkers state the patient was working for the last 2 hours pouring concrete when he started​ "speaking gibberish" and nearly collapsed. The patient is responsive to verbal stimuli. Your initial vital signs are blood pressure​ 90/60, pulse​ 136, and respiratory rate 24. The​ patient's skin is cool to the touch and​ dry, but his shirt is wet with visible salt rings. You​ suspect: A. hypovolemic shock. B. hypernatremia. C. myocardial infarction. D. heat cramps.

hypovolemic shock.

Your​ 38-year-old male patient has been injured in an assault. He has several facial lacerations and a large knot on his head. Bystanders say that he was thrown over a large table and landed on his head and shoulder. As part of your scene​ size-up and primary​ assessment, you​ should: A. immobilize his cervical spine. B. complete an initial set of vital signs. C. splint any other bone or joint injuries. D. wait for ALS to treat the patient

immobilize his cervical spine.

A pediatric patient has extensive burn injuries. Rescuers have applied a wet but sterile burn sheet. You know that this is likely​ to: A. decrease the risk of hypothermia. B. increase the risk of hypothermia. C. increase the risk of infection. D. alleviate the​ patient's pain.

increase the risk of hypothermia.

Tests have shown that​ ________ ambulance operators tend to speed up by 10 to 15 miles per hour when the siren is sounding. A. inexperienced B. privately employed C. younger D. experienced

inexperienced

The medications that transplant patients need to take for the rest of their lives to prevent organ rejection also often lead to high susceptibility​ of: A. renal failure. B. infection. C. congestive heart failure. D. ulcers.

infection.

A bulging fontanelle can be a sign​ of: A. hypothermia. B. intracranial pressure. C. epiglottitis. D. shock.

intracranial pressure.

Common signs and symptoms following an injury to a hollow abdominal organ include: A. massive hemorrhage. B. increasing intrathoracic pressures. C. absence of unilateral pulses. D. irritation and peritonitis.

irritation and peritonitis.

A sunken fontanelle may indicate that the​ infant: A. is hypoglycemic. B. has a birth defect. C. is dehydrated. D. has increased intracranial pressure.

is dehydrated.

There are two main types of​ dialysis, hemodialysis and peritoneal dialysis. The main difference between them is​ hemodialysis: A. is usually done at home and peritoneal dialysis is usually done at a special facility. B. is usually done at a special facility and peritoneal dialysis is usually done at home. C. filters the blood and peritoneal dialysis filters the urine. D. filters the urine and peritoneal dialysis filters the blood.

is usually done at a special facility and peritoneal dialysis is usually done at home.

Cold is sometimes used to help control bleeding. When using​ cold, the following guidelines and statements are true except​: A. it will reduce pain. B. it should not be used alone but rather in conjunction with other manual techniques. C. it should not be left in place for more than 20 minutes. D. it should be applied directly to the​ skin; it will not be effective if anything is between the cold agent and the wound.

it should be applied directly to the​ skin; it will not be effective if anything is between the cold agent and the wound.

The anterior fontanelle would be​ palpated: A. laterally on either side of the skull. B. at the midline of the upper back of the skull. C. at the midline of the lower back of the skull. D. just forward of the center of the skull

just forward of the center of the skull

You respond for an imminent delivery of a baby in the breech position. As you examine the​ mother, you see that the umbilical cord is protruding from the opening of the cervix. You realize that the most important care you can provide for this baby is​ to: A. wrap the cord to prevent heat loss. B. provide oxygen to the mother. C. gently push the cord back into the vaginal space. D. keep the baby off of the cord.

keep the baby off of the cord.

The most frequently transplanted organ is​ the: A. liver. B. pancreas. C. heart. D. kidney.

kidney.

Renal failure occurs when​ the: A. liver fails to function properly. B. pancreas stops working. C. adrenal glands stop working. D. kidneys fail to function as required.

kidneys fail to function as required.

The safest way to extinguish a road flare is​ to: A. pick it up and rub it onto the ground. B. douse it with a fire hose. C. let it burn out. D. pick it up and snuff it out with a gloved hand.

let it burn out.

When treating a pediatric​ patient, you should​ NEVER: A. do anything that causes pain. B. assume that the patient is being honest. C. initiate any treatment without parental consent. D. lie to the patient.

lie to the patient.

While transporting a patient who has an indwelling Foley​ catheter, it is important to keep the collection​ bag: A. with the​ patient's caregiver. B. level with the patient. C. higher than the level of the patient. D. lower than the level of the patient.

lower than the level of the patient.

Smoke inhalation can result in all of the following ​except: A. airway swelling and obstruction. B. cardiac arrest. C. lung contusion. D. respiratory arrest.

lung contusion.

You are standing in line waiting to get a prescription filled for your mother. A woman walks in with a young boy in her arms. As she places him in a nearby​ chair, he begins to seize. Your NEXT action should be​ to: A. assess the child for fever. B. maintain an open airway. C. insert an oropharyngeal airway. D. ask the woman if the child has had prior seizures.

maintain an open airway.

When managing a patient who may hurt himself or​ others, you should do all of the following except​: A. watch for sudden changes in the​ patient's behavior. B. make sure only you and the patient are in the room. C. retreat if the patient becomes threatening. D. keep bystanders a safe distance away.

make sure only you and the patient are in the room.

To determine the appropriate depth an EMT should insert a catheter tip into a tracheostomy tube in order to clear a blocked​ airway, the EMT​ should: A. measure against the length of the obturator. B. insert the catheter until the tip is no longer visible. C. measure from the stoma to the carina. D. multiply the internal diameter of the tracheal tube by 10.

measure against the length of the obturator.

As an​ EMT, your BEST clue indicating the possibility of internal bleeding may be the presence​ of: A. ​bruising, swelling, or pain over vital organs. B. ​painful, swollen, or deformed extremities. C. mechanism of injury. D. the absence of a​ tender, rigid, or distended abdomen.

mechanism of injury.

Organs of the right upper quadrant include: A. small intestine, stomach, and spleen. B. pancreas, spleen, and part of the liver. C. most of the liver, spleen, and gallbladder. D. most of the liver, gallbladder, and part of the large intestine.

most of the liver, gallbladder, and part of the large intestine.

You are transporting a resident of a nursing home to the hospital for a complaint of abdominal pain for 3 days. During your​ assessment, you note a tube protruding from the​ patient's nose, which is connected to a bag of​ milky-white fluid. This tube is known as​ a: A. ​J-tube. B. tracheal tube. C. nasogastric tube. D. gastric tube.

nasogastric tube.

After responding to the scene of a patient complaining of difficulty​ breathing, you and your partner determine that the​ patient's condition is not life threatening based on a thorough assessment. You should transport the patient to an appropriate medical facility​ with: A. the emergency lights​ off, but activating the siren when traffic builds. B. both lights and siren activated. C. neither lights nor siren activated. D. the emergency lights​ on, but without activating the siren.

neither lights nor siren activated.

Your patient has a suspected cervical spine injury from falling from a rope swing. His vital signs are pulse​ 62, respirations​ 20, and blood pressure​ 90/56. He has no feeling below his​ mid-chest area but is able to breathe on his own. You should​ suspect: A. neurogenic shock. .B. hypovolemic shock. C. psychogenic shock. D. cardiogenic shock.

neurogenic shock.

You are at the scene of a vehicle collision and have found that there is one patient who is in stable condition. At this​ point, the call can be categorized​ as: A. an unfounded call. B. a​ non-incident. C. no longer a true emergency. D. a refusal.

no longer a true emergency.

You are called for a man who is not acting right. His wife says he was outside working in the garden but​ didn't come in for lunch when he was called. She went to check on him and found him sitting next to a​ stump, confused. You assessment shows a pulse rate of​ 58, blood pressure of​ 186/82, respirations of​ 16, and one of his pupils is dilated. You are unable to detect any signs of trauma and you​ don't see any obvious indication that he has fallen. You should suspect​ a(n): A. closed head injury. B. insecticide poisoning. C. nontraumatic brain injury. D. open head injury.

nontraumatic brain injury.

You have determined that your​ patient, a​ 6-year-old boy, most likely has epiglottitis. He has developed a high fever and appears very ill. You realize that this is a serious illness and you need to handle your patient carefully. You​ should: A. have him lie down with his head elevated. B. provide only dehumidified oxygen. C. give him ice chips to suck on. D. not place anything in his mouth.

not place anything in his mouth.

You are transporting a patient who is experiencing frequent discharges of her automatic implanted cardiac defibrillator​ (AICD). En route to the hospital you are caring for the patient in the back of the ambulance. To protect yourself from being shocked if the defibrillator​ discharges, you​ should: A. limit direct contact with the patient. B. not​ worry, since the defibrillator does not pose a danger to others. C. make sure you do not touch the patient directly. D. wear protective gloves that cannot conduct electricity.

not​ worry, since the defibrillator does not pose a danger to others.

Tracheostomy tubes are prone​ to: A. deflation. B. overheating. C. inflation. D. obstruction.

obstruction.

For a pediatric patient run over by a​ car, you would LEAST​ expect: A. internal chest injuries. B. Fractures of the extremities. C. obvious external damage to the chest and abdomen. D. internal abdominal injuries.

obvious external damage to the chest and abdomen.

Your patient has sustained a serious laceration to his neck. He appears to have lost a lot of blood and you are considering how you will control the bleeding. Your primary treatment should be to place​ a(n): A. ​dry, sterile dressing. B. pressure dressing. C. bulky dressing. D. occlusive dressing.

occlusive dressing.

Your patient is a​ 16-year-old centerfielder on his high school baseball team. He was injured when he and the left fielder collided trying to catch a fly ball. He is disoriented and is unable to tell you what happened. His vital signs are pulse​ 88, blood pressure​ 132/86, respiratory rate​ 16, and pupils equal. As you assess his​ head, you do not see any cuts or​ bleeding, but you feel a​ spongy, depressed area over his left ear. You should suspect​ a(n): A. closed head injury. B. open head injury. C. direct injury. D. skull injury.

open head injury.

The process of combining the patient and​ patient-carrying device into a unit ready for moving and transporting the patient is​ called: A. immobilizing. B. packaging. C. loading. D. wrapping.

packaging.

You respond to the scene of a 50yearold male complaining of severe abdominal pain. He has a history of alcohol and drug abuse. His vital signs are stable and he presents with epigastric pain that radiates to the back. He has guarding and point tenderness in the upper quadrants. You suspect: A. pancreatitis. B. cholecystitis. C. gastroesophageal reflux disease (GERD). D. myocardial infarction.

pancreatitis.

The term that means of or pertaining to a patient who has yet to reach puberty​ is: A. geriatric. B. pediatric. C. characteristic. D. physiologic.

pediatric

Blood at the meatus of the penis (opening of the urethra) is a sign of: A. spinal fracture or injury. B. pelvic dislocation syndrome. C. pelvic trauma/fracture. D. priapism

pelvic trauma/fracture.

You are caring for an unconscious​ 80-year-old trauma patient. You note the patient has dentures. You first action in managing this patient should​ be: A. perform a jaw thrust as needed. B. remove the dentures. C. prepare to insert an oral airway. D. tilt the head to a sniffing position.

perform a jaw thrust as needed.

Most organs of the abdomen are enclosed within the: A. peritoneum. B. midline. C. retroperitoneal. D. extraperitoneal.

peritoneum.

You are called to a residence for a 48yearold male patient. He is lying in bed, groaning in pain and curled into a fetal position. His blood pressure is 88/50, pulse 136, and respiratory rate of 32. His wife states that he complained of lower right abdominal pain for several days that got progressively worse, until about an hour ago when it became suddenly unbearable. The wife states the husband had been refusing to see a doctor, but she finally called 911. You suspect appendicitis and are concerned about the subsequent onset of: A. toxic shock. B. peritonitis. C. splenic rupture. D. renal colic.

peritonitis.

Signs that identify the hazardous material contained within a transport vehicle are​ called: A. indicators. B. warnings. C. panels. D. placards.

placards.

Your pregnant patient is in active labor. She has been lying on her back throughout her labor and is experiencing signs and symptoms of shock. Her vital signs are pulse rate​ 118, respirations​ 22, blood pressure​ 96/62, and skin cool and diaphoretic. The BEST way to treat her is​ to: A. give her some fluids to drink to help elevate her blood pressure. B. cover her up with a blanket to maintain body heat. C. place a rolled blanket under her left hip to displace the uterus. D. elevate her legs to increase the blood return to the heart.

place a rolled blanket under her left hip to displace the uterus.

A patient with a medical history of sickle cell anemia is complaining of chest pain and shortness of breath. The patient is breathing 26 times a minute in​ short, shallow respirations.​ However, the​ patient's oxygen saturation via pulse oximetry is​ 100% on room air. The best approach regarding supplemental oxygen is​ to: A. place the patient on a simple face mask. B. do​ nothing; no supplemental oxygen is necessary. C. place the patient on a nonrebreather mask. D. place the patient on a nasal cannula.

place the patient on a nonrebreather mask.

When delivering a​ baby, you should do all of the following except​: A. place your fingers in the armpit to assist with delivery of the shoulders. B. have someone stay at the​ mother's head if possible. C. position your gloved hands at the vaginal opening. D. suction the mouth​ first, then the nose.

place your fingers in the armpit to assist with delivery of the shoulders.

You find a middle-aged male is sitting against a wall in obvious distress. The patient appears to be extremely short of breath and has an open wound to his chest that is making a sucking sound. You should IMMEDIATELY: A. cover the wound with an occlusive dressing. B. place a trauma dressing over the wound. C. place your gloved hand over the wound. D. place the patient on high-concentration oxygen.

place your gloved hand over the wound.

You are assessing a pregnant woman whose chief complaint is vaginal bleeding. She is 8 months pregnant and has​ moderate, bright red bleeding. She says that her doctor was concerned about the location of the placenta. This condition is known​ as: A. placenta previa. B. placenta disruption. C. abruptio placenta. D. unstable placenta.

placenta previa.

Stocking pattern burn marks on a pediatric patient are an indication​ of: A. a circulatory disorder. B. hypertensive crisis. C. hypotension. D. possible physical abuse

possible physical abuse

A patient with renal disease who is currently on dialysis and missed an appointment may present with dangerously high levels of the​ electrolyte: A. potassium. B. phosphorous. C. sodium. D. calcium.

potassium.

When deciding whether or not to drive to the hospital with lights and siren activated for any​ patient, the driver should​ consider: A. the​ patient's preference for lights and siren. B. how many calls are waiting for an available ambulance. C. potential risks of making the​ patient's condition worse. D. the​ patient's past medical history.

potential risks of making the​ patient's condition worse.

An​ EMT's BEST defense against potential legal liability​ is: A. practicing excellent prehospital care. B. relying on Good Samaritan immunity. C. documenting as little as possible on the run sheet. D. purchasing medical malpractice insurance.

practicing excellent prehospital care.

You respond to a call for a patient in active labor with her second child. Your interview with the patient shows that she is 40 weeks pregnant and has been in active labor for several hours. You determine that her vital signs are all within normal limits. After your physical​ exam, you determine the baby is crowning. You should​ next: A. begin transport and plan to deliver in the ambulance. B. delay delivery until arrival at the hospital. C. contact medical direction for orders. D. prepare to deliver the baby​ on-scene.

prepare to deliver the baby​ on-scene.

You are at a​ friend's birthday party with people of all ages. There is an​ 11-month-old boy, just learning to stand​ up, who has fallen. He was holding on to the edge of a table and he toppled over. He is crying hard and his mother is trying to console him. She asks if you would mind checking him​ out, and you agree to take a look at him. You notice that his anterior fontanelle is bulging. This is most likely caused​ by: A. the bones of the head not fusing properly. B. an infection causing intracranial pressure. C. fluid loss from dehydration. D. pressure built up because of his crying

pressure built up because of his crying

When you ask an adolescent patient about drug use or possible​ pregnancy, it is best to ask the​ patient: A. in front of law enforcement. B. privately. C. in front of peers. D. in front of parents.

privately.

As an​ EMT, it is important that you be able to do all of the following except​: A. deal with a wide variety of emotions and behaviors. B. ensure your safety and that of others at the scene. C. control your own emotions when dealing with patients who say upsetting things to you. D. provide counseling and advice to patients with personal problems.

provide counseling and advice to patients with personal problems.

Management of the suicidal patient includes all of the following except​: A. psychoanalysis of the patient by the EMT. B. a SAMPLE history. C. communication. D. a physical examination.

psychoanalysis of the patient by the EMT.

Dialysis patients who have missed an appointment may present with signs of​ ________, which is a similar presentation to​ ________. A. shortness of​ breath; pneumonia B. neurological​ disturbances; stroke C. pulmonary​ edema; congestive heart failure D. chest​ pain; acute myocardial infarction

pulmonary​ edema; congestive heart failure

When the gallbladder is diseased, the pain is not only felt in the right upper quadrant (RUQ) but also in the right shoulder. This is known as: A. referred pain. B. parietal pain. C. tearing pain. D. visceral pain.

referred pain

Pain that the patient feels in a body part or area of the body that has nothing to do with a diseased organ is termed: A. abdominal pain. B. retroperitoneal pain. C. epigastric pain. D. referred pain.

referred pain.

Your patient was diagnosed with cholecystitis (gallbladder inflammation) three days ago. The patient now presents with nausea, vomiting, and pain in the right shoulder. The pain in the shoulder can be classified as: A. somatic pain. B. pancreatic pain. C. referred pain. D. visceral pain.

referred pain.

All of the following are signs of adequate breathing and circulation in the newborn​ except: A. vigorous crying. B. cyanosis of only the hands and feet. C. relaxation of the extremities. D. heart rate greater than 100.

relaxation of the extremities.

Your patient was ejected from his motorcycle when he struck a deer. He is drowsy and unable to communicate clearly. You determine that the helmet he is wearing will not allow you to securely immobilize the patient to the long spine board. You​ should: A. tip his head back to gain neutral alignment. B. remove the helmet to better manage proper alignment. C. place towels on either side of the helmet to stabilize it. D. pad under his shoulders to straighten his neck.

remove the helmet to better manage proper alignment.

You are called for a​ 6-year-old girl who is not well. Her mother says that the child has been sick for several days but today she has had trouble keeping her daughter awake. You note that the​ girl's skin temperature is high and she has a skin rash. Given the finding of high​ fever, you should​ NEXT: A. use rubbing alcohol to cool the patient. B. submerge the child into cold water. C. remove the​ patient's heavier clothing without exposing the patient to hypothermic conditions. D. cover the patient with a towel soaked in cold wate

remove the​ patient's heavier clothing without exposing the patient to hypothermic conditions.

You are called to the scene of a patient with abdominal pain. Upon arrival, you find a 38yearold Asian man on the floor next to his desk writhing in pain. He is complaining of severe rightsided flank pain. His blood pressure is 140/90, pulse 100, and his skin is ashen and diaphoretic. You suspect: A. Mittelschmerz. B. renal colic. C. cholecystitis. D. myocardial infarction.

renal colic.

You suspect that your pediatric patient is being abused. To satisfy your legal​ obligation, you​ must: A. email your EMS supervisor of your concerns. B. remain quiet to avoid committing a HIPAA violation. C. report your suspicions to the medical staff at the receiving hospital. D. press charges against the caregiver.

report your suspicions to the medical staff at the receiving hospital.

You and your partner respond to the scene of a private residence for a patient who is a known diabetic patient. The patient is pacing and cursing as you approach. You​ should: A. request assistance from law enforcement personnel. B. immediately restrain the patient. C. approach the patient and firmly order him to sit with his hands in his lap. D. obtain a blood glucose level right away.

request assistance from law enforcement personnel.

In infants and​ children, retraction of the​ muscles, grunting, and nasal flaring are signs​ of: A. infection. B. respiratory distress. C. normal breathing. D. hypoperfusion.

respiratory distress.

The parents of a child who is on an artificial ventilator call EMS for help when the ventilator begins to malfunction. Your primary care to this patient will NOT​ include: A. ensuring an open airway. B. transporting the patient. C. restarting the machine. D. artificially ventilating the patient with a pocket or​ bag-valve mask

restarting the machine.

An area near a hazardous materials incident that is on the same level as the​ incident, but​ upwind, is a good location​ for: A. safe zone. B. danger zone. C. hot zone. D. treatment zone.

safe zone.

Some drawbacks to using sterile aluminum foil as an occlusive dressing include: A. a flutter valve is difficult to create. B. foil cannot create an airtight seal. C. sterility cannot be ensured unless the materials were autoclaved. D. skin lacerations may occur from the sharp edges.

skin lacerations may occur from the sharp edges.

As you care for an elderly patient experiencing a serious medical​ condition, you would expect any deterioration to take​ place: A. slowly and steadily. B. rapidly. C. ​rapidly, with a later shift to slowly. D. slowly, with a later shift to rapidly.

slowly and steadily.

Because of abnormally shaped​ hemoglobin, sickle cell anemia​ (SCA) patients may occasionally experience​ ________, causing a blockage of small blood vessels. A. clumping B. aggregation C. clotting D. sludging

sludging

Adolescent patients will NOT likely cooperate if​ you: A. express empathy. B. are honest with them. C. speak down to them. D. advocate for them.

speak down to them.

Inexperienced drivers tend to​ ________ when they hear a siren approaching. A. speed up B. pull to the right C. stop D. pull to the left

speed up

When responding to an emergency in an authorized vehicle while functioning as an EMT for an authorized emergency​ service, the actions you take while driving​ are: A. covered by your​ service's insurance carrier. B. subject to laws that govern all drivers. C. not likely to result in an investigation in the event of a collision. D. ​allowable, provided you do not exceed 15 mph above the posted speed limit.

subject to laws that govern all drivers.

When a chemical substance is taken for reasons other than therapeutic​ use, this action is​ called: A. addiction. B. tolerance. C. psychological dependence. D. substance abuse.

substance abuse.

You are called to the scene of a​ 14-year-old female patient who is unresponsive. Her mother states she has been teased and bullied at school and she suspects that she tried to commit suicide by taking her pain pills. The patient is unresponsive to painful​ stimuli, with agonal​ respirations, and has vomited. She has a weak carotid pulse. Your first action is​ to: A. administer oxygen by nonrebreather mask​ (NRB). B. administer an oropharyngeal airway​ (OPA). C. suction the​ patient's airway. D. administer oxygen by​ bag-valve mask​ (BVM).

suction the​ patient's airway.

You have been called for a young female in labor. She is lying on the floor in obvious distress from the labor pains. While you are assessing her for​ crowning, your partner is getting a set of vital signs. She tells you that she is getting dizzy and nauseated. Vital signs are pulse rate​ 120, respiratory rate​ 22, and blood pressure​ 98/62. You should be concerned that she​ has: A. supine hypotensive syndrome. B. contracted food poisoning. C. ​Braxton-Hicks contractions. D. placenta previa.

supine hypotensive syndrome.

Prehospital management of all the urinary system problems of the kidney transplant patient by the EMT can be characterized​ by: A. intensive lab testing. B. supportive care. C. varied pharmacological interventions. D. very invasive techniques.

supportive care.

You suspect a patient who has been having a difficult time controlling the bleeding following a small laceration to the foot may have a history​ of: A. taking blood thinners. B. sickle cell anemia. C. poorly controlled diabetes. D. high blood pressure.

taking blood thinners.

You are assessing a​ 67-year-old male patient who is experiencing a dissecting abdominal aneurysm.​ Typically, such a patient will describe the pain​ as: A. dull. B. achy. C. tearing. D. sharp.

tearing.

Peritoneal dialysis allows patients to dialyze at home​ through: A. the abdomen. B. an AV fistula. C. the urethra. D. a dialysis machine.

the abdomen.

A new EMT who is treating a suspected femur injury asks his partner, "How much traction should I pull?" The partner's BEST reply is which of the following? A. No traction splint applied in the field pulls true traction; they must pull 20 pounds of countertraction. B. The amount of traction applied should be roughly 10% of the patient's body weight and not exceed 15 pounds. C. The amount of traction applied should be 15 pounds. D. Pull enough traction to give the patient some relief from the pain.

the amount of traction applied should be roughly 10% of the patient's body weight and not exceed 15 pounds.

You are called to a home where a family has been enjoying a Labor Day picnic and swim party. A​ 2-year-boy slipped to the bottom of the pool unnoticed. By the time you​ arrive, the child has been removed from the water and family members are administering CPR. You assess the child and note that he has a weak pulse and is trying to breathe on his own. You initiate transport and continue providing positive pressure ventilations. You are also very concerned​ about: A. rising blood pressure. B. blood loss. C. the child being hypothermic. D. crying.

the child being hypothermic.

You are assessing an​ 83-year-old female who is having hip pain. When you count her pulse you notice that the rate is irregularly irregular. What is your treatment for this​ patient? A.Check distal​ circulation, sensory, and motor​ sensation, apply a pelvic​ wrap, place on a traction​ splint, and transport the patient. B. The irregularity is not a reason for concern in itself. C. Contact medical control for permission to have the patient transported by helicopter from the scene. D. Alert paramedics to respond for a possible cardiac patient.

the irregularity is not a reason for concern in itself.

A retraction relates​ to: A. skin color. B. tone and position. C. the work of breathing. D. emotional state

the work of breathing.

48. You are treating a 5-year-old for extensive burns. You know that burns pose a greater risk to infants and children. The reason for this is: a. the parents' interference with your assessment and treatment can delay your response. b. the child will likely be upset and crying and therefore hard to evaluate. c. child abuse is usually the reason for the burns. d. their body surface area is greater in relation to their total body size.

their body surface area is greater in relation to their total body size.

When arriving at the location of a motor vehicle collision on an interstate​ highway, you should position the ambulance so​ that: A. there is a safe area between the damaged vehicles and traffic behind the scene. B. there is a fire truck between you and oncoming traffic. C. you are nearest to the command post for easy communications with the incident commander. D. there is a safe area beyond the damaged vehicles.

there is a safe area beyond the damaged vehicles.

You are called to a garden center where four shoppers have collapsed after a section of shelving fell and dumped hazardous products on the floor. When you arrive on the​ scene, you​ should: A. think​ hazmat, dress in a Tyvek​ suit, access the scene to identify the spilled​ product, secure the MSDS​ sheets, and remove victims to fresh air. B. secure the scene​ quickly, triage each​ patient, and move to fresh air. C. think​ hazmat, secure the scene by limiting​ access, isolate the possible contaminated from the​ noncontaminated, and request hazmat response. D. think​ hazmat, dress in a Tyvek​ suit, secure the scene by limiting​ access, isolate the possible contaminated from the​ noncontaminated, and request hazmat response.

think​ hazmat, secure the scene by limiting​ access, isolate the possible contaminated from the​ noncontaminated, and request hazmat response.

The sensation felt when palpating an​ arterial-venous fistula or shunt is known as​ a: A. frill. B. thrill. C. bruit. D. fluttering.

thrill.

You have responded to a woman who has possible premature labor. She is 8 months pregnant and is experiencing labor pains. During your​ assessment, you note that the baby is not yet crowning. You need to determine whether delivery is imminent or not. You should​ next: A. time her contractions. B. check the level of the fundus. C. complete a set of vital signs. D. check for cervix dilation

time her contractions.

You are dispatched to a private residence for a​ 52-year-old African American male sitting in his living room complaining of chest pain. During your​ assessment, he discloses that he has the sickle cell trait. Because of this you​ should: A. administer​ high-concentration oxygen. B. be prepared to treat for shock if he also has a high fever. C. monitor for signs of inadequate respiration. D. treat the patient as any other​ 52-year-old with chest pain.

treat the patient as any other​ 52-year-old with chest pain.

When several agencies work independently but​ cooperatively, rather than one agency exercising control over the​ others, this is​ called: A. single incident command. B. NIMS. C. ICS. D. unified command.

unified command.

You are in charge of your​ department's personnel at the scene of a​ mass-casualty incident. As the incident​ progresses, you request additional resources and a neighboring department assists you. As part of this​ department's response, a battalion chief responds to oversee operations. At the command​ post, you and the battalion chief work together to manage the incident. This cooperative management effort is known​ as: A. dual command. B. unified command. C. redundant command. D. incident command.

unified command.

The muscular organ in which a baby develops during pregnancy is called​ the: A. placenta. B. fetal membrane. C. uterus. D. amniotic sac.

uterus.

When a patient has a serious open wound to the​ neck, you are concerned about the possibility of an air embolism. The reason an air embolism can occur is because​ of: A. the negative pressure in the chest. B. the higher pressure in the vessels of the neck. C. damage to the trachea. D. vessel pressure that is lower than atmospheric pressure.

vessel pressure that is lower than atmospheric pressure.

Pain that originates in an organ, such as the intestines, is called ________ pain. A. referred B. acute C. visceral D. parietal

visceral

The​ "from the​ doorway" impression of the child that is part of the Pediatric Assessment Triangle relies primarily​ on: A. taking of vital signs. B. percussion. C. palpation. D. visual assessment.

visual assessment.

A​ 17-year-old girl was injured when her car was struck from behind while she was stopped at a red light. She is complaining of a headache with neck and back pain. You suspect she has sustained​ a(n): A. compression fracture. B. extension injury. C. distraction injury. D. whiplash injury.

whiplash injury.

Operators of emergency vehicles must drive with the safety of others in mind. This is known as​ driving: A. defensively. B. with emergency privilege. C. with due regard. D. as a public servant.

with due regard.

A pillow is frequently used to splint an ankle or foot injury. It is effective, rapid, and can be used for most patients. Its main weakness is: A. you are not immobilizing the knee and the joint adjacent to the ankle. B. it is hard to access distal pulses after application. C. you might not have a pillow on your ambulance. D. it requires three people to apply.

you are not immobilizing the knee and the joint adjacent to the ankle.

You are caring for an injured child. The​ child's parent asks​ you, "Will she be​ okay?" What should you tell the​ parent? A. ​"We are doing everything we can for​ her." B. ​"Of course she will​ be." C. ​"The next hour is​ critical." D. ​"I'm not allowed to​ say

​"We are doing everything we can for​ her."

Your unresponsive adult patient has a head injury and inadequate breathing. Your​ bag-valve-mask ventilations should be delivered at a rate​ of: A. ​16-20 breaths per minute. B. ​20-24 breaths per minute. C. ​24-30 breaths per minute. D. ​10-12 breaths per minute.

​10-12 breaths per minute.

A respiratory rate of less than​ ________ and greater than​ ________ in cases of trauma are criteria for immediate transportation to a trauma center according to the CDC physiologic guidelines. A. ​10; 29 B. ​12; 20 C. ​8; 32 D. ​5; 45

​10; 29

Which patient would most likely benefit from the administration of activated​ charcoal? A. ​8-year-boy who has a headache from carbon monoxide poisoning B. ​14-year-old female who overdoses on her​ grandfather's cardiac medication C. ​45-year-old patient suffering from organophosphate poisoning D. ​18-year-old patient who is passed out from drinking too much beer

​14-year-old female who overdoses on her​ grandfather's cardiac medication

Of the following​ patients, which injury is the highest priority to receive​ bag-valve-mask ventilations? A. ​25-year-old female victim of battery who is awake but complains of a headache and has bloody fluid draining from her nose and left ear B. ​15-year-old female who was ejected from a​ vehicle, struck her head on a​ tree, and displays decerebrate movements in response to painful stimuli C. ​25-year-old male who regained consciousness one or two minutes after being struck on the head by a baseball bat and is now asking repetitive questions D. ​70-year-old male who struck his head when he fell in the parking​ lot, has a large laceration on his​ forehead, and is disoriented

​15-year-old female who was ejected from a​ vehicle, struck her head on a​ tree, and displays decerebrate movements in response to painful stimuli

Who is the most susceptible to​ hypothermia? A. ​21-year-old near-drowning patient on a warm spring day B. ​55-year-old male patient who sprained his knee snow skiing C. ​80-year-old male patient on the ground who slipped and has a hip fracture D. ​55-year-old male patient who sprained his knee snow skiing

​21-year-old near-drowning patient on a warm spring day

Which of the following patients would be considered to be at the greatest risk for​ suicide? A. ​44-year-old woman who has just recovered from a serious illness B. ​22-year-old man who has lost his job and is getting divorced C. ​Married, 30-year-old man who has just learned that his employer is transferring him to a different state D. ​35-year-old female who has a child with a serious illness

​22-year-old man who has lost his job and is getting divorced

You respond to a major automobile collision involving multiple patients. Which of the following patients would receive Priority 1​ transport? A. ​6-year-old with lower leg fracture and normal vital signs B. ​25-year-old with abdominal bruising and signs of shock C. ​68-year-old with mild respiratory distress and seat belt burns D. ​45-year-old with no vital signs

​25-year-old with abdominal bruising and signs of shock

Approximately how many Americans are currently on some type of​ dialysis? A. ​200,000 B. ​100,000 C. ​400,000 D. ​50,000

​400,000

For which of the following patients would the use of a short spine immobilization device be​ appropriate? A. Unresponsive driver who was found in the​ driver's seat of a pickup truck that crashed into a tree B. ​52-year-old female who crashed her vehicle into the front of a convenience store and who is​ awake, oriented, and complaining of neck pain C. ​77-year-old female who fell down a flight of stairs and is in an awkward position lying on her side on the landing D. ​15-year-old male who struck a curb with his motorized scooter and flipped over the handlebars

​52-year-old female who crashed her vehicle into the front of a convenience store and who is​ awake, oriented, and complaining of neck pain

Approximately what percentage of United States dialysis patients treat themselves at​ home? A. ​16% B. ​8% C. ​25% D. ​50%

​8%

You are treating an unresponsive homeless patient found in an alley. During your assessment you notice bruising behind both​ ears, which is known as which of the​ following? A. ​Battle's signs B. ​Soldier's signs C. ​Warrior's signs D. Fatigue signs

​Battle's signs

What clinical finding is MOST suggestive of toxic smoke​ inhalation? A. Tachypnea B. ​Black-colored sputum C. Swollen tongue D. Altered mental status

​Black-colored sputum

You are en route to the trauma center with a patient who was injured when he was thrown from a horse. You suspect multiple trauma injuries including a closed head injury. You will monitor his mental status using the Glasgow Coma Scale​ (GCS) per your protocols. Which of the following will you​ check? A. Motor​ response, verbal​ response, and attentiveness B. ​Eye-opening, motor​ response, and verbal response C. Verbal​ responses, motor​ skills, and mental status D. Speech​ patterns, motor​ patterns, and mental status

​Eye-opening, motor​ response, and verbal response

Which of the following is most likely to keep a rescuer safe and dry during a water rescue through broken​ ice? A. Ladder B. ​Flat-bottomed aluminum boat C. Ring buoy D. ​Jet-ski

​Flat-bottomed aluminum boat

What are the three elements of the Revised Trauma​ Score? A. ​GCS, systolic blood​ pressure, and respiratory rate B. ​GCS, systolic blood​ pressure, and pulse rate C. ​GCS, pulse​ rate, and respiratory rate D. Level of​ consciousness, systolic blood​ pressure, and pulse rate

​GCS, systolic blood​ pressure, and respiratory rate

Your patient is a conscious​ 4-year-old female who has ingested a medication prescribed for her grandfather. To determine appropriate​ treatment, which of the following information is NOT necessary to provide to medical​ control? A. ​Grandfather's name and the name of the physician who prescribed the medication B. Any home remedies attempted by the​ patient's family C. ​Patient's vital signs and the time the medication was taken D. ​Patient's weight and the amount of medication taken

​Grandfather's name and the name of the physician who prescribed the medication

You are dispatched to a multiple vehicle collision on a busy interstate highway. Your crew identifies a critical patient entrapped in a small sedan with significant intrusion into the occupant area on the front and left side. One of your crew​ members, dressed in fully protective​ gear, volunteers to enter the vehicle to begin assessment and treatment. Given that access to the patient is​ limited, you tell him to concentrate on assessing which of the following parts of the​ patient's body? A. ​Head, chest, and upper extremities B. ​Head, chest, and torso C. ​Head, posterior​ torso, and lower extremities D. ​Torso, pelvis, and lower extremities

​Head, chest, and torso

Which of the following is NOT indicated in the management of a patient in​ shock? A. Delaying a detailed exam until en route to the hospital B. ​On-scene spinal​ precautions, if indicated C. ​High-speed ambulance transportation D. Minimizing​ on-scene time

​High-speed ambulance transportation

Which of the following is NOT an acceptable method or adjunct in restraining a​ patient? A. Restraining the patient​ face-up B. Placing a surgical mask over the​ patient's face to prevent spitting C. Securing all four limbs with approved restraint devices D. ​Hog-tying the patient

​Hog-tying the patient

Because of the extra equipment now placed on ambulances for specialty​ rescue, advanced life​ support, and hazardous materials​ operations, their gross vehicle weight has been easily exceeded in some communities. This has necessitated introduction of a​ ________ truck chassis built for rugged durability and large storage and work areas. A. ​Medium-duty B. Type I C. Type III D. Type II

​Medium-duty

When auscultating the lateral lung fields of an​ infant, what is the preferred​ position? A. Posterior B. ​Mid-axillary C. Apical D. Inferior

​Mid-axillary

Which of the following describes the proper position of the​ patient's head for spinal​ immobilization? A. Chin tilted upward for airway maintenance B. The​ "sniffing" position C. ​Neutral, in-line​ "eyes forward" position D. Stabilized in position found

​Neutral, in-line​ "eyes forward" position

Which of the following MOST affects the extent of damage a poison does to the​ body? A. ​Patient's genetics,​ height, and medications taken B. ​Patient's nutritional​ status, genetics, and strength of the immune system C. ​Patient's age,​ weight, and general health D. ​Patient's race,​ age, and medications taken

​Patient's age,​ weight, and general health

When restraining a​ patient, which of the following is NOT a​ consideration? A. How to position the patient B. ​Patient's informed consent C. Number of people available to carry out the required actions D. ​Patient's size and strength

​Patient's informed consent

Simple Triage and Rapid Treatment​ (START) is a system that relies on some simple commands and what three physiologic​ parameters? A. ​Respiration, pain, and mental status B. ​Respiration, sensory, and motor status C. ​Circulation, sensory, and motor status D. ​Respiration, pulse, and mental status

​Respiration, pulse, and mental status

What are three elements of successful trauma care that field practitioners can use which will ultimately translate into greater rates of​ survival? A. Physiological​ determinants, anatomic​ criteria, and mechanism of injury B. ​Lights, sirens, and diesel C. Ground​ ambulances, air​ helicopters, and trauma centers D. ​Teamwork, timing, and transport

​Teamwork, timing, and transport

Which of the following is the correct sequence for securing the straps on a long spine​ board? A. ​Head, legs, torso B. ​Legs, torso, head C. ​Torso, legs, head D. ​Head, torso, legs

​Torso, legs, head

The correct terminology used for patients whose kidneys are damaged to the point where they require dialysis to survive​ is: A. ​end-stage renal disease. B. hepatic failure. C. acute renal failure. D. terminal kidney disease.

​end-stage renal disease.

Two chronic medical conditions that dialysis patients frequently have in addition to kidney failure are​ ________ and​ ________. A. ​hypertension; diabetes B. high​ cholesterol; cerebral aneurysms C. blood​ clots; COPD D. heart​ failure; stroke

​hypertension; diabetes

Volatile chemicals are agents that are able to change easily from a​ ________ form to a​ ________ form. A. ​liquid; gas B. ​gas; liquid C. ​solid; liquid D. ​solid; vapor

​liquid; gas

Hemodialysis is used to help the kidneys filter​ ________ and remove excess​ ________. A. ​blood; cholesterol B. ​toxins; fluids C. ​urine; poisons D. ​electrolytes; hormones

​toxins; fluids


Ensembles d'études connexes

CH 44 Assessment and Management of Patients with Biliary Disorders

View Set

Chapter 9 ( Development Across the Lifespan)

View Set